You are on page 1of 203

TABLE OF CONTENTS

BIOCHEMISTRY, 3
ANATOMY, 37
PHYSIOLOGY, 68
MICROBIOLOGY, 97
PATHOLOGY, 135
PHARMACOLOGY, 169

2
BIOCHEMISTRY

3
QUESTION ANSWER EXPLANATION

1. All mucopolysaccharidoses A A. Hunter syndrome


are autosomal recessive,
EXCEPT? Hunter syndrome is an X-linked recessive
disease, not autosomal recessive.
A. Hunter syndrome
B. Hurler syndrome All mucopolysaccharidoses are autosomal
C. Sanfilippo syndrome recessive except for Hunter syndrome.
D. Sly syndrome
References:
● Ferrier, D. (2017). Lippincott Illustrated Reviews:
Biochemistry (7th ed., p. 164). Lippincott Williams
and Wilkins.
● Le, T., Bhushan, V., & Sochat, M. (2023). First Aid
for the USMLE Step 1 2022, Thirty Third Edition
(33rd ed., pp. 58-59). McGraw-Hill Education.

2. Which of the following D D. Heparan sulfate


substrates accumulate in Hurler
syndrome? In Hurler syndrome, the degradation of
dermatan sulfate and heparan sulfate is
A. Sphingomyelin affected, due to a deficiency of the enzyme
B. Ceramide trihexoside α-L-iduronidase.
C. GM2 ganglioside
D. Heparan sulfate Disease Biochemical
imbalance

Niemann-Pick Sphingomyelin
disease accumulation

Fabry disease Ceramide


trihexoside
accumulation

Tay-Sachs disease GM2 ganglioside


accumulation

References:
● Ferrier, D. (2017). Lippincott Illustrated Reviews:
Biochemistry (7th ed., p. 164). Lippincott Williams
and Wilkins.
● Le, T., Bhushan, V., & Sochat, M. (2023). First Aid
for the USMLE Step 1 2022, Thirty Third Edition
(33rd ed., p. 86). McGraw-Hill Education.

3. Hunter syndrome results D D. Iduronate-2-sulfatase


from the deficiency of which
enzyme? In Hunter syndrome, the degradation of
dermatan sulfate and heparan sulfate is
A. Galactocerebrosidase affected, due to a deficiency of the enzyme
B. Glucocerebrosidase Iduronate-2-sulfatase.
C. α-L-iduronidase
D. Iduronate-2-sulfatase
Disease Enzyme deficiency

4
Krabbe disease Galactocerebrosidase
deficiency

Gaucher disease Glucocerebrosidase


deficiency

Hurler syndrome α-L-iduronidase


deficiency

References:
● Ferrier, D. (2017). Lippincott Illustrated Reviews:
Biochemistry (7th ed., p. 164). Lippincott Williams
and Wilkins.
● Le, T., Bhushan, V., & Sochat, M. (2023). First Aid
for the USMLE Step 1 2022, Thirty Third Edition
(33rd ed., p. 86). McGraw-Hill Education.

4. This is a type I collagen C C. Osteogenesis imperfecta


defect that results from an
inability to form triple helices: Most cases of osteogenesis imperfecta (OI)
are caused by dominant mutations to the
A. Menkes disease genes that encode the α1 or α2 chains in type
B. Marfan syndrome 1 collagen. The resultant, structurally
C. Osteogenesis abnormal α chains prevent the formation of the
imperfecta required triple-helical conformation.
D. Cystic fibrosis
● Menkes disease results from decreased
collagen cross-linking due to decreased
lysyl oxidase activity.
● Marfan syndrome results from defective
fibrillin
● Cystic fibrosis results from an impaired
ATP-gated chloride channel.

References:
● Ferrier, D. (2017). Lippincott Illustrated Reviews:
Biochemistry (7th ed., p. 49). Lippincott Williams and
Wilkins.
● Le, T., Bhushan, V., & Sochat, M. (2023). First Aid
for the USMLE Step 1 2022, Thirty Third Edition
(33rd ed., p. 710). McGraw-Hill Education.

5. Glycolysis occurs in what B B. Cytoplasm


part of the cell?
Other metabolic processes occurring in the
A. Cell membrane cytoplasm:
B. Cytoplasm ● Hexomonophosphate shunt
C. Mitochondria ● Cholesterol synthesis
D. Nucleus ● Synthesis of proteins (ribosomes)
● Fatty acid synthesis

References:
● Ferrier, D. (2017). Lippincott Illustrated Reviews:
Biochemistry (7th ed., p. 98). Lippincott Williams and
Wilkins.

5
● Le, T., Bhushan, V., & Sochat, M. (2023). First Aid
for the USMLE Step 1 2022, Thirty Third Edition
(33rd ed., p. 72). McGraw-Hill Education.

6. In the well-fed state, insulin A A. Increased


is:
During the well-fed state, there are
A. Increased decreased levels of glucagon and elevated
B. Decreased levels of insulin (which may occur after a
C. Unchanged carbohydrate-rich meal).
D. None of the above
Reference:
Ferrier, D. (2017). Lippincott Illustrated Reviews:
Biochemistry (7th ed., p. 100). Lippincott Williams and
Wilkins.

7. During fasting, glucagon is: A A. Increased

A. Increased During fasting, there are elevated levels of


B. Decreased glucagon and low levels of insulin.
C. Unchanged
D. None of the above Reference:
Ferrier, D. (2017). Lippincott Illustrated Reviews:
Biochemistry (7th ed., p. 100). Lippincott Williams and
Wilkins.

8. Which of the following B B. Alzheimer disease


diseases is NOT caused by
prions? In Alzheimer disease, normal proteins, after
abnormal chemical processing, take on a
A. Creutzfeldt-Jakob unique conformational state that leads to the
disease formation of neurotoxic amyloid β peptide (Aβ)
B. Alzheimer disease assemblies consisting of β-pleated sheets.
C. Kuru
D. Mad cow disease In transmissible spongiform
encephalopathies (CJD, kuru, mad cow
disease), the infective agent is an altered
version of a normal prion protein that acts as a
template for converting normal protein to the
pathogenic conformation.

Reference:
Ferrier, D. (2017). Lippincott Illustrated Reviews:
Biochemistry (7th ed., p. 22). Lippincott Williams and
Wilkins.

9. Maple syrup urine disease C C. Branched-chain α-keto acid dehydrogenase


(MSUD) results from the
deficiency of which of the MSUD is due to the deficiency of branched-
following enzymes? chain α-keto acid dehydrogenase. In this
disease, there are increased levels of
A. Fumaryl-acetoacetate branched-chain α-amino acids and their α-keto
hydrolase analogs in plasma and urine.
B. Methylmalonyl CoA
mutase Disease Enzyme deficient
C. Branched-chain α-keto
acid dehydrogenase

6
D. Cystathionine synthase Tyrosinemia type I Fumaryl-acetoacetate
hydrolase

Methylmalonic Methylmalonyl CoA


aciduria mutase

Classic form of Cystathionine


homocystinuria synthase

Reference:
Ferrier, D. (2017). Lippincott Illustrated Reviews:
Biochemistry (7th ed., p. 269). Lippincott Williams and
Wilkins.

10. Which of the following is A A. Fabry disease


caused by a deficiency of
alpha-galactosidase? Fabry disease is caused by a deficiency of
alpha-galactosidase.
A. Fabry disease
B. Tay-Sachs disease Disease Enzyme deficient
C. Krabbe disease
D. Gaucher disease Tay-Sachs Hexosaminidase A
disease

Krabbe disease Galactocerebrosidase

Gaucher disease Glucocerebrosidase

Reference:
Le, T., Bhushan, V., & Sochat, M. (2023). First Aid for
the USMLE Step 1 2022, Thirty Third Edition (33rd ed.,
p. 86). McGraw-Hill Education.

11. This occurs when calorie C C. Marasmus


deprivation is relatively greater
than the reduction in protein: Marasmus occurs when calorie deprivation is
relatively greater than the reduction in protein.
A. Kwashiorkor Typical symptoms include arrested growth,
B. Cachexia extreme muscle wasting and depletion of
C. Marasmus subcutaneous fat (emaciation), weakness, and
D. Rickets anemia.
● Kwashiorkor occurs when protein
deprivation is relatively greater than the
reduction in total calories.
● Cachexia is a wasting disorder that is
seen with several chronic diseases.
● Rickets is the softening and weakening of
bones due to prolonged vitamin D
deficiency.
Reference:
Ferrier, D. (2017). Lippincott Illustrated Reviews:
Biochemistry (7th ed., p. 369). Lippincott Williams and
Wilkins.

7
12. Bilateral pitting edema is A A. Kwashiorkor
seen in which of the following
conditions? In kwashiorkor, edema results from the lack
of adequate blood proteins, primarily albumin,
A. Kwashiorkor to maintain the distribution of water between
B. Cachexia blood and tissues.
C. Marasmus
D. Rickets Reference:
Ferrier, D. (2017). Lippincott Illustrated Reviews:
Biochemistry (7th ed., p. 369). Lippincott Williams and
Wilkins.

13. Both very-long-chain fatty C C. Zellweger syndrome


acids (VLCFA) and long-chain
phytanic acid accumulate in Zellweger syndrome is caused by an inability
which of the following to target matrix proteins to the peroxisome,
disorders? therefore, all activities of the peroxisome are
affected because there are no functional
A. X-linked peroxisomes.
adrenoleukodystrophy
B. Refsum disease ● In X-linked adrenoleukodystrophy,
C. Zellweger syndrome VLCFA cannot be transported into the
D. None of the above peroxisome, but α-oxidation and other
peroxisomal activities are normal.
● Refsum disease results from the
deficiency of phytanoyl CoA hydroxylase,
leading to an accumulation of phytanic
acid in plasma and tissues.

References:
● Ferrier, D. (2017). Lippincott Illustrated Reviews:
Biochemistry (7th ed., p. 200). Lippincott Williams
and Wilkins.
● Le, T., Bhushan, V., & Sochat, M. (2023). First Aid
for the USMLE Step 1 2022, Thirty Third Edition
(33rd ed., p. 46). McGraw-Hill Education.

14. Which of the following A A. Selenocysteine


amino acids is synthesized from
serine and selenium and is Selenocysteine is synthesized from serine
found in enzymes such as and selenium and is found in enzymes such
glutathione peroxidase? as glutathione peroxidase and thioredoxin
reductase.
A. Selenocysteine
B. Threonine It is known as the 21st genetically encoded
C. Glycine amino acid.
D. Leucine

Reference:
Ferrier, D. (2017). Lippincott Illustrated Reviews:
Biochemistry (7th ed., p. 268). Lippincott Williams and
Wilkins.

15. Which of the following C C. Hyperpigmentation


DOES NOT describe
phenylketonuria (PKU)? PKU is caused by a deficiency of
phenylalanine hydroxylase (PAH).

8
A. Increased phenylalanine
B. Deficient tyrosine PKU is characterized by increased levels of
C. Hyperpigmentation phenylalanine. Tyrosine, which is normally
D. Intellectual disability formed from phenylalanine by PAH, is
deficient.

The hydroxylation of tyrosine by tyrosinase is


necessary for the formation of melanin →
hypopigmentation, not hyperpigmentation.

References:
● Ferrier, D. (2017). Lippincott Illustrated Reviews:
Biochemistry (7th ed., pp. 270-271). Lippincott
Williams and Wilkins.
● Le, T., Bhushan, V., & Sochat, M. (2023). First Aid
for the USMLE Step 1 2022, Thirty Third Edition
(33rd ed., p. 82). McGraw-Hill Education.

16. Which of the following B B. Glycine


amino acids is considered
achiral? Amino acids with a chiral α-carbon exist in two
different isomeric forms, which are
A. Alanine enantiomers.
B. Glycine
C. Leucine Glycine is an exception because its α-carbon
D. Valine has two hydrogen substituents.

Reference:
Ferrier, D. (2017). Lippincott Illustrated Reviews:
Biochemistry (7th ed., p. 5). Lippincott Williams and
Wilkins.

17. A decrease in the blood pH B B. Shift to the right


would cause the oxygen-
dissociation curve to: In the Bohr effect, the release of oxygen from
hemoglobin is enhanced when the pH is
A. Shift to the left lowered or when there is increased pCO2.
B. Shift to the right
C. Be unchanged These two conditions result in decreased
D. None of the above
oxygen affinity of hemoglobin → shift to the
right in the oxygen-dissociation curve.

Reference:
Ferrier, D. (2017). Lippincott Illustrated Reviews:
Biochemistry (7th ed., p. 30). Lippincott Williams and
Wilkins.

18. What enzyme is deficient in A A. Uroporphyrinogen decarboxylase


porphyria cutanea tarda?
Porphyria cutanea tarda is the most common
A. Uroporphyrinogen porphyria.
decarboxylase
B. Coproporphyrinogen III It can be caused by an autosomal dominant
oxidase deficiency in uroporphyrinogen decarboxylase
C. Protoporphyrinogen

9
oxidase → uroporphyrin accumulation in the urine.
D. Uroporphyrinogen III
synthase Reference:
Ferrier, D. (2017). Lippincott Illustrated Reviews:
Biochemistry (7th ed., p. 281). Lippincott Williams and
Wilkins.

19. In the heme biosynthesis C C. Lead


pathway, the enzymes ALA
dehydratase and ferrochelatase Ferrochelatase and ALA dehydratase are
are sensitive to inhibition by inhibited by lead.
which of the following?
In lead poisoning, protoporphyrin and ALA
A. Iron accumulate in the urine.
B. Oxygen
C. Lead Reference:
D. Carbon dioxide Ferrier, D. (2017). Lippincott Illustrated Reviews:
Biochemistry (7th ed., p. 281). Lippincott Williams and
Wilkins.

20. Which of the following is B B. Unconjugated bilirubin


elevated in Crigler-Najjar
syndrome? In Crigler-Najjar syndrome, there is absent
bilirubin UDP-glucuronosyltransferase, which
A. Conjugated bilirubin
B. Unconjugated bilirubin prevents the conjugation of bilirubin →
C. Direct and indirect increase in unconjugated bilirubin.
bilirubin
D. Biliverdin References:
● Ferrier, D. (2017). Lippincott Illustrated Reviews:
Biochemistry (7th ed., p. 283). Lippincott Williams
and Wilkins.
● Le, T., Bhushan, V., & Sochat, M. (2023). First Aid
for the USMLE Step 1 2022, Thirty Third Edition
(33rd ed., p. 401). McGraw-Hill Education.

21. Which of the following is D D. Increased indirect bilirubin


NOT associated with biliary
atresia? In biliary atresia, there is fibro-obliterative
destruction of bile ducts which leads to
A. Acholic stools cholestasis.
B. Hepatomegaly
C. Increased direct bilirubin Common manifestations include:
D. Increased indirect ● Acholic stools
bilirubin ● Hepatomegaly
● Persistence of jaundice after 2 weeks of
life
● Darkening of urine.

Lab workup will reveal increased direct


(conjugated) bilirubin and gamma-
glutamyltransferase.

Reference:
Le, T., Bhushan, V., & Sochat, M. (2023). First Aid for
the USMLE Step 1 2022, Thirty Third Edition (33rd ed.,

10
p. 401). McGraw-Hill Education.

22. Which of the following is a D D. B5


component of Coenzyme A
Pantothenic acid (vitamin B5) is a
A. B1 component of CoA, which functions in the
B. B2 transfer of acyl groups.
C. B9
D. B5 Reference:
Ferrier, D. (2017). Lippincott Illustrated Reviews:
Biochemistry (7th ed., p. 385). Lippincott Williams and
Wilkins.

23. Vitamin B6 deficiency can A A. Isoniazid


be induced by which of the
following drugs? Isoniazid, a drug commonly used to treat
tuberculosis, can induce a vitamin B6
A. Isoniazid deficiency by forming an inactive derivative
B. Rifampicin with pyridoxal phosphate.
C. Pyrazinamide
D. Ethambutol Reference:
Ferrier, D. (2017). Lippincott Illustrated Reviews:
Biochemistry (7th ed., p. 382). Lippincott Williams and
Wilkins.

24. The clinical manifestations B B. Decreased collagen hydroxylation


in the connective tissue of
people with scurvy may be Ascorbic acid deficiency results in scurvy, a
attributed to which of the disease characterized by:
following mechanisms? ● Sore and spongy gums
● Loose teeth
A. Decreased iron ● Fragile blood vessels
absorption ● Hemorrhage
B. Decreased collagen ● Swollen joints
hydroxylation ● Bone changes
C. Increased iron ● Fatigue.
absorption
D. Increased collagen Many of the deficiency symptoms can be
hydroxylation explained by the decreased hydroxylation of
collagen, resulting in defective connective
tissue.

Microcytic anemia caused by decreased


absorption of iron may also be seen.

Reference:
Ferrier, D. (2017). Lippincott Illustrated Reviews:
Biochemistry (7th ed., p. 381). Lippincott Williams and
Wilkins.

25. Which of the following C C. Ribozymes


enzymes is NOT a protein?
Ribozymes are RNAs with catalytic activity.
A. Kinase
B. Hydrolase Reference:
C. Ribozymes Ferrier, D. (2017). Lippincott Illustrated Reviews:
Biochemistry (7th ed., p. 54). Lippincott Williams and
D. Lyase

11
Wilkins.

26. Low-dose aspirin therapy A A. COX-1


lowers the risk of stroke and
heart attacks by inhibiting Aspirin’s anti-thrombogenic effect occurs
thromboxane synthesis by by the inhibition of TXA2 synthesis by COX-1
which enzyme? in platelets and PGI2 synthesis by COX-2 in
endothelial cells via irreversible acetylation of
A. COX-1 these isozymes.
B. COX-2
C. PGH2 synthase Reference:
D. Phospholipase A2 Ferrier, D. (2017). Lippincott Illustrated Reviews:
Biochemistry (7th ed., p. 214). Lippincott Williams and
Wilkins.

27. Which of the following is the B B. Oxygen


terminal acceptor of electrons in
the electron transport chain? Electrons tend to flow down the electron
transport chain (ETC) to oxygen.
A. NADH
B. Oxygen This makes oxygen the terminal acceptor of
C. FAD electrons.
D. Cytochrome c
Reference:
Ferrier, D. (2017). Lippincott Illustrated Reviews:
Biochemistry (7th ed., p. 82). Lippincott Williams and
Wilkins.

28. This X-linked disorder is B B. Menkes syndrome


caused by an impairment of
ATP7A that leads to systemic Menkes syndrome (“kinky hair” disease) is a
copper deficiency: rare X-linked disorder.

A. Wilson disease In this disorder, there is an impairment of


B. Menkes syndrome efflux of dietary copper out of intestinal
C. Osteogenesis enterocytes into the circulation by a copper-
imperfecta transporting ATPase (ATP7A) → systemic
D. Marfan syndrome
copper deficiency.

● Wilson disease is an autosomal recessive


disorder in which the efflux of excess
copper from the liver by ATP7B is
impaired.
● Osteogenesis imperfecta results from a
decreased production of normal type I
collagen
● Marfan syndrome results in a defective
fibrillin-1

References:
● Ferrier, D. (2017). Lippincott Illustrated Reviews:
Biochemistry (7th ed., p. 402). Lippincott Williams
and Wilkins.
● Le, T., Bhushan, V., & Sochat, M. (2023). First Aid
for the USMLE Step 1 2022, Thirty Third Edition
(33rd ed., p. 50). McGraw-Hill Education.

12
29. Xanthine is formed from the B B. Guanine
deamination of which of the
following nucleotides? Xanthine is formed from the deamination of
guanine.
A. Adenine
B. Guanine ● Deamination of cytosine → uracil
C. Cytosine
D. Thymine ● Deamination of adenine → hypoxanthine
● Deamination of 5-methylcytosine →
thymine

Reference:
Ferrier, D. (2017). Lippincott Illustrated Reviews:
Biochemistry (7th ed., p. 33). Lippincott Williams and
Wilkins.

30. The following are water- D D. Phylloquinone


soluble vitamins, EXCEPT?
Vitamin K (phylloquinones, menaquinones)
A. Riboflavin are fat-soluble vitamins. Other fat-soluble
B. Folic acid vitamins include:
C. Pyridoxine ● Vitamin A (retinol, beta-carotenes)
D. Phylloquinone ● Vitamin D (cholecalciferol)
● Vitamin E (tocopherols)
Reference:
Ferrier, D. (2017). Lippincott Illustrated Reviews:
Biochemistry (7th ed., p. 377). Lippincott Williams and
Wilkins.

31. This vitamin acts as a D D. Phytomenadione


cofactor in the carboxylation of
glutamic acid residues to Formation of the functional clotting factors
gamma-carboxyglutamate requires the vitamin K-dependent
residues: carboxylation of glutamic acid residues to
gamma-carboxyglutamate residues.
A. Riboflavin
B. Folic acid Reference:
C. Pyridoxine Ferrier, D. (2017). Lippincott Illustrated Reviews:
Biochemistry (7th ed., p. 393). Lippincott Williams and
D. Phytomenadione Wilkins.

32. One of the following bases D D. Uracil


is NOT used by DNA
The bases of one strand of DNA are paired
A. Adenine with the bases of the second strand, so that:
B. Guanine ● an adenine (A) is always paired with a
C. Thymine thymine (T), and
D. Uracil ● a cytosine (C) is always paired with a
guanine (G).

Reference:
Ferrier, D. (2017). Lippincott Illustrated Reviews:
Biochemistry (7th ed., p. 413). Lippincott Williams and
Wilkins.

33. Cataract formation in B B. Galactitol

13
galactokinase deficiency results
from the accumulation of which Galactokinase deficiency is a rare
of the following? autosomal-recessive disorder that causes
galactitol accumulation if galactose is present
A. Tyrosine in the diet.
B. Galactitol
C. Glycogen Elevated galactitol → cataract formation.
D. Fatty acids
The treatment of galactokinase deficiency is
dietary restriction.

Reference:
Ferrier, D. (2017). Lippincott Illustrated Reviews:
Biochemistry (7th ed., p. 141). Lippincott Williams and
Wilkins.

34. Megaloblastic anemia C C. Folate


associated with elevated
homocysteine but with normal Folic acid and cobalamin are both required
methylmalonic acid levels is in the remethylation of homocysteine to
seen in deficiency of which of methionine.
the following? Deficiency of both of these vitamins →
A. Pyridoxine increased levels of homocysteine.
B. Biotin
C. Folate However, cobalamin also acts as a cofactor
D. Cobalamin for methylmalonyl-CoA mutase which is why a
deficiency of B12 also leads to the elevation of
methylmalonic acid.

References:
● Ferrier, D. (2017). Lippincott Illustrated Reviews:
Biochemistry (7th ed., pp. 378-379). Lippincott
Williams and Wilkins.
● Le, T., Bhushan, V., & Sochat, M. (2023). First Aid
for the USMLE Step 1 2022, Thirty Third Edition
(33rd ed., p. 167). McGraw-Hill Education.

35. Acrodermatitis C C. Zinc


enteropathica results from a
congenital defect in intestinal A congenital defect in intestinal zinc
absorption of which of the absorption results in acrodermatitis
following? enteropathica, which manifests with a triad
of:
A. Vitamin K ● Hair loss
B. Ascorbic acid ● Diarrhea, and
C. Zinc ● Inflammatory skin rash around body
D. Cobalamin openings (periorificial) and tips of
fingers (acral).

References:
● Ferrier, D. (2017). Lippincott Illustrated Reviews:
Biochemistry (7th ed., p. 408). Lippincott Williams
and Wilkins.
● Le, T., Bhushan, V., & Sochat, M. (2023). First Aid
for the USMLE Step 1 2022, Thirty Third Edition
(33rd ed., p. 69). McGraw-Hill Education.

14
36. What is the rate-limiting A A. Glycogen phosphorylase
enzyme of glycogenolysis?
Glycogen phosphorylase is the rate-limiting
A. Glycogen enzyme of glycogenolysis.
phosphorylase
B. Glycogen synthase Glycogen synthase: rate-limiting enzyme of
C. G6PD glycogenesis
D. Phosphofructokinase-1
G6PD: rate-limiting enzyme of HMP shunt

Phosphofructokinase-1: rate-limiting enzyme


of glycolysis

References:
● Ferrier, D. (2017). Lippincott Illustrated Reviews:
Biochemistry (7th ed., p. 121). Lippincott Williams
and Wilkins.
● Le, T., Bhushan, V., & Sochat, M. (2023). First Aid
for the USMLE Step 1 2022, Thirty Third Edition
(33rd ed., p. 71). McGraw-Hill Education.

37. What enzyme is deficient in A A. Glycogen phosphorylase


McArdle disease?
Disease Enzyme deficient
A. Glycogen
phosphorylase McArdle disease Glycogen
B. Lysosomal α(1→4)- phosphorylase
glucosidase Pompe disease Lysosomal α(1→4)-
C. 4:4 transferase
D. Glucose-6-phosphatase glucosidase

Cori disease 4:4 transferase

Gierke disease Glucose-6-


phosphatase

Reference:
Ferrier, D. (2017). Lippincott Illustrated Reviews:
Biochemistry (7th ed., pp. 129-130). Lippincott Williams
and Wilkins.

38. Cataract formation in a C C. Aldose reductase


patient with poorly controlled
diabetes mellitus is caused by Insulin is not required for glucose entry into
an increase in the activity of this the cells of the retina, lens, kidneys, and
enzyme: peripheral nerves.

A. Fructokinase In hyperglycemia, large amounts of glucose


B. Aldolase B
C. Aldose reductase may enter these cells, and elevated
D. Sorbitol dehydrogenase intracellular glucose plus an adequate supply
of NADPH → production of elevated amounts

15
of sorbitol by aldose reductase → strong
osmotic effects and cell swelling due to water
influx and retention → cataract formation.

References:
● Ferrier, D. (2017). Lippincott Illustrated Reviews:
Biochemistry (7th ed., p. 140). Lippincott Williams
and Wilkins.
● Le, T., Bhushan, V., & Sochat, M. (2023). First Aid
for the USMLE Step 1 2022, Thirty Third Edition
(33rd ed., p. 79). McGraw-Hill Education.

39. Which Apo E isoform is A A. E-2


associated with familial
dysbetalipoproteinemia? Intermediate-density lipoproteins may be
taken up by the liver through receptor-
A. E-2 mediated endocytosis that uses apo E as
B. E-3 ligand. The three isoforms of apo E are:
C. E-4 ● E-2: least common, implicated in
D. IDL familial dysbetalipoproteinemia
● E-3: most common
● E-4: associated with increased
susceptibility to early-onset Alzheimer
disease

Reference:
Ferrier, D. (2017). Lippincott Illustrated Reviews:
Biochemistry (7th ed., p. 231). Lippincott Williams and
Wilkins.

40. Carbamoyl phosphate A A. A N-acetyl-glutamate


synthetase I (CPS I), which is
involved in the urea cycle, is In the urea cycle (occurs in mitochondria):
activated by which of the ● CPS I acts to synthesize carbamoyl
following? phosphate from ammonia and
bicarbonate.
A. N-acetyl-glutamate ● CPS I is activated by N-acetyl-
B. Phosphoribosyl glutamate.
pyrophosphate
C. Uridine triphosphate In pyrimidine synthesis (occurs in cytosol):
D. None of the above ● CPS II acts to synthesize carbamoyl
phosphate from glutamine and CO2.
● CPS II is inhibited by uridine
triphosphate (the end product of this
pathway) and is activated by PRPP.

Reference:
Ferrier, D. (2017). Lippincott Illustrated Reviews:
Biochemistry (7th ed., p. 254). Lippincott Williams and
Wilkins.

41. Which force is responsible B B. Hydrogen bonds


for maintaining the alpha-helix
structure? An alpha helix is a rigid, right-handed spiral
structure that is stabilized by extensive

16
A. Covalent bonds hydrogen bonding between the peptide bond
B. Hydrogen bonds carbonyl oxygens and amide hydrogens that
C. Disulfide bonds are part of the polypeptide backbone.
D. Hydrophobic
interactions Reference:
Ferrier, D. (2017). Lippincott Illustrated Reviews:
Biochemistry (7th ed., p. 16). Lippincott Williams and
Wilkins.

42. This DNA repair mechanism B B. Nucleotide excision repair


is defective in xeroderma
pigmentosum: Nucleotide excision repair is impaired in
xeroderma pigmentosum (XP).
A. Mismatch repair In this disorder, the cells cannot repair
B. Nucleotide excision damaged DNA, resulting in an extensive
repair accumulation of mutations and, consequently,
C. Base excision repair early and numerous skin cancers.
D. Double-strand break
repair
Mutations to proteins involved in mismatch
repair are associated with hereditary
nonpolyposis colorectal cancer (Lynch
syndrome.

Mutations to BRCA1 or BRCA2, which are


involved in homologous recombination in
double-strand break repair, increase the risk of
developing breast and ovarian cancer.

Reference:
Ferrier, D. (2017). Lippincott Illustrated Reviews:
Biochemistry (7th ed., pp. 427-428). Lippincott Williams
and Wilkins.

43. Which of the following is a B B. Telomerase


reverse transcriptase that helps
in avoiding the loss of genetic Telomerase is a reverse transcriptase that
material in chromosomes with helps in avoiding the loss of genetic material in
each duplication? chromosomes with each duplication. It does
this by adding DNA to the 3’ ends of the
A. TATA box chromosomes.
B. Telomerase
C. DNA polymerase Reference:
D. Primase Le, T., Bhushan, V., & Sochat, M. (2023). First Aid for
the USMLE Step 1 2022, Thirty Third Edition (33rd ed.,
p. 36). McGraw-Hill Education.

44. This blotting procedure C C. Western blot


separates the sample protein
via gel electrophoresis and Western blots are similar to Southern blot,
“blotted” or transfers it to a except that it is protein molecules in the
membrane: sample that are separated by electrophoresis
and blotted to a membrane.
A. Northern blot
B. Southern blot A labeled antibody is then used to bind the
C. Western blot relevant protein.

17
D. Southwestern blot
References:
● Ferrier, D. (2017). Lippincott Illustrated Reviews:
Biochemistry (7th ed., p. 500). Lippincott Williams
and Wilkins.
● Le, T., Bhushan, V., & Sochat, M. (2023). First Aid
for the USMLE Step 1 2022, Thirty Third Edition
(33rd ed., p. 51). McGraw-Hill Education.

45. Which HLA subtype is B B. HLA DR3


associated with type 1 diabetes
mellitus? HLA DR3 and DR4 are associated with type 1
diabetes mellitus.
A. HLA B27
B. HLA DR3 HLA B27: Psoriatic arthritis, ankylosing
C. HLA DQ2/DQ8 spondylitis, IBD-associated arthritis, reactive
D. HLA B57 arthritis

HLADQ2/DQ8: Celiac disease

HLA B57: Abacavir hypersensitivity

Reference:
Le, T., Bhushan, V., & Sochat, M. (2023). First Aid for
the USMLE Step 1 2022, Thirty Third Edition (33rd ed.,
p. 98). McGraw-Hill Education.

46. Which of the following is B B. Huntington disease


associated with CAG
trinucleotide repeats? Huntington disease: CAG repeats

A. Myotonic dystrophy Myotonic dystrophy: CTG repeats


B. Huntington disease
C. Fragile X syndrome Fragile X syndrome: CGG repeats
D. Friedreich syndrome
Friedreich ataxia: GAA repeats

Reference:
Le, T., Bhushan, V., & Sochat, M. (2023). First Aid for
the USMLE Step 1 2022, Thirty Third Edition (33rd ed.,
p. 60). McGraw-Hill Education.

47. Which HLA subtype is A A. HLA B27


associated with ankylosing
spondylitis? HLA B27 is associated with ankylosing
spondylitis
A. HLA B27
B. HLA DR3 HLADQ2/DQ8: Celiac disease
C. HLA DQ2/DQ8
D. HLA B57 HLA B57: Abacavir hypersensitivity

Reference:
Le, T., Bhushan, V., & Sochat, M. (2023). First Aid for
the USMLE Step 1 2022, Thirty Third Edition (33rd ed.,
p. 98). McGraw-Hill Education.

48. The enzyme hypoxanthine B B. Lesch-Nyhan syndrome


guanine phosphoribosyl

18
transferase (HGPRT) is Lesch-Nyhan syndrome results from
deficient in which of the defective purine salvage. There is absent
following?
HGPRT → excess uric acid production.
A. Severe combined
immunodeficiency Reference:
B. Lesch-Nyhan syndrome Le, T., Bhushan, V., & Sochat, M. (2023). First Aid for
the USMLE Step 1 2022, Thirty Third Edition (33rd ed.,
C. Chediak-Higashi p. 35). McGraw-Hill Education.
syndrome
D. Krabbe disease

49. The classical type of Ehlers- D D. V


Danlos syndrome is caused by
a mutation in which collagen Mutation in type V collagen: classical type of
type? Ehlers-Danlos syndrome

A. I Mutation in fibrillin-1 protein: Marfan


B. III syndrome
C. IV
D. V Decreased production of type I collagen:
Osteogenesis imperfecta

Mutations in type III procollagen: vascular


type of Ehlers-Danlos syndrome

Reference:
Le, T., Bhushan, V., & Sochat, M. (2023). First Aid for
the USMLE Step 1 2022, Thirty Third Edition (33rd ed.,
p. 49). McGraw-Hill Education.

50. A breastfed infant was B B. Classic galactosemia


brought to the clinic for failure
to thrive and jaundice. On This is most likely a case of classic
examination, there was galactosemia from the manifestations of:
hepatomegaly. ● Failure to thrive
● Jaundice
He was referred to ● Hepatomegaly
ophthalmology for consideration ● Infantile cataracts
of infantile cataracts. ● Intellectual disability

Which of the following is the Classic galactosemia is a deficiency of


most likely diagnosis? galactose-1-phosphate uridyltransferase.

A. Essential fructosuria Galactokinase deficiency is a milder


B. Classic galactosemia condition which is a deficiency of
C. G6PD deficiency galactokinase.
D. None of the above
Both conditions are autosomal recessive.

Essential fructosuria is a benign condition in


which there is a deficiency of fructokinase.

G6PD deficiency presents with back pain and


hemoglobinuria a few days after intake of
substances that increase oxidative stress such

19
as sulfa drugs, antimalarials, and fava beans.

References:
● Ferrier, D. (2017). Lippincott Illustrated Reviews:
Biochemistry (7th ed., pp. 149-151). Lippincott
Williams and Wilkins.
● Le, T., Bhushan, V., & Sochat, M. (2023). First Aid
for the USMLE Step 1 2022, Thirty Third Edition
(33rd ed., p. 78, p. 428). McGraw-Hill Education.

51. Which of the following is A A. Aminoacyl-tRNA synthetase


responsible for recognizing a
specific amino acid and all the Aminoacyl-tRNA synthetase is responsible
tRNA that correspond to that for recognizing a specific amino acid and all
amino acid? the tRNA that matches that amino acid.

A. Aminoacyl-tRNA Ribonucleases are used to synthesize rRNA


synthetase from pre-rRNA.
B. Ribonuclease
C. Helicase Helicase acts to unwind the DNA template at
D. None of the above the replication fork.

References:
● Ferrier, D. (2017). Lippincott Illustrated Reviews:
Biochemistry (7th ed., p. 461). Lippincott Williams
and Wilkins.
● Le, T., Bhushan, V., & Sochat, M. (2023). First Aid
for the USMLE Step 1 2022, Thirty Third Edition
(33rd ed., p. 36). McGraw-Hill Education.

52. The H+ pumped out across D D. Complex V


the inner mitochondrial
membrane (to create a proton The H+ pumped out across the inner
gradient) reenters the
mitochondrial matrix through mitochondrial membrane (to create a proton
which of the following? gradient) reenters the mitochondrial matrix
A. Complex I through Complex V → ATP synthesis
B. Complex III
C. Complex IV Reference:
D. Complex V Ferrier, D. (2017). Lippincott Illustrated Reviews:
Biochemistry (7th ed., pp. 77-78). Lippincott Williams and
Wilkins.

53. Overdose of which of the B B. Aspirin


following drugs can cause
uncoupling of oxidative Overdose of aspirin → uncoupling of
phosphorylation?
oxidative phosphorylation → fever
A. Morphine
B. Aspirin Other uncouplers:
C. Paracetamol ● UCP1 from brown fat
D. Tricyclic antidepressants ● 2,4 dinitrophenol

References:
● Ferrier, D. (2017). Lippincott Illustrated Reviews:
Biochemistry (7th ed., p. 79). Lippincott Williams and
Wilkins.

20
● Le, T., Bhushan, V., & Sochat, M. (2023). First Aid
for the USMLE Step 1 2022, Thirty Third Edition
(33rd ed., p. 76). McGraw-Hill Education.

54. All of the following amino D D. Valine


acids are used in collagen
synthesis, EXCEPT? All of the following amino acids are used in
collagen synthesis:
A. Proline ● Proline
B. Lysine ● Glycine
C. Glycine ● Lysine
D. Valine
Reference:
Ferrier, D. (2017). Lippincott Illustrated Reviews:
Biochemistry (7th ed., p. 45). Lippincott Williams and
Wilkins.

55. Which of the following A A. Molybdenum


substances is necessary to
convert hypoxanthine to The enzyme that converts hypoxanthine to
xanthine? xanthine is xanthine oxidase and it requires
molybdenum.
A. Molybdenum
B. Tyrosine Reference:
C. Phenylalanine Lieberman, M., & Ricer, R. E. (2013). BRS biochemistry,
molecular biology, and genetics (6th ed., p. 307).
D. Calcium Lippincott Williams and Wilkins.

56. What enzyme converts A A


xanthine to uric acid for its
excretion through the kidneys? Xanthine oxidase converts xanthine to uric
acid for its excretion through the kidneys.
A. Xanthine oxidase
B. Urate oxidase Urate oxidase (uricase) is for conversion of
C. Glucokinase uric acid to allantoin, a more water-soluble
D. Uricase product. Pegloticase, a recombinant uricase,
has been used clinically to decrease uric acid
levels in the blood.

References:
● Ferrier, D. (2017). Lippincott Illustrated Reviews:
Biochemistry (7th ed., p. 298). Lippincott Williams
and Wilkins.
● Le, T., Bhushan, V., & Sochat, M. (2023). First Aid
for the USMLE Step 1 2022, Thirty Third Edition
(33rd ed., p. 496). McGraw-Hill Education.

57. Celiac disease is D D. Peanuts


intolerance to gliadin which is
found in all of the following food Celiac disease is intolerance to gliadin which
products, EXCEPT? is found in all of the following:
● Barley
A. Barley ● Rye
B. Rye ● Wheat
C. Wheat
D. Peanuts There is no gliadin in peanuts.
Reference:

21
Ferrier, D. (2017). Lippincott Illustrated Reviews:
Biochemistry (7th ed., p. 249). Lippincott Williams and
Wilkins.

58. What is the rate-limiting B B. Phosphofructokinase-1


step in glycolysis?
Phosphofructokinase-1 is the rate-limiting
A. Glucokinase step of glycolysis.
B. Phosphofructokinase-1
C. Pyruvate kinase Glucokinase converts glucose to glucose-6-
D. Phosphoglycerate phosphate.
kinase
Pyruvate kinase converts
phosphoenolpyruvate to pyruvate.

Phosphoglycerate kinase converts 1,3-BPG


to 3-phosphoglycerate and in the process
synthesizes ATP.

Reference:
Ferrier, D. (2017). Lippincott Illustrated Reviews:
Biochemistry (7th ed., p. 102). Lippincott Williams and
Wilkins.

59. Melanin is a derivative of A A. Tyrosine


____:
DOPA (dihydroxyphenylalanine) is converted
A. Tyrosine to melanin by the action of tyrosinase.
B. Dopamine
C. Norepinephrine The other options are derivatives of tyrosine
D. Homogentisic acid catabolism or catecholamine synthesis.

Reference:
Le, T., Bhushan, V., & Sochat, M. (2023). First Aid for
the USMLE Step 1 2022, Thirty Third Edition (33rd ed.,
p. 81). McGraw-Hill Education.

60. A 5-month-old patient was B B. Tay-Sachs disease


brought to the clinic for
developmental delay, Tay-Sachs disease (a disease with an
hyperactive reflexes, with no accumulation of gangliosides) is the most
hepatosplenomegaly. On likely diagnosis in this 5-month-old patient.
examination, there’s a cherry-
red spot on the macula. The presence of developmental delay,
hyperactive reflexes, with NO
What is the most likely hepatosplenomegaly, and a cherry-red spot on
diagnosis in this case? the macula support the most likely diagnosis.

A. Fabry disease Other diseases that can present with a cherry-


B. Tay-Sachs disease red spot on the macula are:
C. Krabbe disease ● Niemann-Pick disease is an
D. Gaucher disease accumulation of sphingomyelin (which has
findings of foam cells and
hepatosplenomegaly).
● Central retinal artery occlusion

22
Reference:
Le, T., Bhushan, V., & Sochat, M. (2023). First Aid for
the USMLE Step 1 2022, Thirty Third Edition (33rd ed.,
p. 86, p. 723). McGraw-Hill Education.

61. What enzyme is deficient in A A. Hexosaminidase A


the patient case above?
Hexosaminidase A deficiency is seen in
A. Hexosaminidase A Tay-Sachs disease
B. Alpha-galactosidase A
C. Sphingomyelinase Disease Enzyme deficient
D. Galactocerebrosidase
Niemann-Pick Sphingomyelinase
disease

Fabry disease Alpha-galactosidase A

Krabbe disease Galactocerebrosidase

Reference:
Le, T., Bhushan, V., & Sochat, M. (2023). First Aid for
the USMLE Step 1 2022, Thirty Third Edition (33rd ed.,
p. 86). McGraw-Hill Education.

62. Which of the following is C C. Glycerol


produced in the hydrolysis of
triacylglycerol (TAG) in adipose Glycerol is produced in the hydrolysis of
tissues which is then converted triacylglycerol (TAG) in adipose tissues which
to an intermediate product of is then converted to dihydroxyacetone
glycolysis and phosphate, an intermediate product of
gluconeogenesis? glycolysis and gluconeogenesis.

A. Pyruvate Reference:
B. Glucose-6-phosphate Ferrier, D. (2017). Lippincott Illustrated Reviews:
Biochemistry (7th ed., p. 117). Lippincott Williams and
C. Glycerol Wilkins.
D. 1,3 bisphosphoglycerate

63. A patient with a A A. Fumarylacetoacetate


characteristic cabbage-like odor
in the urine most likely has an A patient with a characteristic cabbage-like
accumulation of which of the odor in the urine most likely has accumulation
following substances in the of fumarylacetoacetate in the urine resulting
body? from deficiencies in fumarylacetoacetate
hydrolase. This disorder is called
A. Fumarylacetoacetate Tyrosinemia type I.
B. Branched-chain alpha-
amino acids Branched-chain alpha-amino acids
C. Homocysteine accumulate in patients with deficient
D. Homogentisic acid branched-chain alpha-ketoacid
dehydrogenase leading to Maple syrup urine
disease.

Homogentisic acid accumulates in patients

23
with a deficiency in homogentisic acid oxidase.
Urine appears entirely black from oxidation of
homogentisic acid from air exposure. This is
called Alkaptonuria.

Homocysteine accumulates in Homocystinuria


due to cystathionine synthase deficiency.

References:
● Ferrier, D. (2017). Lippincott Illustrated Reviews:
Biochemistry (7th ed., p. 269). Lippincott Williams
and Wilkins.
● Le, T., Bhushan, V., & Sochat, M. (2023). First Aid
for the USMLE Step 1 2022, Thirty Third Edition
(33rd ed., p. 82). McGraw-Hill Education.

64. Which of the following A A. Dubin-Johnson syndrome


hereditary hyperbilirubinemias
is characterized by conjugated Dubin-Johnson is a hereditary
hyperbilirubinemia with a hyperbilirubinemia characterized by
grossly black liver? conjugated hyperbilirubinemia with a grossly
black liver which is a result of impaired
A. Dubin-Johnson excretion of epinephrine metabolites.
syndrome
B. Rotor syndrome Rotor syndrome has the same presentation
C. Criggler-Najjar as Dubin-Johnson but is milder, and it has no
syndrome type 1 black liver.
D. Gilbert syndrome
Criggler-Najjar and Gilbert syndrome have
increased UNconjugated bilirubin, not
conjugated.

Reference:
Le, T., Bhushan, V., & Sochat, M. (2023). First Aid for
the USMLE Step 1 2022, Thirty Third Edition (33rd ed.,
p. 401). McGraw-Hill Education.

65. The liver can synthesize C C. Thiophorase


ketone bodies but can NOT use
them as fuel because of the The liver can synthesize ketone bodies but
lack of what enzyme? can NOT use them as fuel because of the lack
of thiophorase which is an enzyme that
A. Glycogen transfers CoA from succinyl CoA to
phosphorylase acetoacetate to become 2 molecules of acetyl
B. Glucokinase CoA.
C. Thiophorase
D. 3-hydroxybutyrate Reference:
dehydrogenase Ferrier, D. (2017). Lippincott Illustrated Reviews:
Biochemistry (7th ed., p. 517). Lippincott Williams and
Wilkins.

66. Ketone bodies are C C. Liver


produced in which of the
following? Ketogenesis occurs in the liver mitochondria.

A. Brain Reference:
B. Heart Ferrier, D. (2017). Lippincott Illustrated Reviews:

24
C. Liver Biochemistry (7th ed., p. 195). Lippincott Williams and
D. All of the above Wilkins.

67. A urine test for ketones can B B. Acetoacetate


detect which of the following
ketone bodies? A urine test for ketones can detect the
presence of acetoacetate, but not the
A. Beta-hydroxybutyrate presence of beta-hydroxybutyrate.
B. Acetoacetate
C. Acetyl CoA Reference:
D. All of the above Le, T., Bhushan, V., & Sochat, M. (2023). First Aid for
the USMLE Step 1 2022, Thirty Third Edition (33rd ed.,
p. 88). McGraw-Hill Education.

68. 1 gram of protein is equal to A A. 4


how many kcal?
1g carb = 4 kcal
A. 4 1g protein = 4 kcal
B. 7 1g alcohol = 7 kcal
C. 9 1 g fat = 9 kcal
D. 15
Reference:
Ferrier, D. (2017). Lippincott Illustrated Reviews:
Biochemistry (7th ed., p. 359). Lippincott Williams and
Wilkins.

69. Which of the following A A. Increased protein phosphatase activity


occurs in the well-fed state?
B, C, and D occur in the fasting state.
A. Increased protein
phosphatase activity Well-fed state:
B. Phosphorylation of ● Increased protein phosphatase activity
glycogen phosphorylase ● Dephosphorylation of glycogen
C. Increase protein kinase synthase (active form)
activity ● Dephosphorylation of glycogen
D. Phosphorylation of phosphorylase (inactive form)
glycogen synthase ● Decreased conversion of glycogen to
glucose

Fasting state:
● Increased protein kinase activity
● Phosphorylation of glycogen synthase
(inactive form)
● Phosphorylation of glycogen
phosphorylase (active form)
● Increased conversion of glycogen to
glucose

Reference:
Ferrier, D. (2017). Lippincott Illustrated Reviews:
Biochemistry (7th ed., p. 135). Lippincott Williams and
Wilkins.

70. Which of the following B B. Phosphorylation of glycogen phosphorylase


occurs in the fasting state?
A, C, and D occur in the well-fed state.

25
A. Increased protein
phosphatase activity Well-fed state:
B. Phosphorylation of ● Increased protein phosphatase activity
glycogen phosphorylase ● Dephosphorylation of glycogen
C. Decreased protein synthase (active form)
kinase activity ● Dephosphorylation of glycogen
D. Dephosphorylation of phosphorylase (inactive form)
glycogen synthase ● Decreased conversion of glycogen to
glucose

Fasting state:
● Increased protein kinase activity
● Phosphorylation of glycogen synthase
(inactive form)
● Phosphorylation of glycogen
phosphorylase (active form)
● Increased conversion of glycogen to
glucose

Reference:
Ferrier, D. (2017). Lippincott Illustrated Reviews:
Biochemistry (7th ed., p. 135). Lippincott Williams and
Wilkins.

71. Which of the following does A A. Insulin


NOT serve as a signal for the
degradation of glycogen to Epinephrine and glucagon serve as signals for
increase blood glucose?
the degradation of glycogen to increase blood
A. Insulin glucose → increased cAMP → activation of
B. Glucagon
C. Epinephrine cAMP-dependent protein kinase A (PKA)
D. None of the above which leads to:
● Increased protein kinase activity
● Phosphorylation of glycogen synthase
(inactive form)
● Phosphorylation of glycogen
phosphorylase (active form)
● Increased conversion of glycogen to
glucose

Reference:
Ferrier, D. (2017). Lippincott Illustrated Reviews:
Biochemistry (7th ed., p. 132). Lippincott Williams and
Wilkins.

72. Nitric oxide is made from A A. Arginine


what amino acid?
Nitric oxide is made from the amino acid
A. Arginine Arginine.
B. Lysine
C. Tyrosine Reference:
D. Aspartate Ferrier, D. (2017). Lippincott Illustrated Reviews:
Biochemistry (7th ed., p. 154). Lippincott Williams and

26
Wilkins.

73. The initiation codon for C C. Methionine


translation is AUG which codes
for what amino acid? The initiation codon for translation is AUG
which codes for methionine.
A. Lysine
B. Arginine Reference:
C. Methionine Ferrier, D. (2017). Lippincott Illustrated Reviews:
Biochemistry (7th ed., p. 448). Lippincott Williams and
D. Tryptophan Wilkins.

74. Which of the following A A. X-linked adrenoleukodystrophy


disorders is a genetic defect
that results from inability to Zellweger syndrome is described as a
transport very long-chain fatty peroxisomal biogenesis disorder that results
acids (VLCFA) across the from the inability to target matrix proteins to
peroxisomal membrane? the peroxisomes. In this disorder, all
peroxisomal activities are affected.
A. X-linked
adrenoleukodystrophy In X-linked adrenoleukodystrophy, the only
B. Zellweger syndrome affected function is the transport of very long-
C. Metachromatic chain fatty acids (VLCFA) into the
leukodystrophy peroxisomes but the other functions are
D. All of the above normal.

Reference:
Ferrier, D. (2017). Lippincott Illustrated Reviews:
Biochemistry (7th ed., p. 195, p. 210). Lippincott Williams
and Wilkins.

75. The causative agent of B B. Beta-pleated form


Creutzfeldt-Jakob disease is
caused by the conversion of a The causative agent of Creutzfeldt-Jakob
normal protein to a/an ______: disease is caused by the conversion of a
normal prion protein (alpha-helical form) to a
A. Alpha-helix form beta-pleated form which is transmissible.
B. Beta-pleated form
C. Beta-turns form References:
D. All of the above ● Ferrier, D. (2017). Lippincott Illustrated Reviews:
Biochemistry (7th ed., p. 22). Lippincott Williams and
Wilkins.
● Le, T., Bhushan, V., & Sochat, M. (2023). First Aid
for the USMLE Step 1 2022, Thirty Third Edition
(33rd ed., p. 175). McGraw-Hill Education.

76. The neurodegenerative A A. Beta-amyloid


signs and symptoms of
Alzheimer disease are The neurodegenerative signs and symptoms
attributed to the accumulation of Alzheimer disease (AD) are attributed to
of senile plaques which are the accumulation of senile plaques which are
made up of ___: made up of EXTRAcellular beta-amyloid.

A. Beta-amyloid The hyperphosphorylated tau protein is


B. Hyperphosphorylated involved in the formation of neurofibrillary
tau protein tangles which are INTRAcellular. These are

27
C. Alpha-synuclein protein also responsible for the manifestations of AD.
D. All of the above
Alpha-synuclein is present in patients with
Parkinson disease.

References:
● Ferrier, D. (2017). Lippincott Illustrated Reviews:
Biochemistry (7th ed., p. 21). Lippincott Williams and
Wilkins.
● Le, T., Bhushan, V., & Sochat, M. (2023). First Aid
for the USMLE Step 1 2022, Thirty Third Edition
(33rd ed., p. 536). McGraw-Hill Education.

77. The following are TRUE in D D. None of the above


Alzheimer disease, EXCEPT?
All are true for AD.
A. Associated with Down
syndrome It is associated with Down syndrome
B. Findings include because the amyloid precursor protein (from
hippocampal atrophy which amyloid beta is derived) is found on
C. The most common chromosome 21.
cause of dementia in
older adults It is the most common cause of dementia in
D. None of the above older adults and patients exhibit hippocampal
atrophy.

Reference:
Le, T., Bhushan, V., & Sochat, M. (2023). First Aid for
the USMLE Step 1 2022, Thirty Third Edition (33rd ed.,
p. 536). McGraw-Hill Education.

78. Maple syrup urine disease D D. Lysine


(MSUD) is the accumulation of
branched amino acids such as The branched amino acids are:
the following, EXCEPT? ● Leucine
● Isoleucine
A. Leucine ● Valine
B. Valine
C. Isoleucine MSUD is due to the deficiency of branched-
D. Lysine
chain alpha-ketoacid dehydrogenase (BCKD).
This leads to the accumulation of branched
amino acids → maple-syrup-like odor of the
urine because of the increase in isoleucine.

References:
● Ferrier, D. (2017). Lippincott Illustrated Reviews:
Biochemistry (7th ed., p. 271). Lippincott Williams
and Wilkins.
● Le, T., Bhushan, V., & Sochat, M. (2023). First Aid
for the USMLE Step 1 2022, Thirty Third Edition
(33rd ed., p. 82). McGraw-Hill Education.

79. What vitamin should be A A. Vitamin B1


supplemented when treating
patients with MSUD? Vitamin B1 or thiamine should be

28
supplemented when treating patients with
A. Vitamin B1 MSUD because it serves as a cofactor for
B. Vitamin B9 several dehydrogenase enzyme reactions
C. Vitamin B12 such as branched chain alpha-ketoacid
D. Vitamin A dehydrogenase (BCKD) which is deficient in
MSUD.

Reference:
Le, T., Bhushan, V., & Sochat, M. (2023). First Aid for
the USMLE Step 1 2022, Thirty Third Edition (33rd ed.,
p. 64, p. 82). McGraw-Hill Education.

80. Sickle cell anemia results A A Glutamic acid to valine


from a single amino acid
substitution in the beta-globin Sickle cell anemia results from a single
gene from __ to ___: amino acid substitution in the beta-globin gene
from glutamic acid to valine.
A. Glutamic acid to valine
B. Valine to glutamic acid Glutamic acid to lysine mutation in the beta-
C. Glutamic acid to lysine globin is Hemoglobin C disease.
D. None of the above
References:
● Ferrier, D. (2017). Lippincott Illustrated Reviews:
Biochemistry (7th ed., p. 36). Lippincott Williams and
Wilkins.
● Le, T., Bhushan, V., & Sochat, M. (2023). First Aid
for the USMLE Step 1 2022, Thirty Third Edition
(33rd ed., p. 428). McGraw-Hill Education.

81. The alpha carbon of amino B B. Glycine


acids are chiral carbons,
EXCEPT for which one? The alpha carbon of amino acids is chiral
carbons, except for Glycine, which has 2
A. Glutamine hydrogen substituents in its alpha carbon.
B. Glycine
C. Arginine Chiral means the alpha carbon of an amino
D. Lysine acid is attached to 4 different chemical groups.

Reference:
Ferrier, D. (2017). Lippincott Illustrated Reviews:
Biochemistry (7th ed., p. 5). Lippincott Williams and
Wilkins.

82. Which of the following is A A. Fab fragment binds complement


NOT true about the structure of
an antibody? Fab fragment binds antigens, not complement.

A. Fab fragment binds Fc region is responsible for complement


complement fixation.
B. Fab fragment binds
antigens Light chain contributes to the Fab region
C. Light chain contributes only.
to the Fab region only Heavy chain contributes to both the Fc and
D. Heavy chain contributes Fab regions.
to the Fc and Fab
regions Reference:
Le, T., Bhushan, V., & Sochat, M. (2023). First Aid for

29
the USMLE Step 1 2022, Thirty Third Edition (33rd ed.,
p. 102). McGraw-Hill Education.

83. Which of the following D D. IgE


immunoglobulins helps in
mounting an immune response IgE helps in mounting an immune response
against parasites by activating against parasites by activating eosinophils.
eosinophils?
Reference:
A. IgA Le, T., Bhushan, V., & Sochat, M. (2023). First Aid for
the USMLE Step 1 2022, Thirty Third Edition (33rd ed.,
B. IgG p. 103). McGraw-Hill Education.
C. IgM
D. IgE

84. All of the following foods D D. None of the above


can precipitate a hypertensive
crisis, EXCEPT? Tyramine-rich foods can precipitate a
hypertensive crisis by displacing
A. Aged cheese norepinephrine into the synaptic cleft leading
B. Cured meats to increased stimulation of the sympathetic
C. Chocolates nervous system.
D. None of the above
Examples of tyramine-rich foods are:
● Aged cheese
● Cured meats
● Wine
● Chocolates
Reference:
Ferrier, D. (2017). Lippincott Illustrated Reviews:
Biochemistry (7th ed., p. 589). Lippincott Williams and
Wilkins.

85. A newborn was referred to B B. Crigler-Najjar syndrome


you for signs and symptoms of
jaundice and kernicterus. On Crigler-Najjar syndrome type 1 is
workup, the patient has characterized by signs and symptoms of
increased unconjugated jaundice and kernicterus which is due to the
bilirubin. deposition of unconjugated bilirubin in the
What is the most likely brain.
diagnosis? It is caused by the absence of UDP-
A. Gilbert syndrome glucuronosyltransferase → increased
B. Crigler-Najjar syndrome unconjugated bilirubin.
C. Dubin-Johnson
syndrome
D. Rotor syndrome Gilbert syndrome causes only mild jaundice
from mildly reduced UDP-
glucuronosyltransferase.

C and D are caused by conjugated


hyperbilirubinemia, not unconjugated.

References:

30
● Ferrier, D. (2017). Lippincott Illustrated Reviews:
Biochemistry (7th ed., p. 292). Lippincott Williams
and Wilkins.
● Le, T., Bhushan, V., & Sochat, M. (2023). First Aid
for the USMLE Step 1 2022, Thirty Third Edition
(33rd ed., p. 401). McGraw-Hill Education.

86. What is the rate-limiting C C. Carnitine acyltransferase I


step of fatty acid oxidation?
The rate-limiting step of fatty acid oxidation is
A. Phosphofructokinase-1 Carnitine acyltransferase I.
B. Acetyl-CoA carboxylase
C. Carnitine PFK-1 is the rate-limiting step of glycolysis.
acyltransferase I
D. HMG-CoA reductase Acetyl-CoA carboxylase is the rate-limiting
step of fatty acid synthesis.

HMG-CoA reductase is the rate-limiting step


of cholesterol synthesis.

References:
● Ferrier, D. (2017). Lippincott Illustrated Reviews:
Biochemistry (7th ed., p. 191). Lippincott Williams
and Wilkins.
● Le, T., Bhushan, V., & Sochat, M. (2023). First Aid
for the USMLE Step 1 2022, Thirty Third Edition
(33rd ed., p. 71). McGraw-Hill Education.

87. What form of hemoglobin A A. HbA


makes up the majority of total
hemoglobin in a normal adult? HbA makes up the majority (90%) of total
hemoglobin in a normal adult.
A. HbA
B. HbA2 HbA2 is only a minor (2-3%) component of
C. HbF normal adult hemoglobin.
D. All are of equal amounts
HbF is major hemoglobin found in the fetus
and newborn, not in a normal adult.

Reference:
Ferrier, D. (2017). Lippincott Illustrated Reviews:
Biochemistry (7th ed., p. 33). Lippincott Williams and
Wilkins.

88. A right shift in the oxygen D D. Presence of fetal hemoglobin


dissociation curve (ODC)
occurs in the following, Fetal hemoglobin causes a left shift, not
EXCEPT? right shift.

A. Fever Left shift in the ODC means a higher affinity


B. Increased 2,3 BPG for oxygen and reduced unloading of oxygen
C. Increased CO2 in the tissues.
D. Presence of fetal
hemoglobin Fetal Hb weakly binds 2,3 BPG → high affinity
for oxygen → left shift in the ODC.

31
Causes of right shift in the ODC:
● Increased 2,3 bisphosphoglycerate
(BPG)
● Decreased pH
● Exercise
● Fever
● Hypoxia

Causes of left shift in the ODC:


● Decreased temperature
● Presence of fetal Hb
● Increased carbon monoxide
● Increased methemoglobin

Reference:
Le, T., Bhushan, V., & Sochat, M. (2023). First Aid for
the USMLE Step 1 2022, Thirty Third Edition (33rd ed.,
p. 43). McGraw-Hill Education.

89. Translation is stopped when D D. AUG


release factors recognize stop
codons. Which of the following AUG is the start codon and codes for
is NOT a stop codon? methionine in eukaryotes and codes for N-
formylmethionine in prokaryotes.
A. UGA
B. UAA Reference:
C. UAG Le, T., Bhushan, V., & Sochat, M. (2023). First Aid for
the USMLE Step 1 2022, Thirty Third Edition (33rd ed.,
D. AUG p. 42). McGraw-Hill Education.

90. A point mutation occurred in A A. Silent


the coding region of an mRNA
which resulted in UCA (which Silent mutation: codes for the same amino
codes for serine) being acid
changed to UCU (which also
codes for serine). Missense mutation: codes for a different
amino acid
What type of mutation occurred
in this situation? Nonsense mutation: codes for a stop codon
leading to premature termination of translation
A. Silent
B. Missense References:
C. Nonsense ● Ferrier, D. (2017). Lippincott Illustrated Reviews:
Biochemistry (7th ed., p. 449). Lippincott Williams
D. None of the above and Wilkins.
● Le, T., Bhushan, V., & Sochat, M. (2023). First Aid
for the USMLE Step 1 2022, Thirty Third Edition
(33rd ed., p. 38). McGraw-Hill Education.

91. Sickle cell disease is an B B. Missense mutation


example of ____:
Sickle cell disease is an example of a
A. Silent mutation missense mutation (substitution of glutamic
B. Missense mutation acid with valine)
C. Nonsense mutation
D. None of the above References:

32
● Ferrier, D. (2017). Lippincott Illustrated Reviews:
Biochemistry (7th ed., p. 449). Lippincott Williams
and Wilkins.
● Le, T., Bhushan, V., & Sochat, M. (2023). First Aid
for the USMLE Step 1 2022, Thirty Third Edition
(33rd ed., p. 38). McGraw-Hill Education.

92. SOD1 (superoxide C C. Amyotrophic lateral sclerosis


dismutase 1) mutations are
associated with which of the
following neurological disease? SOD1 mutations are associated with the
familial form of Amyotrophic lateral
A. Spinal muscular atrophy sclerosis (ALS).
B. Tabes dorsalis
C. Amyotrophic lateral Tabes dorsalis is a demyelination of the
sclerosis dorsal column and roots in patients with
D. Subacute combined tertiary syphilis.
degeneration
Spinal muscular atrophy is linked to SMN1
(survival motor neuron protein) mutation →
apoptosis of lower motor neurons.

Subacute combined degeneration occurs in


patients with Vitamin B12 deficiency.

Reference:
Le, T., Bhushan, V., & Sochat, M. (2023). First Aid for
the USMLE Step 1 2022, Thirty Third Edition (33rd ed.,
p. 546). McGraw-Hill Education.

93. 1 kcal is how many C C. 4.2 kJ


kilojoules?
1 kcal = 4.2 kJ
A. 2.2 kJ
B. 3.2 kJ 1g carb = 4 kcal
C. 4.2 kJ 1g protein = 4 kcal
D. 5.2 kJ 1g alcohol = 7 kcal
1 g fat = 9 kcal

(I’m sorry for this question, but this was


apparently asked in the last PLE)

Reference:
Ferrier, D. (2017). Lippincott Illustrated Reviews:
Biochemistry (7th ed., p. 70, p. 359). Lippincott Williams
and Wilkins.

94. A newborn was referred to B B. Phenylketonuria


you for a characteristic musty
(“mousy”) odor in his urine.
A characteristic musty (“mousy”) odor in the
If he undergoes a newborn urine of a newborn is most likely a diagnosis of
screening test, he will most phenylketonuria (PKU).
likely be diagnosed with ____:
A patient with a characteristic cabbage-like
A. Alkaptonuria odor in the urine most likely has accumulation

33
B. Phenylketonuria of fumarylacetoacetate in the urine resulting
C. Maple syrup urine from deficiencies in fumarylacetoacetate
disease hydrolase. This disorder is called
D. Tyrosinemia Tyrosinemia type I.

Branched-chain alpha-amino acids


accumulate in patients with deficient
branched-chain alpha-ketoacid
dehydrogenase leading to Maple syrup urine
disease.

Homogentisic acid accumulates in patients


with a deficiency in homogentisic acid oxidase.
Urine appears entirely black from oxidation of
homogentisic acid from air exposure. This is
called Alkaptonuria.
References:
● Ferrier, D. (2017). Lippincott Illustrated Reviews:
Biochemistry (7th ed., p. 268, 271). Lippincott
Williams and Wilkins.
● Le, T., Bhushan, V., & Sochat, M. (2023). First Aid
for the USMLE Step 1 2022, Thirty Third Edition
(33rd ed., p. 82). McGraw-Hill Education.

95. Which of the following B B. Phenylpyruvate


substances account for the
characteristic odor in the patient Patients with PKU have a characteristic musty
case above? (“mousy”) odor in the urine due to the
accumulation of:
A. Homogentisic acid ● Phenylpyruvate
B. Phenylpyruvate ● Phenylacetate
C. Isoleucine ● Phenyllactate
D. Fumarylacetoacetate
Reference:
Ferrier, D. (2017). Lippincott Illustrated Reviews:
Biochemistry (7th ed., p. 271). Lippincott Williams and
Wilkins.

96. What is the smallest amino D D. Glycine


acid?
Glycine is the smallest amino acid.
A. Glutamic acid
B. Valine Reference:
C. Isoleucine Ferrier, D. (2017). Lippincott Illustrated Reviews:
Biochemistry (7th ed., p. 45). Lippincott Williams and
D. Glycine Wilkins.

97. Collagen synthesis requires C C. Vitamin C


what vitamin?
The hydroxylation reactions of specific proline
A. Vitamin A and lysine residues necessary in collagen
B. Vitamin B1 synthesis require Vitamin C as a reducing
C. Vitamin C agent.
D. Vitamin D
References:

34
● Ferrier, D. (2017). Lippincott Illustrated Reviews:
Biochemistry (7th ed., p.47). Lippincott Williams and
Wilkins.
● Le, T., Bhushan, V., & Sochat, M. (2023). First Aid
for the USMLE Step 1 2022, Thirty Third Edition
(33rd ed., p. 48). McGraw-Hill Education.

98. Mutation in which of the A A. Fibrillin-1


following proteins is responsible
for Marfan syndrome? Mutation in fibrillin-1 protein: Marfan
syndrome
A. Fibrillin-1
B. Type I collagen Decreased production of type I collagen:
C. Type V collagen Osteogenesis imperfecta
D. Type III procollagen
Mutation in type V collagen: classical type of
Ehlers-Danlos syndrome

Mutations in type III procollagen: vascular


type of Ehlers-Danlos syndrome

References:
● Ferrier, D. (2017). Lippincott Illustrated Reviews:
Biochemistry (7th ed., p.49). Lippincott Williams and
Wilkins.
● Le, T., Bhushan, V., & Sochat, M. (2023). First Aid
for the USMLE Step 1 2022, Thirty Third Edition
(33rd ed., p. 49). McGraw-Hill Education.

99. Which of the following is an C C. Refsum disease


autosomal recessive
peroxisomal disorder of alpha Refsum disease is an autosomal recessive
oxidation which leads to peroxisomal disorder of alpha-oxidation
accumulation of phytanic acid? characterized by the buildup of phytanic acid.

A. Zellweger syndrome Adrenoleukodystrophy is an X-linked


B. Adrenoleukodystrophy recessive disorder that results in the
C. Refsum disease accumulation of very long chain fatty acids
D. None of the above (VLCFA) in the adrenal glands, white matter of
the brain, and testes.

Zellweger syndrome is caused by an inability


to target matrix proteins to the peroxisome,
therefore, all activities of the peroxisome are
affected because there are no functional
peroxisomes.

Refsum disease is characterized by the


buildup of phytanic acid, not VLCFA.

Reference:
Le, T., Bhushan, V., & Sochat, M. (2023). First Aid for
the USMLE Step 1 2022, Thirty Third Edition (33rd ed.,
p. 46). McGraw-Hill Education.

100. Which of the following D D. Fc region


determines an antibody’s

35
isotype (i.e., IgG, IgA, IgM, IgD, The antibody isotype (i.e., IgG, IgA, IgM, IgD,
IgE)? IgE) is determined by the Fc region.

A. Fab fragment Reference:


B. Hinge Le, T., Bhushan, V., & Sochat, M. (2023). First Aid for
the USMLE Step 1 2022, Thirty Third Edition (33rd ed.,
C. Light chain p. 46). McGraw-Hill Education.
D. Fc region

36
ANATOMY

37
QUESTION ANSWER EXPLANATIONS

1. What is the largest branch of the B B. Splenic artery


celiac trunk?
The largest branch of the celiac trunk is the
A. Left gastric artery splenic artery.
B. Splenic artery
C. Common hepatic arteries Reference:
D. Superior mesenteric artery Halliday, N. L., & Chung, H. M. (2018). BRS Gross
Anatomy (9th ed., p. 133). Lippincott Williams and
Wilkins

2. The epiploic foramen is bounded A A. Caudate lobe of the liver


superiorly by the _______.
The foramen of Winslow (epiploic
A. Caudate lobe of the liver foramen) is bounded:
B. 1st part of the duodenum
C. Free edge of the lesser Superiorly Caudate lobe of the liver
omentum
D. IVC Inferiorly 1st part of duodenum

Anteriorly Free edge of the lesser


omentum

Posteriorly IVC

Reference:
Halliday, N. L., & Chung, H. M. (2018). BRS Gross
Anatomy (9th ed., p. 120). Lippincott Williams and
Wilkins

3. The portal vein is formed by the A A. Splenic vein and superior mesenteric vein
merging of ________.
The portal vein is formed by the merging of
A. Splenic vein and superior the splenic vein and the SMV.
mesenteric vein
B. Splenic vein and inferior The IMV will merge with either the:
mesenteric vein ● Splenic vein
C. Superior and inferior ● SMV
mesenteric vein ● Or junction of the splenic vein and the
D. None of the above SMV

Reference:
Halliday, N. L., & Chung, H. M. (2018). BRS Gross
Anatomy (9th ed., p. 137). Lippincott Williams and
Wilkins

4. The aorta goes through the aortic B B. T12


hiatus in the diaphragm at what level?
At T12 level, the aorta goes through the
A. T11 aortic hiatus found in the diaphragm.
B. T12
C. L1 Reference:
D. L2 Halliday, N. L., & Chung, H. M. (2018). BRS Gross
Anatomy (9th ed., p. 144). Lippincott Williams and
Wilkins.

38
5. Damage to the superior mesenteric C C. Right gastric artery
artery will affect the following vessels,
EXCEPT: The right gastric artery comes from the
common hepatic artery, which is a branch of
A. Middle colic artery the celiac trunk, not the superior mesenteric
B. Ileocolic artery artery.
C. Right gastric artery
D. Right colic artery The rest of the options are branches of the
superior mesenteric artery.

Reference:
Halliday, N. L., & Chung, H. M. (2018). BRS Gross
Anatomy (9th ed., p. 135). Lippincott Williams and
Wilkins.

6. The aortic valve is best heard at the A A. Right 2nd ICS


_______.
The aortic valve is best heard at the right 2nd
A. Right 2nd ICS ICS.
B. Left 2nd ICS
C. Right lower part of the sternum Valve Location best heard
D. Left 5th ICS at the MCL
Aortic valve Right 2nd ICS

Pulmonic valve Left 2nd ICS

Tricuspid valve Right lower part of the


sternum

Mitral valve Left 5th ICS at the


midclavicular line

Reference:
Halliday, N. L., & Chung, H. M. (2018). BRS Gross
Anatomy (9th ed., pp. 75-76). Lippincott Williams and
Wilkins.

7. The sinoatrial (SA) node is found B B. At the opening of the superior vena cava
____: (SVC) in the RA

A. In the septal wall of the (right ● The SA node is found at the opening of
atrium) RA the SVC in the right atrium (RA).
B. At the opening of the superior
vena cava (SVC) in the RA ● The AV node is found in the septal wall
C. At the membranous part of the of the RA.
interventricular septum
D. At all 3 locations above ● The bundle of His is found at the
membranous part of the interventricular
septum.

Reference:
Halliday, N. L., & Chung, H. M. (2018). BRS Gross
Anatomy (9th ed., p. 77). Lippincott Williams and
Wilkins.

8. The blood supply to the SA node is A A. Right coronary artery

39
provided by ____:
The RCA provides blood supply to the SA
A. Right coronary artery node through the SA nodal artery.
B. Left coronary artery
C. Left circumflex artery Reference:
D. Posterior descending artery Halliday, N. L., & Chung, H. M. (2018). BRS Gross
Anatomy (9th ed., p. 77). Lippincott Williams and
Wilkins.

9. Which of the following is considered C C. Internal vertebral venous plexus


the route of early metastasis of
carcinomas such as prostate cancer to The internal vertebral venous plexus is
the spine? considered the route of early metastasis to
the spine and CNS carcinomas such as:
A. External iliac vein ● Lung cancer
B. Internal iliac vein ● Breast cancer
C. Internal vertebral venous ● Prostate cancer
plexus
D. All of the above Reference:
Halliday, N. L., & Chung, H. M. (2018). BRS Gross
Anatomy (9th ed., p. 30). Lippincott Williams and
Wilkins.

10. What vein receives the A A. Azygos vein


hemiazygos and accessory
hemiazygos veins and directly empties The azygos vein receives the hemiazygos
into the SVC? and accessory hemiazygos veins and directly
empties into the SVC.
A. Azygos vein
B. Superior intercostal vein Reference:
C. Posterior intercostal vein Halliday, N. L., & Chung, H. M. (2018). BRS Gross
Anatomy (9th ed., p. 86). Lippincott Williams and
D. Thoracic duct Wilkins.

11. What is the estimated capacity of B B. 30 mL


the gallbladder?
The approximate capacity of the gallbladder
A. 3 mL is 30-50mL.
B. 30 mL
C. 10 mL Reference:
D. 100 mL Halliday, N. L., & Chung, H. M. (2018). BRS Gross
Anatomy (9th ed., p. 128). Lippincott Williams and
Wilkins.

12. The lymphatic drainage of the B B. Internal iliac nodes


upper part of the vagina goes into the
____: The lymphatic drainage of the upper (¾)
part of the vagina goes into the internal iliac
A. Superficial inguinal nodes nodes.
B. Internal iliac nodes
C. Deep inguinal nodes The lymphatic drainage of the lower ¼ of
D. Para-aortic lymph nodes the vagina drains into the superficial inguinal
nodes.

Reference:
Halliday, N. L., & Chung, H. M. (2018). BRS Gross
Anatomy (9th ed., p. 196). Lippincott Williams and
Wilkins.

40
13. The lymphatic drainage of the D D. Para-aortic lymph nodes
testes goes into the ___:
The lymphatic drainage of the
A. Superficial inguinal nodes testes/ovaries/uterine fundus goes into the
B. Internal iliac nodes para-aortic lymph nodes.
C. Deep inguinal nodes
D. Para-aortic lymph nodes Reference:
Le, T., Bhushan, V., & Sochat, M. (2023). First Aid for
the USMLE Step 1 2022, Thirty Third Edition (33rd ed.,
p. 644). McGraw-Hill Education.

14. Which of the following shoulder A A. Supraspinatus


muscles is responsible for the initial
abduction of the arm before the action The rotator cuff muscles consist of SITS:
of the deltoid? ● Supraspinatus
● Infraspinatus
A. Supraspinatus ● Teres minor
B. Infraspinatus ● Subscapularis
C. Teres minor
D. Subscapularis Supraspinatus is responsible for the initial
abduction of the arm before the action of the
deltoid.

Reference:
Le, T., Bhushan, V., & Sochat, M. (2023). First Aid for
the USMLE Step 1 2022, Thirty Third Edition (33rd ed.,
p. 451). McGraw-Hill Education.

15. Which of the following rotator cuff A A. Supraspinatus


muscles is most commonly injured?
Supraspinatus is the most commonly injured
A. Supraspinatus rotator cuff muscle.
B. Infraspinatus
C. Teres minor Reference:
D. Subscapularis Le, T., Bhushan, V., & Sochat, M. (2023). First Aid for
the USMLE Step 1 2022, Thirty Third Edition (33rd ed.,
p. 451). McGraw-Hill Education.

16. A patient was accidentally hit by a C C. Common peroneal nerve


car on his left leg. He now comes to
the clinic complaining of loss of Injury to the lateral aspect of the leg can
sensation on the dorsal aspect of his result to damage to the common peroneal
left foot and inability to lift the foot nerve which leads to loss of sensation on the
while walking resulting to a “steppage dorsal aspect of the affected foot and inability
gait”. to lift the foot while walking resulting to a
“steppage gait”.
What is the most likely injured nerve? This inability to evert and dorsiflex the foot is
also called foot drop.
A. Tibial nerve
B. Superior gluteal nerve A useful mnemonic from First Aid is:
C. Common peroneal nerve
D. Inferior gluteal nerve PED → Peroneal Everts and Dorsiflexes.

If Common Peroneal Nerve gets injured, it


will result to foot dropPED

41
Reference:
Le, T., Bhushan, V., & Sochat, M. (2023). First Aid for
the USMLE Step 1 2022, Thirty Third Edition (33rd ed.,
p. 457). McGraw-Hill Education.

17. Which of the following ligaments in A A. Anterior talofibular ligament


most commonly injured in ankle
sprains? Anterior talofibular ligament is the most
commonly injured ligament in ankle sprains.
A. Anterior talofibular ligament
B. Posterior talofibular ligament Reference:
C. Posterior inferior tibiofibular Le, T., Bhushan, V., & Sochat, M. (2023). First Aid for
the USMLE Step 1 2022, Thirty Third Edition (33rd ed.,
ligament p. 458). McGraw-Hill Education.
D. Anterior inferior tibiofibular
ligament

18. A patient was brought to the ER A A. L4


for a possible disc herniation.
On examination, there was noted Disc herniation that affects the L4 nerve root
weakness of knee extension and
decreased patellar reflex. → weakness of knee extension and
decreased patellar reflex.
What is the most likely nerve root
affected?
Nerve root Motor function affected
A. L4 injured
B. L5
C. S1 L4 ● Weakness of knee
D. S2 extension
● Decreased patellar reflex

L5 ● Weakness of dorsiflexion
● Heel walking

S1 ● Decreased Achilles
reflex
● Difficulty in toe walking
● Weakness of plantar
flexion

Reference:
Le, T., Bhushan, V., & Sochat, M. (2023). First Aid for
the USMLE Step 1 2022, Thirty Third Edition (33rd ed.,
p. 458). McGraw-Hill Education.

19. Injury to the surgical neck of the B B. Axillary


humerus will damage which of the
following nerves? ● Trauma or surgery on the axillary
A. Long thoracic area/lateral thorax → injury to the long
B. Axillary thoracic nerve and lateral thoracic artery
C. Radial
D. Median
● Injury to the surgical neck of the

42
humerus → injury to the axillary nerve
and posterior circumflex artery

● Injury to the midshaft of the humerus


→ injury to the radial nerve and deep
brachial artery

● Injury to the distal humerus/cubital


fossa → injury to the median nerve and
brachial artery

Reference:
Le, T., Bhushan, V., & Sochat, M. (2023). First Aid for
the USMLE Step 1 2022, Thirty Third Edition (33rd ed.,
p. 458). McGraw-Hill Education.

20. What artery is involved with injury A A. Deep brachial


to the midshaft of the humerus?
● Trauma or surgery on the axillary
A. Deep brachial
B. Posterior circumflex area/lateral thorax → injury to the long
C. Lateral thoracic thoracic nerve and lateral thoracic artery
D. Brachial
● Injury to the surgical neck of the
humerus → injury to the axillary nerve
and posterior circumflex artery

● Injury to the midshaft of the humerus


→ injury to the radial nerve and deep
brachial artery

● Injury to the distal humerus/cubital


fossa → injury to the median nerve and
brachial artery

Reference:
Le, T., Bhushan, V., & Sochat, M. (2023). First Aid for
the USMLE Step 1 2022, Thirty Third Edition (33rd ed.,
p. 458). McGraw-Hill Education.

21. An elderly patient with B B. Inferior gluteal nerve


osteoporosis suffers a right posterior
hip dislocation. Which of the following ● The inferior gluteal nerve innervates the
nerves is most likely injured in this gluteus maximus and is associated with
case? posterior hip dislocations.

A. Tibial nerve ● The tibial nerve gets injured in cases of


B. Inferior gluteal nerve knee trauma, Baker cyst and tarsal tunnel
C. Pudendal nerve syndrome.
D. Common peroneal nerve
● The pudendal nerve is commonly
injured in prolonged cycling, horseback

43
riding or during childbirth as a stretch
injury.

Injury to the lateral aspect of the leg can


result to damage to the common peroneal
nerve which leads to loss of sensation on the
dorsal aspect of the affected foot and inability
to lift the foot while walking resulting to a
“steppage gait”.

This inability to evert and dorsiflex the foot is


also called foot drop.

A useful mnemonic from First Aid is:

PED → Peroneal Everts and Dorsiflexes.

If Common Peroneal Nerve gets injured, it


will result to foot dropPED

Reference:
Le, T., Bhushan, V., & Sochat, M. (2023). First Aid for
the USMLE Step 1 2022, Thirty Third Edition (33rd ed.,
p. 457). McGraw-Hill Education.

22. When taking blood pressure in the C C. Brachial artery


arm, the arterial pulse of what artery
(found in the cubital fossa) is being The brachial artery is found in the cubital
listened to? fossa.

A. Posterior circumflex artery It is lateral to the median nerve, medial to the


B. Ulnar artery biceps tendon, and deep to the bicipital
C. Brachial artery aponeurosis.
D. Radial artery
Medial → Lateral
● Median Nerve
● Brachial artery
● Tendon of biceps
Radial nerve
Posterior interosseus branch of radial
nerve
We listen to its arterial pulse when taking
blood pressure.

Reference:
Halliday, N. L., & Chung, H. M. (2018). BRS Gross
Anatomy (9th ed., p. 316). Lippincott Williams and
Wilkins.

23. A patient comes in with signs and A A. Superficial inguinal nodes


symptoms consistent with an infected
Bartholin cyst. Which of the following The lymphatic drainage of the labia majora
lymph nodes would most likely get (where the Bartholin glands are located) is

44
swollen? through the superficial inguinal lymph
nodes.
A. Superficial inguinal nodes
B. Internal iliac nodes Reference:
C. Deep inguinal nodes Halliday, N. L., & Chung, H. M. (2018). BRS Gross
Anatomy (9th ed., p. 196). Lippincott Williams and
D. Para-aortic lymph nodes Wilkins.

24. A male patient comes to the ER C C. Spongy urethra


for a straddle injury. On examination,
there’s a note of scrotal hematoma. Injury to the spongy urethra can occur in a
perineal straddle injury. This is also called
Which part of the urethra is most likely anterior urethral injury.
injured?
Injury to the spongy urethra may manifest as
A. Prostatic urethra scrotal hematoma.
B. Membranous urethra
C. Spongy urethra Reference:
D. Any of the above Le, T., Bhushan, V., & Sochat, M. (2023). First Aid for
the USMLE Step 1 2022, Thirty Third Edition (33rd ed.,
p. 647). McGraw-Hill Education.

25. What is the main arterial blood A A. Left anterior descending artery
supply to the apex of the heart?
The LAD artery is the main arterial blood
A. Left anterior descending artery supply to the cardiac apex.
B. Left circumflex artery
C. Right coronary artery Reference:
D. SA nodal artery Halliday, N. L., & Chung, H. M. (2018). BRS Gross
Anatomy (9th ed., p. 78). Lippincott Williams and
Wilkins.

26. The common bile and main B B. Descending


pancreatic ducts open into what part
of the duodenum? The common bile and main pancreatic ducts
open into the major duodenal papilla which is
A. Superior found in the descending (2nd) part of the
B. Descending duodenum.
C. Transverse
D. Ascending Reference:
Halliday, N. L., & Chung, H. M. (2018). BRS Gross
Anatomy (9th ed., p. 123). Lippincott Williams and
Wilkins.

27. Cerebrospinal fluid made in the C C. Aqueduct of Sylvius


choroid plexus flows into the 4th
ventricle from the 3rd ventricle via the CSF flow:
______:
● Lateral ventricles → Left and right
A. Foramina of Luschka
B. Foramina of Magendie interventricular foramina of Monro → 3rd
C. Aqueduct of Sylvius ventricle
D. Foramina of Monro

● 3rd ventricle → Cerebral aqueduct of

45
Sylvius → 4th ventricle

● 4th ventricle → Foramina of Luschka


(lateral) and foramina of Magendie
(medial) → subarachnoid space

Reference:
Le, T., Bhushan, V., & Sochat, M. (2023). First Aid for
the USMLE Step 1 2022, Thirty Third Edition (33rd ed.,
p. 516). McGraw-Hill Education.

28. Taste sensation from the anterior B B. CN VII


⅔ of the tongue is carried by which of
the following cranial nerves? Taste from the anterior ⅔ of the tongue is
carried by chorda tympani which is a branch
A. CN V of CN VII.
B. CN VII
C. CN IX Cranial Innervation
D. CN X
nerve

CN V Somatosensation from the


anterior ⅔ of the tongue, not
taste sensation.

CN VII Taste from the anterior ⅔ of


the tongue

CN IX Taste and sensation from the


posterior ⅓ of the tongue.

Reference:
Le, T., Bhushan, V., & Sochat, M. (2023). First Aid for
the USMLE Step 1 2022, Thirty Third Edition (33rd ed.,
p. 521). McGraw-Hill Education.

29. What is the afferent branch of the A A. CN II


pupillary reflex? Branch Pupillary Corneal
reflex reflex
A. CN II
B. CN III Afferent CN II CN V1
C. CN V
D. CN VII Efferent CN III CN VII
Reference:
Le, T., Bhushan, V., & Sochat, M. (2023). First Aid for
the USMLE Step 1 2022, Thirty Third Edition (33rd ed.,
p. 521). McGraw-Hill Education.

30. The breast is supplied by the A A. Subclavian


medial mammary branches from the
internal thoracic artery. The internal thoracic artery comes from the
The internal thoracic artery comes subclavian artery.
from what artery?
The rest of the choices are the other

46
A. Subclavian branches of the subclavian artery.
B. Costocervical
C. Vertebral The branches of the subclavian artery
D. Thyrocervical include:
● Vertebral artery
● Thyrocervical trunk
● Internal thoracic artery
● Costocervical trunk

Reference:
Halliday, N. L., & Chung, H. M. (2018). BRS Gross
Anatomy (9th ed., p. 352). Lippincott Williams and
Wilkins.

31. The following are the contents of C C. External jugular vein


the carotid sheath, EXCEPT?
The contents of the carotid sheath are:
A. Internal carotid arteries ● Common and internal carotid arteries
B. Internal jugular vein ● Internal jugular vein
C. External jugular vein ● Vagus nerve
D. Vagus nerve
Reference:
Halliday, N. L., & Chung, H. M. (2018). BRS Gross
Anatomy (9th ed., p. 361). Lippincott Williams and
Wilkins.

32. The 5 secondary vesicles are the A A. Telencephalon


telencephalon, diencephalon,
mesencephalon, metencephalon, and ● Telencephalon → Cerebral hemispheres
myelencephalon.
and basal ganglia
The basal ganglia develops from
which of those 5? ● Diencephalon → Thalamus,
A. Telencephalon hypothalamus
B. Diencephalon
C. Mesencephalon ● Mesencephalon → Midbrain
D. Metencephalon

● Metencephalon → Pons, cerebellum

● Myelencephalon → Medulla

Reference:
Le, T., Bhushan, V., & Sochat, M. (2023). First Aid for
the USMLE Step 1 2022, Thirty Third Edition (33rd ed.,
p. 500). McGraw-Hill Education.

33. Which of the following is a branch D D. None of the above


of the internal carotid artery in the
neck? The internal carotid artery has no branches
in the neck.
A. Superior thyroid artery
B. Maxillary artery The rest of the options are branches of the
C. Superficial temporal artery external carotid artery in the neck.

47
D. None of the above
Reference:
Halliday, N. L., & Chung, H. M. (2018). BRS Gross
Anatomy (9th ed., p. 435). Lippincott Williams and
Wilkins.

34. Which of the following D D. Tuberoinfundibular pathway


dopaminergic pathways is responsible
for regulating the secretion of Dopaminergic pathways:
prolactin?
● Mesocortical and mesolimbic pathway
A. Mesocortical pathway → motivation and reward
B. Mesolimbic pathway
C. Nigrostriatal pathway
D. Tuberoinfundibular pathway ● Nigrostriatal pathway → motor control

● Tuberoinfundibular → prolactin
secretion

Reference:
Le, T., Bhushan, V., & Sochat, M. (2023). First Aid for
the USMLE Step 1 2022, Thirty Third Edition (33rd ed.,
p. 510). McGraw-Hill Education.

35. What structure perforates the B B. Esophagus


diaphragm at the T10 level?
● T8 level → right phrenic nerve and IVC
A. IVC
B. Esophagus
C. Aorta ● T10 level → Vagus nerve, esophagus
D. Right phrenic nerve
● T12 level → Aorta, thoracic duct, azygos
vein

Reference:
Le, T., Bhushan, V., & Sochat, M. (2023). First Aid for
the USMLE Step 1 2022, Thirty Third Edition (33rd ed.,
p. 525). McGraw-Hill Education.

36. The inguinal ligament is in what D D. L1


dermatomal level?
● T10 dermatomal level → umbilicus
A. T10
B. T11 ● L1 dermatomal level → inguinal ligament
C. T12
D. L1 Reference:
Le, T., Bhushan, V., & Sochat, M. (2023). First Aid for
the USMLE Step 1 2022, Thirty Third Edition (33rd ed.,
p. 525). McGraw-Hill Education.

37. What is the medial border of the A A. Linea semilunaris


inguinal triangle?

48
The inguinal triangle (Hesselbach
A. Linea semilunaris triangle) has the following borders:
B. Inferior epigastric vessels ● Medial – linea semilunaris (lateral
C. Pubic tubercle edge of the rectus abdominis)
D. Inguinal ligament ● Lateral – Inferior epigastric vessels
● Inferior – Inguinal ligament

Reference:
Halliday, N. L., & Chung, H. M. (2018). BRS Gross
Anatomy (9th ed., p. 112). Lippincott Williams and
Wilkins.

38. Which of the following is NOT true D D. Occurs medial to inferior epigastric
of indirect inguinal hernias? vessels

A. Goes through deep inguinal Indirect inguinal hernias:


ring ● Goes through the deep inguinal ring →
B. Goes through superficial
inguinal ring inguinal canal → superficial inguinal ring
C. Descends into the scrotum → descends into scrotum
D. Occurs medial to the inferior
● Occurs lateral to inferior epigastric
epigastric vessels
vessels

Direct inguinal hernias:


● Protrudes directly through the
Hesselbach triangle
● Occurs medial to inferior epigastric
vessels

References:
● Halliday, N. L., & Chung, H. M. (2018). BRS Gross
Anatomy (9th ed., p. 113). Lippincott Williams and
Wilkins.
● Le, T., Bhushan, V., & Sochat, M. (2023). First Aid
for the USMLE Step 1 2022, Thirty Third Edition
(33rd ed., p. 377). McGraw-Hill Education.

39. Which of the following layers of the D D. Internal spermatic fascia: Internal oblique
spermatic cord is incorrectly paired? muscle

A. External spermatic fascia: The layers of the spermatic cord are the
external oblique muscle following:
B. Cremasteric fascia: Internal
oblique muscle Internal spermatic fascia: Transversalis
C. Internal spermatic fascia: fascia
transversalis fascia
D. Internal spermatic fascia: Cremasteric fascia: Internal oblique muscle
Internal oblique muscle
External spermatic fascia: External oblique
muscle

A useful mnemonic from First Aid is:


ICE- TIE

References:

49
● Halliday, N. L., & Chung, H. M. (2018). BRS Gross
Anatomy (9th ed., p. 114). Lippincott Williams and
Wilkins.
● Le, T., Bhushan, V., & Sochat, M. (2023). First Aid
for the USMLE Step 1 2022, Thirty Third Edition
(33rd ed., p. 376). McGraw-Hill Education.

40. Which of the following hernias is A A. Femoral hernia


more common in females and likely
presents with incarceration? Femoral hernia is more common in females
and more likely presents with incarceration or
A. Femoral hernia strangulation.
B. Indirect inguinal hernia
C. Direct inguinal hernia Reference:
D. Spigelian hernia Le, T., Bhushan, V., & Sochat, M. (2023). First Aid for
the USMLE Step 1 2022, Thirty Third Edition (33rd ed.,
p. 377). McGraw-Hill Education.

41. What is the largest and most A A. Astrocytes


abundant glial cell in the CNS?
Astrocytes are the largest and most
A. Astrocytes abundant glial cells in the central nervous
B. Neurons system.
C. Microglia
D. Ependymal cell Reference:
Le, T., Bhushan, V., & Sochat, M. (2023). First Aid for
the USMLE Step 1 2022, Thirty Third Edition (33rd ed.,
p. 503). McGraw-Hill Education.

42. Which of the following cells A A. Microglia


originated from the mesoderm?
Microglia originate from the mesoderm.
A. Microglia
B. Astrocytes Astrocytes and oligodendrocytes originate
C. Oligodendrocytes from neural tube which is from the ectoderm.
D. Schwann cells Schwann cells are from the neural crest
which is also from the ectoderm.

Reference:
Le, T., Bhushan, V., & Sochat, M. (2023). First Aid for
the USMLE Step 1 2022, Thirty Third Edition (33rd ed.,
p. 633). McGraw-Hill Education.

43. Which of the following structures B B. Hepatoduodenal ligament


contains the portal triad?
The hepatoduodenal ligament contains the
A. Hepatogastric ligament portal triad which consists of the following:
B. Hepatoduodenal ligament ● Bile duct
C. Greater omentum ● Proper hepatic artery
D. All of the above ● Portal vein

The hepatoduodenal ligament is a part of the


lesser omentum.

Reference:
Halliday, N. L., & Chung, H. M. (2018). BRS Gross
Anatomy (9th ed., p. 168). Lippincott Williams and
Wilkins.

50
44. Which of the following forms the A A. Ventral bud
uncinate process of the pancreas?
The uncinate process is formed from the
A. Ventral bud ventral pancreatic bud.
B. Dorsal bud
C. Lateral bud The rest of the head, body and tail of the
D. Medial bud pancreas is formed from the dorsal
pancreatic bud.

Reference:
Halliday, N. L., & Chung, H. M. (2018). BRS Gross
Anatomy (9th ed., p. 132). Lippincott Williams and
Wilkins.

45. The following structures in the A A. Respiratory bronchioles


respiratory tract function to warm and
humidify inspired air, EXCEPT? The conduction portion of the respiratory
tract function to warm, humidify and filter
A. Respiratory bronchioles inhaled air. These include:
B. Terminal bronchioles ● Nasal cavity
C. Bronchi ● Nasopharynx
D. Trachea ● Larynx
● Trachea
● Bronchi
● Terminal bronchioles

The respiratory portion is where gas


exchange occurs. This includes:
● Respiratory bronchioles
● Alveolar ducts
● Alveolar sacs

Reference:
Halliday, N. L., & Chung, H. M. (2018). BRS Gross
Anatomy (9th ed., p. 64). Lippincott Williams and
Wilkins.

46. The following are characteristics of D D. Longer


the right main bronchus which makes
it more susceptible to obstruction by The right main bronchus is more susceptible
inhaled foreign bodies, EXCEPT? to obstruction by inhaled foreign bodies due
to the following characteristics:
A. Shorter ● Shorter
B. Wider ● Wider
C. More vertical ● More vertical
D. Longer
Reference:
Halliday, N. L., & Chung, H. M. (2018). BRS Gross
Anatomy (9th ed., p. 59). Lippincott Williams and
Wilkins.

47. The recurrent laryngeal nerve D D. Cricothyroid


innervates all the intrinsic muscles of
the larynx, EXCEPT? The cricothyroid is innervated by the
external laryngeal nerve.
A. Posterior cricoarytenoid

51
B. Lateral cricoarytenoid The rest of the muscles of the larynx are
C. Vocalis innervated by the recurrent laryngeal
D. Cricothyroid nerve.

Reference:
Halliday, N. L., & Chung, H. M. (2018). BRS Gross
Anatomy (9th ed., pp. 425-426). Lippincott Williams
and Wilkins.

48. Which of the following tracts in the A A. Corticospinal tracts


spinal cord is responsible for voluntary
and motor function? ● The corticospinal tracts in the spinal
cord (anterior and lateral) are responsible
A. Corticospinal tracts for voluntary and motor function.
B. Dorsal column ● The dorsal column is responsible for
C. Anterior spinothalamic tract pressure, vibration, fine touch, and
D. None of the above proprioception.

● The lateral spinothalamic tracts are


responsible for pain and temperature
sensation

● The anterior spinothalamic tracts are


for crude touch and pressure.

Reference:
Le, T., Bhushan, V., & Sochat, M. (2023). First Aid for
the USMLE Step 1 2022, Thirty Third Edition (33rd ed.,
p. 523). McGraw-Hill Education.

49. Which hypothalamic area is C C. Lateral


involved in increasing appetite?
The lateral area of the hypothalamus is
A. Anterior known as the hunger center and is involved
B. Posterior in increasing appetite.
C. Lateral
D. All of the above The anterior hypothalamus is involved in
increasing the temperature set point leading
to fever.

References:
● Halliday, N. L., & Chung, H. M. (2018). BRS Gross
Anatomy (9th ed., p. 209). Lippincott Williams and
Wilkins.
● Le, T., Bhushan, V., & Sochat, M. (2023). First Aid
for the USMLE Step 1 2022, Thirty Third Edition
(33rd ed., p. 340). McGraw-Hill Education.

50. The ureter may be obstructed by a D D. Where it crosses below the gonadal
kidney stone in either of the following arteries
locations, EXCEPT?
The common points of ureteral
A. Where the ureter joins the obstruction are the following:
renal pelvis
B. Where the ureter crosses the ● Where the ureter joins the renal
pelvic brim pelvis (ureteropelvic junction)
C. Where it enters the wall of the ● Where the ureter crosses the pelvic

52
urinary bladder brim (pelvic inlet)
D. Where it crosses below the ● Where it enters the wall of the urinary
gonadal arteries bladder (ureterovesical junction)

Reference:
● Halliday, N. L., & Chung, H. M. (2018). BRS Gross
Anatomy (9th ed., p. 141. Lippincott Williams and
Wilkins.
● Le, T., Bhushan, V., & Sochat, M. (2023). First Aid
for the USMLE Step 1 2022, Thirty Third Edition
(33rd ed., p. 601). McGraw-Hill Education.

51. Different parts of the ureter have D D. Superior vesical artery


different sources of blood supply. The
distal ureter’s blood supply comes Ureteral blood supply:
from which of the following? ● Proximal ureter→ renal arteries
A. Renal arteries ● Middle of the ureter → gonadal
B. Gonadal artery artery, aorta, common and internal
C. External iliac artery
D. Superior vesical artery iliac artery
● Distal ureter → internal iliac artery
and superior vesical artery
Reference:
Le, T., Bhushan, V., & Sochat, M. (2023). First Aid for
the USMLE Step 1 2022, Thirty Third Edition (33rd ed.,
p. 601). McGraw-Hill Education.

52. Which of the following cells C C. Leydig cells


produce testosterone in the presence
of LH? Leydig cells produce testosterone under the
control of LH.
A. Sertoli cells
B. Spermatogonia Sertoli cells are responsible for:
C. Leydig cells ● Nourishing and supporting
D. Granulosa cells spermatozoa and regulating
spermatogenesis
● Secreting inhibin B to inhibit FSH
secretion

Reference:
Le, T., Bhushan, V., & Sochat, M. (2023). First Aid for
the USMLE Step 1 2022, Thirty Third Edition (33rd ed.,
p. 648). McGraw-Hill Education.

53. What vessel carries blood from the A A. Pulmonary veins


alveoli in the lungs to the left atrium?
The pulmonary veins carry blood from the
A. Pulmonary veins alveoli in the lungs to the left atrium.
B. Left pulmonary artery
C. Right pulmonary artery The pulmonary arteries carry blood from
D. Bronchial veins the RV to the lungs for oxygenation.

Reference:
Halliday, N. L., & Chung, H. M. (2018). BRS Gross

53
Anatomy (9th ed., p. 65). Lippincott Williams and
Wilkins.

54. The site of the obliterated A A. Foramen cecum


thyroglossal duct is called ___ :
Foramen cecum is the site of an obliterated
A. Foramen cecum thyroglossal duct.
B. Foramen ovale
C. Thyroglossal duct cyst Thyroglossal duct cyst is when there’s
D. None of the above failure of closure of the thyroglossal duct.

Reference:
Halliday, N. L., & Chung, H. M. (2018). BRS Gross
Anatomy (9th ed., p. 362). Lippincott Williams and
Wilkins.

55. An infection in which layer of the D D. Loose connective tissue layer


scalp is associated with increased risk
of spreading to the intracranial sinuses An infection in the loose connective tissue
via the emissary veins? layer of the scalp is associated with
increased risk of spreading to the intracranial
A. Skin sinuses via the emissary veins.
B. Dense connective tissue layer
C. Galea aponeurotica The layers of the scalp are:
D. Loose connective tissue layer ● Skin
● Connective tissue
● Aponeurosis
● Loose connective tissue
● Pericranium

Reference:
Halliday, N. L., & Chung, H. M. (2018). BRS Gross
Anatomy (9th ed., p. 368). Lippincott Williams and
Wilkins.

56. A fracture sustained at the thinnest B B. Middle meningeal artery


area of the lateral skull will result in
rupture of which of the following The thinnest part of the lateral skull is known
vessels? as the pterion.

A. Aneurysmal vessel Fracture at the pterion will rupture the middle


B. Middle meningeal artery meningeal artery resulting to an epidural
C. Bridging veins hematoma.
D. Charcot-Bouchard
microaneurysms Rupture of the bridging veins will result in
subdural hematoma.

Charcot-Bouchard microaneurysms occur


in patients with hypertension.

Reference:
Le, T., Bhushan, V., & Sochat, M. (2023). First Aid for
the USMLE Step 1 2022, Thirty Third Edition (33rd ed.,
p. 530). McGraw-Hill Education.

57. What is the longest and widest B B. Ampulla


part of the uterine tubes?

54
Ampulla is the widest and longest part of the
A. Infundibulum uterine tubes.
B. Ampulla
C. Isthmus The ampulla is the most common location of
D. Fimbriae fertilization.

References:
● Halliday, N. L., & Chung, H. M. (2018). BRS Gross
Anatomy (9th ed., p. 194). Lippincott Williams and
Wilkins.
● Le, T., Bhushan, V., & Sochat, M. (2023). First Aid
for the USMLE Step 1 2022, Thirty Third Edition
(33rd ed., p. 653). McGraw-Hill Education.

58. The thyroid isthmus usually B B. 2nd and 3rd


crosses which of the following set of
tracheal rings? The thyroid isthmus usually crosses the
2nd and 3rd (and even 4th) tracheal rings.
A. 1st and 2nd
B. 2nd and 3rd Reference:
C. 4th and 5th Halliday, N. L., & Chung, H. M. (2018). BRS Gross
Anatomy (9th ed., p. 357). Lippincott Williams and
D. 5th and 6th Wilkins.

59. Prostate cancer develops in which D D. Peripheral zone


zone of the prostate gland?
Prostate cancer develops usually in the
A. Transitional zone peripheral zone of the prostate gland.
B. Anterior zone
C. Central zone Reference:
D. Peripheral zone Halliday, N. L., & Chung, H. M. (2018). BRS Gross
Anatomy (9th ed., p. 193). Lippincott Williams and
Wilkins.

60. In a thyroidectomy procedure, B B. Inferior thyroid artery


accidental transection of the recurrent
laryngeal nerve may occur during In a thyroidectomy procedure, accidental
ligation of which of the following transection of the recurrent laryngeal nerve
arteries? may occur during ligation of the inferior
thyroid artery which leads to hoarseness of
A. Superior thyroid artery voice.
B. Inferior thyroid artery
C. Transverse cervical artery Reference:
D. Thyrocervical trunk Le, T., Bhushan, V., & Sochat, M. (2023). First Aid for
the USMLE Step 1 2022, Thirty Third Edition (33rd ed.,
p. 347). McGraw-Hill Education.

61. Eliciting the patellar reflex results B B. Quadriceps femoris.


in contraction of which muscle?
Eliciting the patellar reflex results in
A. Calves contraction of the quadriceps femoris.
B. Quadriceps femoris
C. Hamstring muscles Reference:
D. Gluteus maximus Halliday, N. L., & Chung, H. M. (2018). BRS Gross
Anatomy (9th ed., p. 247). Lippincott Williams and
Wilkins.

62. What is the most commonly A A. Scaphoid


fractured carpal bone?

55
The most commonly fractured carpal bone is
A. Scaphoid the scaphoid.
B. Lunate
C. Pisiform Reference:
D. Hamate Halliday, N. L., & Chung, H. M. (2018). BRS Gross
Anatomy (9th ed., p. 453). Lippincott Williams and
Wilkins.

63. Which of the following is NOT part D D. Abductor pollicis brevis


of the borders of the anatomic
snuffbox? The borders of the anatomic snuffbox are:
● Medial – Extensor pollicis longus
A. Extensor pollicis longus ● Lateral – Extensor pollicis brevis and
B. Extensor pollicis brevis Abductor pollicis longus
C. Abductor pollicis longus
D. Abductor pollicis brevis Reference:
Halliday, N. L., & Chung, H. M. (2018). BRS Gross
Anatomy (9th ed., p. 308). Lippincott Williams and
Wilkins.

64. Dislocation of which of the C C. Lunate


following carpal bones is associated
with carpal tunnel syndrome? Impingement of the median nerve may occur
in lunate dislocation which leads to carpal
A. Scaphoid tunnel syndrome.
B. Hamate
C. Lunate Reference:
D. Trapezoid Le, T., Bhushan, V., & Sochat, M. (2023). First Aid for
the USMLE Step 1 2022, Thirty Third Edition (33rd ed.,
p. 453). McGraw-Hill Education.

65. The “Hand of Benediction” sign, A A. Proximal median nerve


which occurs when the patient
attempts to make a fist, is a ● The “Hand of Benediction” sign, which
manifestation of a lesion in which of occurs when the patient attempts to
the following? make a fist, is a manifestation of a lesion
in the proximal median nerve.
A. Proximal median nerve
B. Distal median nerve ● Median claw sign while extending the
C. Proximal ulnar nerve
D. Distal ulnar nerve fingers → distal median nerve lesion

● Ulnar claw sign while extending the


fingers → distal ulnar nerve lesion

● Trouble making a fist while closing the


hand → proximal ulnar nerve

Reference:
Le, T., Bhushan, V., & Sochat, M. (2023). First Aid for
the USMLE Step 1 2022, Thirty Third Edition (33rd ed.,
p. 454). McGraw-Hill Education.

66. An elderly patient was brought to A A Colles fracture

56
the ER after a fall on an outstretched
hand. Her right hand shows a dinner Dinner fork deformity is characteristic of
fork deformity. Which of the following Colles fracture where the distal fragment is
is the most likely diagnosis? displaced posteriorly.

A. Colles fracture It can happen in patients with osteoporosis.


B. Smith fracture
C. Reverse Colles fracture Smith fracture or reverse Colles fracture
D. None of the above is when the distal fragment becomes
displaced anteriorly.

Reference:
Halliday, N. L., & Chung, H. M. (2018). BRS Gross
Anatomy (9th ed., p. 284). Lippincott Williams and
Wilkins.

67. The carina lies at which of the A A. Sternal angle


following levels?
The carina lies at the level of the sternal
A. Sternal angle angle.
B. Xiphoid
C. Subcostal angle Reference:
D. Cricoid cartilage Halliday, N. L., & Chung, H. M. (2018). BRS Gross
Anatomy (9th ed., p. 58). Lippincott Williams and
Wilkins.

68. Which of the following structures is A A. Median umbilical ligament


a remnant of the urachus?
● Median umbilical ligament is from the
A. Median umbilical ligament urachus
B. Medial umbilical ligaments
C. Ligamentum teres ● Medial umbilical ligaments are
D. Ligamentum venosum remnants the umbilical arteries

● Ligamentum teres hepatitis is a


remnant the umbilical vein

● Ligamentum venosum is from the


ductus venosus

References:
● Halliday, N. L., & Chung, H. M. (2018). BRS Gross
Anatomy (9th ed., p. 114). Lippincott Williams and
Wilkins.
● Le, T., Bhushan, V., & Sochat, M. (2023). First Aid
for the USMLE Step 1 2022, Thirty Third Edition
(33rd ed., p. 287). McGraw-Hill Education.

69. The esophagus is lined with ___ : A A. Nonkeratinized stratified squamous


epithelium
A. Nonkeratinized stratified
squamous epithelium The esophagus is lined with nonkeratinized
B. Keratinized stratified stratified squamous epithelium.

57
squamous epithelium
C. Simple columnar epithelium Reference:
D. Simple cuboidal epithelium Le, T., Bhushan, V., & Sochat, M. (2023). First Aid for
the USMLE Step 1 2022, Thirty Third Edition (33rd ed.,
p. 369). McGraw-Hill Education.

70. Which of the following is A A. Has closely-packed plicae circulares


characteristic of the jejunum versus
the ileum? The rest of the options are found in the
ileum.
A. Has closely-packed plicae
circulares Jejunum characteristics:
B. Longer ● Has tall and closely packed plicae
C. Shorter vasa recta circulares
D. None of the above ● Emptier
● Larger in diameter
● Thicker walled versus the ileum

Ileum characteristics:
● Longer than the jejunum
● Has shorter vasa recta

Reference:
Halliday, N. L., & Chung, H. M. (2018). BRS Gross
Anatomy (9th ed., p. 151). Lippincott Williams and
Wilkins.

71. Enlargement of which of the D D. Left atrium


following chambers of the heart can
result in compression of the The left atrium (LA) of the heart lies close to
esophagus leading to symptoms of the anterior surface of the esophagus. It is
dysphagia? the most posterior part of the heart.

A. Right atrium Therefore, LA enlargement can compress


B. Right ventricle the esophagus leading to symptoms of
C. Left ventricle dysphagia.
D. Left atrium
LA enlargement can also lead to
compression of the left recurrent laryngeal
nerve which causes hoarseness. This is also
called Ortner syndrome.

Reference:
● Halliday, N. L., & Chung, H. M. (2018). BRS Gross
Anatomy (9th ed., p. 85). Lippincott Williams and
Wilkins.
● Le, T., Bhushan, V., & Sochat, M. (2023). First Aid
for the USMLE Step 1 2022, Thirty Third Edition
(33rd ed., p. 288). McGraw-Hill Education.

72. What is the most anterior part of B B. Right ventricle


the heart?
The most anterior part of the heart is the right
A. Right atrium ventricle (RV).
B. Right ventricle
C. Left ventricle Reference:
D. Left atrium Le, T., Bhushan, V., & Sochat, M. (2023). First Aid for

58
the USMLE Step 1 2022, Thirty Third Edition (33rd ed.,
p. 288). McGraw-Hill Education.

73. Breast cancer commonly occurs in A A. Upper lateral


which quadrant of the breast?
Breast cancer occurs commonly in the upper
A. Upper lateral lateral quadrant.
B. Lower lateral
C. Upper medial Reference:
D. Lower medial Halliday, N. L., & Chung, H. M. (2018). BRS Gross
Anatomy (9th ed., p. 321). Lippincott Williams and
Wilkins.

74. The finding of skin dimpling in B B. Suspensory ligament of Cooper


patients with breast cancer is due to
involvement of which of the following? Skin dimpling is due to cancer involvement
of the suspensory ligament of Cooper.
A. Subcutaneous lymphatics
B. Suspensory ligament of Peau d’orange is due to dermal lymphatic
Cooper obstruction.
C. Sentinel nodes
D. All of the above References:
● Halliday, N. L., & Chung, H. M. (2018). BRS Gross
Anatomy (9th ed., p. 321). Lippincott Williams and
Wilkins.
● Le, T., Bhushan, V., & Sochat, M. (2023). First Aid
for the USMLE Step 1 2022, Thirty Third Edition
(33rd ed., p. 670). McGraw-Hill Education.

75. Nonunion is a common A A. Radial artery


complication in proximal scaphoid
fractures. Proximal scaphoid fractures are prone to
develop nonunion and avascular necrosis
The blood supply to this bone comes because of the retrograde blood supply from
from which of the following? a branch of the radial artery.

A. Radial artery Reference:


B. Ulnar artery Le, T., Bhushan, V., & Sochat, M. (2023). First Aid for
the USMLE Step 1 2022, Thirty Third Edition (33rd ed.,
C. Brachial artery p. 453). McGraw-Hill Education.
D. Any of the above

76. Tennis elbow results from A A. Origin of the extensor forearm muscles
inflammation of the ___ :
Tennis elbow results from inflammation of
A. Origin of the extensor forearm the origin of the extensor forearm muscles.
muscles
B. Origin of the flexor forearm Golfer’s elbow results from inflammation of
muscles the origin of the flexor forearm muscles
C. Subluxation of the radial head
D. Subluxation of the ulnar head Subluxation of the radial head is also called
nursemaid’s elbow.

Reference:
Halliday, N. L., & Chung, H. M. (2018). BRS Gross
Anatomy (9th ed., p. 321). Lippincott Williams and
Wilkins.

59
77. Which of the following organs are C C. 1st part of the duodenum
NOT retroperitoneal?
Only the 2nd to 4th parts of the duodenum is
A. Ascending colon retroperitoneal.
B. Ureter
C. 1st part of the duodenum Other retroperitoneal organs can be
D. Abdominal aorta remembered by the mnemonic SAD
PUCKER which is found in First Aid:
● Suprarenal (adrenal) glands
● Aorta and IVC
● Duodenum (2nd- 4th parts)
● Pancreas
● Ureters
● Colon
● Kidneys
● Esophagus (thoracic portion)
● Rectum (partially)
Reference:
Halliday, N. L., & Chung, H. M. (2018). BRS Gross
Anatomy (9th ed., p. 367). Lippincott Williams and
Wilkins.

78. During liver surgery, bleeding is B B. Hepatoduodenal ligament


minimized by doing the Pringle
maneuver. Which of the following The hepatoduodenal ligament, which
structures is clamped in this contains the portal triad (proper hepatic
maneuver? artery, portal vein, common bile duct), is
clamped during liver surgery to minimize
A. Falciform ligament bleeding. This is called the Pringle
B. Hepatoduodenal ligament maneuver.
C. Hepatogastric ligament
D. Splenorenal ligament References:
● Halliday, N. L., & Chung, H. M. (2018). BRS Gross
Anatomy (9th ed., p. 135). Lippincott Williams and
Wilkins.
● Le, T., Bhushan, V., & Sochat, M. (2023). First Aid
for the USMLE Step 1 2022, Thirty Third Edition
(33rd ed., p. 368). McGraw-Hill Education.

79. The blood supply to the C C. Inferior mesenteric artery


descending colon comes from the ___
: ● Foregut structures: celiac artery
● Midgut structures: superior mesenteric
A. Celiac trunk artery
B. Superior mesenteric artery ● Hindgut structures: inferior mesenteric
C. Inferior mesenteric artery artery
D. Superior rectal artery
The descending colon is a hindgut structure
which means the blood supply is from the
IMA.
Reference:
Le, T., Bhushan, V., & Sochat, M. (2023). First Aid for
the USMLE Step 1 2022, Thirty Third Edition (33rd ed.,
p. 371). McGraw-Hill Education.

80. In the femoral triangle, which of A A. Femoral artery

60
the following is found lateral to the
femoral vein? The structures in the femoral triangle, from
lateral to medial, are:
A. Femoral artery
B. Femoral nerve Nerve–Artery–Vein–Lymphatics
C. Lymph nodes
D. Inguinal ligament Remembered by the mnemonic NAVeL

Reference:
Halliday, N. L., & Chung, H. M. (2018). BRS Gross
Anatomy (9th ed., p. 244). Lippincott Williams and
Wilkins.

81. Which of the following zones in the C C. Zone III


liver is known to be the most sensitive
to metabolic toxins such as Zone I is the most oxygenated and is
paracetamol? therefore most resistant to ischemia. It also
the first to be affected by viral hepatitis.
A. Zone I
B. Zone II Zone III is the least oxygenated, therefore
C. Zone III the first to be affected by ischemia. It is also
D. Zone IV the most sensitive to metabolic toxins such
as paracetamol, ethanol, rifampin and CCl4.

Reference:
Le, T., Bhushan, V., & Sochat, M. (2023). First Aid for
the USMLE Step 1 2022, Thirty Third Edition (33rd ed.,
p. 374). McGraw-Hill Education.

82. Which of the following is NOT a B B. At the level of the thyroid cartilage
site of constriction of the esophagus?
The 3 constrictions of the esophagus are
A. At the level of the cricoid the following:
cartilage 1. At the level of the cricoid cartilage
B. At the level of the thyroid (C6) or the beginning of the
cartilage esophagus
C. Where it is crossed by the 2. Middle constriction where it is
aortic arch crossed by the aortic arch and then
D. At the level of T10 left main bronchus
3. At the level of T10 where the
esophageal hiatus is located

Reference:
Halliday, N. L., & Chung, H. M. (2018). BRS Gross
Anatomy (9th ed., p. 85). Lippincott Williams and
Wilkins.

61
83. The left testicular vein drains into C C Left renal vein
which of the following?
The right testicular vein drains directly into
A. Inferior vena cava the IVC.
B. Right renal vein
C. Left renal vein The left testicular vein drains into the left
D. Left internal iliac artery renal vein which then drains into the IVC.

The left testicular vein has higher venous


pressure than the right (because the left
testicular vein joins the left renal vein at a
90degree angle) which results to more cases
of varicocele on the left.

Reference:
● Halliday, N. L., & Chung, H. M. (2018). BRS Gross
Anatomy (9th ed., p. 191). Lippincott Williams and
Wilkins.
● Le, T., Bhushan, V., & Sochat, M. (2023). First Aid
for the USMLE Step 1 2022, Thirty Third Edition
(33rd ed., p. 644). McGraw-Hill Education.

84. Cavernous sinus thrombosis from A A. CN II


an infection will affect all of the
following structures, EXCEPT? CN II does not pass through the cavernous
sinus.
A. CN II
B. CN III The structures which pass through the
C. CN IV cavernous sinus are the following:
D. CN V1 ● CN III
● CN IV
● CN V2
● CN V1
● CN VI
● Postganglionic sympathetic pupillary
fibers
● Cavernous portion of the internal
carotid artery

Reference:
● Halliday, N. L., & Chung, H. M. (2018). BRS Gross
Anatomy (9th ed., p. 385). Lippincott Williams and
Wilkins.
● Le, T., Bhushan, V., & Sochat, M. (2023). First Aid
for the USMLE Step 1 2022, Thirty Third Edition
(33rd ed., p. 559). McGraw-Hill Education.

85. A patient with a nonfunctioning A A. At the optic chiasm


pituitary adenoma presents with
bitemporal hemianopia. ● Lesion at the optic chiasm →
bitemporal hemianopia
Where is the lesion most likely
located?
● Lesion at the left optic nerve →
A. At the optic chiasm left anopia
B. At the left optic nerve

62
C. At the Meyer loop ● Lesion at the left Meyer loop →
D. At the left visual cortex
right upper quadrantanopia

● Lesion at the left visual cortex →


right hemianopia with macular sparing

Reference:
Halliday, N. L., & Chung, H. M. (2018). BRS Gross
Anatomy (9th ed., p. 559). Lippincott Williams and
Wilkins.

86. Which of the following A A. suprachiasmatic nucleus


hypothalamic nuclei is responsible for
regulating the circadian rhythm? The suprachiasmatic nucleus is
responsible for regulating the circadian
A. Suprachiasmatic nucleus rhythm.
B. Supraoptic nucleus
C. Paraventricular nucleus The supraoptic and paraventricular nuclei
D. Anterior nucleus are responsible for the synthesis of ADH and
oxytocin.

The anterior nucleus is for cooling of the


body.

Reference:
Halliday, N. L., & Chung, H. M. (2018). BRS Gross
Anatomy (9th ed., p. 509). Lippincott Williams and
Wilkins.

87. The spleen can be located in the C C. 9th-11th ribs


left upper quadrant covered by which
set of ribs? The spleen can be located in the left upper
quadrant covered by the 9th-11th ribs.
A. 4-5th ribs
B. 6-8th ribs Reference:
C. 9-11th ribs Halliday, N. L., & Chung, H. M. (2018). BRS Gross
Anatomy (9th ed., p. 130). Lippincott Williams and
D. 10-12th ribs Wilkins.

88. Incomplete closure of the B B. Hydrocele


processus vaginalis can give rise to
which of the following? Incomplete closure of the processus
vaginalis can give rise to a congenital
A. Varicocele hydrocele.
B. Hydrocele
C. Spermatocele Varicocele is from enlargement of the
D. Any of the above pampiniform plexus, not incomplete closure
of the processus vaginalis.
Spermatocele is a cyst from a dilated
epididymal duct.

References:
● Halliday, N. L., & Chung, H. M. (2018). BRS Gross
Anatomy (9th ed., p. 176). Lippincott Williams and
Wilkins.
● Le, T., Bhushan, V., & Sochat, M. (2023). First Aid

63
for the USMLE Step 1 2022, Thirty Third Edition
(33rd ed., p. 672). McGraw-Hill Education.

89. The presence of Wernicke aphasia B B. Middle cerebral


is attributed to a stroke in what artery?
MCA stroke most likely presents with
A. Anterior cerebral Wernicke aphasia.
B. Middle cerebral
C. Posterior cerebral EFFECTS OF STROKE
D. Basilar

Artery Lesion area Signs and


symptoms

Anterior Motor and ● Contralateral


cerebral sensory paralysis and
artery cortices – sensory loss –
lower limb
lower limb
● Urinary
incontinence

Middle Motor and ● Contralateral


cerebral sensory paralysis and
artery cortices – sensory loss –
upper limb
lower face and
and face;
Temporal lobe upper limb
(Wernicke ● Aphasia if in
area); the dominant
Frontal lobe (usually left)
(Broca area) hemisphere.
● Hemineglect if
lesion affects
nondominant
(usually right)
hemisphere

Lenticulo- Striatum, ● Contralateral


striate internal paralysis
artery capsule ● Absence of
cortical signs
(e.g., neglect,
aphasia,
visual field
loss)
Adapted from Le, T., Bhushan, V., & Sochat, M.
(2022). First Aid for the USMLE Step 1 2022, Thirty
Second Edition (32nd ed.). McGraw-Hill Education.

Reference:
Le, T., Bhushan, V., & Sochat, M. (2023). First Aid for
the USMLE Step 1 2022, Thirty Third Edition (33rd ed.,
p. 528). McGraw-Hill Education

90. What is the remnant of the C C. Nucleus pulposus


embryonic notochord?
The remnant of the embryonic notochord is

64
A. Umbilical ligament the nucleus pulposus of the intervertebral
B. Ligamentumm venosum disc.
C. Nucleus pulposus
D. 1st cervical vertebra Reference:
Halliday, N. L., & Chung, H. M. (2018). BRS Gross
Anatomy (9th ed., p. 27). Lippincott Williams and
Wilkins.

91. The glomerular filtration barrier B B. Basement membrane with Type II


consists of the following, EXCEPT? collagen

A. Capillary endothelium with The glomerular basement membrane (GBM)


fenestrations is composed of type IV collagen chains, not
B. Basement membrane with type II.
Type II collagen
C. Podocyte foot processes The other components of the glomerular
D. None of the above filtration barrier are:
● Capillary endothelium with
fenestrations
● Podocyte foot processes

Reference:
Le, T., Bhushan, V., & Sochat, M. (2023). First Aid for
the USMLE Step 1 2022, Thirty Third Edition (33rd ed.,
p. 601). McGraw-Hill Education.

92. Which of the following structures in B B. Juxtaglomerular cells


the kidney produces renin?
The juxtaglomerular cells in the afferent
A. Macula densa arteriole produce renin.
B. Juxtaglomerular cells
C. Distal convoluted tubule Reference:
D. Renal medulla Halliday, N. L., & Chung, H. M. (2018). BRS Gross
Anatomy (9th ed., p. 140). Lippincott Williams and
Wilkins.

93. Which of the following zones of the C C. Zona reticularis.


adrenal gland produces androgens?
Androgens are produced in the zona
A. Zona glomerulosa reticularis.
B. Zona fasciculata
C. Zona reticularis Zona glomerulosa → aldosterone
D. Any of the above
Zona fasciculata → glucocorticoids

Reference:
Halliday, N. L., & Chung, H. M. (2018). BRS Gross
Anatomy (9th ed., p. 142). Lippincott Williams and
Wilkins.

94. The adrenal medulla is A A Neural crest


embryologically derived from the
___: The adrenal medulla is embryologically
derived from the neural crest.

65
A. Neural crest Other structures derived from the neural
B. Neural tube crest are (mnemonic: EMO-PASSES)
C. Surface ectoderm ● Enterochromaffin cells
● Melanocytes
D. Mesoderm
● Odontoblasts
● PNS ganglia
(cranial, dorsal root, autonomic)
● Adrenal medulla
● Schwann cells
● Spiral membrane
(aorticopulmonary septum)
● Endocardial cushions
(also derived partially from
mesoderm)
● Skull bones

References:
● Halliday, N. L., & Chung, H. M. (2018). BRS Gross
Anatomy (9th ed., p. 142). Lippincott Williams and
Wilkins.
● Le, T., Bhushan, V., & Sochat, M. (2023). First Aid
for the USMLE Step 1 2022, Thirty Third Edition
(33rd ed., p. 633). McGraw-Hill Education.

95. The levator ani muscle consists of D D. Coccygeus


the following, EXCEPT?
The 3 muscles that form the levator ani
A. Puborectalis muscle are:
B. Pubococcygeus 1. Puborectalis
C. Iliococcygeus 2. Pubococcygeus
D. Coccygeus 3. Iliococcygeus

The coccygeus is not part of the levator ani.

Reference:
Halliday, N. L., & Chung, H. M. (2018). BRS Gross
Anatomy (9th ed., p. 175). Lippincott Williams and
Wilkins.

96. During hysterectomy, the ligation C C. Cardinal ligament


of which of the following ligaments
may result in ureteral injury? During hysterectomy, the ligation of the
cardinal ligament (which contains the
A. Infundibulopelvic ligament uterine vessels) may result in ureteral injury.
B. Ovarian ligament
C. Cardinal ligament References:
● Halliday, N. L., & Chung, H. M. (2018). BRS Gross
D. Broad ligament Anatomy (9th ed., p. 196). Lippincott Williams and
Wilkins.
● Le, T., Bhushan, V., & Sochat, M. (2023). First Aid
for the USMLE Step 1 2022, Thirty Third Edition
(33rd ed., p. 645). McGraw-Hill Education.

97. Which of the following ligaments C C. Round ligament


connects the uterine horn to the labia
majora via the inguinal canal? The round ligament connects the uterine

66
horn to the labia majora via the inguinal
A. Infundibulopelvic ligament canal.
B. Ovarian ligament
C. Round ligament Reference:
D. Uterosacral ligament Le, T., Bhushan, V., & Sochat, M. (2023). First Aid for
the USMLE Step 1 2022, Thirty Third Edition (33rd ed.,
p. 645). McGraw-Hill Education.

98. Which of the following is B B. Labia majora


homologous to the scrotum?
The labia majora is homologous to the
A. Labia minora scrotum.
B. Labia majora
C. Clitoris The clitoris is homologous to the penis.
D. Pudendal nerve
Reference:
Halliday, N. L., & Chung, H. M. (2018). BRS Gross
Anatomy (9th ed., p. 206). Lippincott Williams and
Wilkins.

99. The phrenic nerve arises from C C. C3 – C5


which of the following cervical nerves?
The phrenic nerve arises from C3-C5.
A. C1 – C2
B. C2 – C4 A useful mnemonic from First Aid is shown
C. C3 – C5 below.
D. C6 – C7
C3,4,5 keeps the diaphragm alive

References:
● Halliday, N. L., & Chung, H. M. (2018). BRS Gross
Anatomy (9th ed., p. 67). Lippincott Williams and
Wilkins.
● Le, T., Bhushan, V., & Sochat, M. (2023). First Aid
for the USMLE Step 1 2022, Thirty Third Edition
(33rd ed., p. 683). McGraw-Hill Education.

100. What cranial nerve is responsible B B. CN III


for eyelid opening by innervating the
levator palpebrae superioris? The levator palpebrae superioris is
innervated by CN III. This muscle opens the
A. CN II eyelids.
B. CN III
C. CN IV References:
D. CN VI ● Halliday, N. L., & Chung, H. M. (2018). BRS Gross
Anatomy (9th ed., p. 393). Lippincott Williams and
Wilkins.
● Le, T., Bhushan, V., & Sochat, M. (2023). First Aid
for the USMLE Step 1 2022, Thirty Third Edition
(33rd ed., p. 521). McGraw-Hill Education.

67
PHYSIOLOGY
Biochemistry
Biochemistry

68
QUESTION ANSWER EXPLANATION

1. Which of the following intercellular B B. Gap junctions


connections enable the flow of current
in myocardial cells? Gap junctions allow the flow of current
between myocardial cells.
A. Zonula occludens
B. Gap junctions Tight junctions, also called zonula
C. Tight junctions occludens, may either be:
D. Hemidesmosome ● Tight such as in the distal
tubules of the kidney, or
● Leaky, such as in the proximal
tubules of the kidney or the
gallbladder

Reference:
Costanzo, L.S. (2018). BRS Physiology (7th ed.,
p. 14). Lippincott Williams and Wilkins.

2. Which of the following cell junctions D D. Hemidesmosomes


connects the basal cells to the
basement membrane underneath it? Hemidesmosomes connect the basal
cells to the basement membrane
A. Zonula occludens underneath it.
B. Gap junctions
C. Tight junctions Gap junctions allow the flow of current
D. Hemidesmosome between myocardial cells.

Tight junctions, also called zonula


occludens, may either be:
● Tight such as in the distal
tubules of the kidney, or
● Leaky, such as in the proximal
tubules of the kidney or the
gallbladder

Reference:
Le, T., Bhushan, V., & Sochat, M. (2023). First
Aid for the USMLE Step 1 2022, Thirty Third
Edition (33rd ed., p. 482). McGraw-Hill
Education.

3. The following are considered A A. GABA


excitatory neurotransmitters, EXCEPT?
GABA is an inhibitory neurotransmitter.
A. GABA
B. Glutamate The rest of the options are excitatory.
C. Norepinephrine
D. Serotonin Reference:
Costanzo, L.S. (2018). BRS Physiology (7th ed.,
p. 36). Lippincott Williams and Wilkins.

4. Which of the following troponin C C. Troponin C


allows the binding of actin to myosin?
● Troponin C allows the binding of
A. Troponin T actin to myosin by exposing the

69
B. Troponin I myosin-binding sites.
C. Troponin C
D. Troponin H ● Troponin I inhibits the binding of
actin to myosin.

● Troponin T attaches tropomyosin


to the troponin complex.

Reference:
Costanzo, L.S. (2018). BRS Physiology (7th ed.,
p. 42). Lippincott Williams and Wilkins.

5. The binding of Ca2+ to calmodulin is B B. Smooth muscle


the basis of contraction for which of the
following muscles? The molecular basis of contraction of
skeletal and cardiac muscle is Ca2+ –
A. Skeletal muscle troponin C.
B. Smooth muscle
C. Cardiac muscle Reference:
D. All of the above Costanzo, L.S. (2018). BRS Physiology (7th ed.,
p. 53). Lippincott Williams and Wilkins.

6. Which of the following is C C. Anaerobic metabolism is the


characteristic of Type II skeletal muscle predominant metabolism
fibers?
Type I skeletal muscle fiber
A. Slow contraction velocity characteristics:
B. Red color ● Slow contraction velocity
C. Anaerobic metabolism is the ● Red fiber color
predominant metabolism ● Oxidative phosphorylation is the
D. None of the above predominant metabolism

Type II skeletal muscle fiber


characteristics:
● Fast contraction velocity
● White fiber color
● Anaerobic glycolysis is the
predominant metabolism

Reference:
Le, T., Bhushan, V., & Sochat, M. (2023). First
Aid for the USMLE Step 1 2022, Thirty Third
Edition (33rd ed., p. 460). McGraw-Hill
Education.

7. A lesion affecting the optic nerve A A. Ipsilateral eye blindness


will result to ___:
Location of lesion Disease
A. Ipsilateral eye blindness associated
B. Bitemporal hemianopia
C. Homonymous contralateral Optic nerve Ipsilateral eye
hemianopia blindness
D. Homonymous hemianopia with

70
macular sparing Optic chiasm Bitemporal
hemianopia

Optic tract Homonymous


contralateral
hemianopia

Geniculocalcarine Homonymous
tract hemianopia with
macular sparing

Reference:
Costanzo, L.S. (2018). BRS Physiology (7th ed.,
p. 86). Lippincott Williams and Wilkins.

8. A patient with myopia will benefit B B. Biconcave lens


from using a ___ lens:
● Myopia → corrected with biconcave
A. Cylindric lens
B. Biconcave lens lens
C. Biconvex lens
D. Any of the above ● Hyperopia → corrected with
biconvex lens

● Astigmatism → corrected with


cylindric lens

Reference:
Costanzo, L.S. (2018). BRS Physiology (7th ed.,
p. 85). Lippincott Williams and Wilkins.

9. The medulla contains the following D D. Pneumotaxic


autonomic centers, EXCEPT?
The medulla contains the following
A. Vasomotor centers:
B. Respiratory ● Vasomotor
C. Swallowing ● Respiratory
D. Pneumotaxic ● Swallowing, coughing, vomiting

The pneumotaxic center is in the pons.

Reference:
Costanzo, L.S. (2018). BRS Physiology (7th ed.,
p. 78). Lippincott Williams and Wilkins.

10. A substance with increased B. Conduction velocity


dromotropic effects will result in
increased ___: ● Increased chronotropic effect →
increased HR
A. Heart rate
B. Conduction velocity
C. Contractility ● Increased dromotropic effect →
D. Activation of ionotropic increased conduction velocity
receptors through the AV node

71
● Increased inotropic effect →
increased contractility

Reference:
Costanzo, L.S. (2018). BRS Physiology (7th ed.,
p. 144). Lippincott Williams and Wilkins.

11. A sarcomere runs from ____: A A. Z line to Z line

A. Z line to Z line A sarcomere runs from Z line to Z line.


B. I band to I band
C. A band to A band Reference:
D. T tubule to T tubule Costanzo, L.S. (2018). BRS Physiology (7th ed.,
p.42). Lippincott Williams and Wilkins.

12. During muscle contraction, which of A A. A band


the following remains the same length?
During muscle contraction or
A. A band shortening, the A band remains the
B. I band same length.
C. Z lines
D. H band The I band and the H band both
shorten.

Reference:
Le, T., Bhushan, V., & Sochat, M. (2023). First
Aid for the USMLE Step 1 2022, Thirty Third
Edition (33rd ed., p. 459). McGraw-Hill
Education.

13. Which of the following glucose D D. GLUT 4


transporters is dependent on the
presence of insulin? GLUT 4 is found in adipose tissue and
striated muscles. It is known as an
A. GLUT 1 insulin-dependent glucose transporter.
B. GLUT 2
C. GLUT 3 GLUT 1,2, and 3 are insulin-
D. GLUT 4 independent glucose transporters.

Reference:
Le, T., Bhushan, V., & Sochat, M. (2023). First
Aid for the USMLE Step 1 2022, Thirty Third
Edition (33rd ed., p. 338). McGraw-Hill
Education.

14. The affinity of hemoglobin for A A. Increase in PCO2


oxygen is decreased in conditions that
shift the hemoglobin-oxygen Conditions that shift the hemoglobin-
dissociation curve to the right. oxygen dissociation curve to the RIGHT
(reduced Hgb affinity for oxygen):
Which of the following is included in ● Exercise
those conditions? ● Decreased pH
● Increase in PCO2
A. Increase in PCO2 ● Increased temperature/fever
B. Increase in pH ● Hypoxia
C. Decrease in temperature ● Increased 2,3 DPG

72
D. Decrease in 2,3 DPG
concentration Conditions that shift the hemoglobin-
oxygen dissociation curve to the LEFT
(increased Hgb affinity for oxygen):
● Increased pH
● Decreased temperature
● Presence of fetal Hgb (higher
O2 affinity)
● Decreased 2,3 DPG
● Increased carbon monoxide and
methemoglobin
References:
● Costanzo, L.S. (2018). BRS Physiology (7th
ed., pp. 240-241). Lippincott Williams and
Wilkins.
● Le, T., Bhushan, V., & Sochat, M. (2023).
First Aid for the USMLE Step 1 2022, Thirty
Third Edition (33rd ed., p. 689). McGraw-Hill
Education.

15. Majority of blood CO2 is transported A A. HCO3 −


to lungs in what form?
Majority of blood CO2 is transported to
A. HCO3−
lungs in the form of HCO3 −.
B. Carbaminohemoglobin
C. Dissolved CO2 References:
D. All of the above ● Costanzo, L.S. (2018). BRS Physiology (7th
ed., p. 247). Lippincott Williams and Wilkins.
● Le, T., Bhushan, V., & Sochat, M. (2023).
First Aid for the USMLE Step 1 2022, Thirty
Third Edition (33rd ed., p. 688). McGraw-Hill
Education.

16. Central chemoreceptors are C C. Decrease in pH


stimulated to increase breathing rate
by which of the following? Central chemoreceptors are
stimulated by decrease in pH of the
A. Increase in H+ CSF which then increases the
B. Increase in pH breathing rate.
C. Decrease in pH
D. Decrease in pO2 H+ does not cross the blood-brain
barrier.

Central chemoreceptors do not respond


directly to changes in PO2.

References:
● Costanzo, L.S. (2018). BRS Physiology (7th
ed., p. 256). Lippincott Williams and Wilkins.
● Le, T., Bhushan, V., & Sochat, M. (2023).
First Aid for the USMLE Step 1 2022, Thirty
Third Edition (33rd ed., p. 299). McGraw-Hill
Education.

17. What is the EEG waveform present B B. Beta

73
in an awake and alert patient with eyes
open? ● An awake, alert patient with eyes
open will show beta waves.
A. Alpha
B. Beta ● An awake, alert patient with eyes
C. Delta closed will show alpha waves.
D. Theta
● Theta waves are present in
non-REM sleep, stage N1.

● Delta waves are present in


non-REM sleep, stage N3.

Reference:
Le, T., Bhushan, V., & Sochat, M. (2023). First
Aid for the USMLE Step 1 2022, Thirty Third
Edition (33rd ed., p. 508). McGraw-Hill
Education.

18. Sleep spindles in the EEG are C C. Stage N2 of non-REM sleep


found in what stage of sleep?
Sleep spindles and K complexes in the
A. REM sleep EEG are found in stage N2 of non-REM
B. Stage N1 of non-REM sleep sleep.
C. Stage N2 of non-REM sleep
D. Stage N3 of non-REM sleep Reference:
Le, T., Bhushan, V., & Sochat, M. (2023). First
Aid for the USMLE Step 1 2022, Thirty Third
Edition (33rd ed., p. 508). McGraw-Hill
Education.

19. What immunoglobulin isotype is B B. IgA


found in bodily secretions such as
tears, saliva, and mucus? IgA is found in bodily secretions such
as tears, saliva, mucus and breast milk.
A. IgG
B. IgA IgG is the most abundant in serum. It is
C. IgM the only isotype that can cross the
D. IgE placenta.

IgM is the first to be produced in an


immune response.

IgE functions to bind mast cells and


basophils.

Reference:
Le, T., Bhushan, V., & Sochat, M. (2023). First
Aid for the USMLE Step 1 2022, Thirty Third
Edition (33rd ed., p. 103). McGraw-Hill
Education.

20. Which of the following ECG B B. PR interval


waveforms is influenced by the
conduction velocity through the AV The PR interval is influenced by the
node? conduction velocity through the AV
node.

74
A. P wave
B. PR interval For example, if the conduction velocity
C. QRS complex slows down such as in cases of heart
D. QT interval block, the PR interval becomes
prolonged.

Reference:
Costanzo, L.S. (2018). BRS Physiology (7th ed.,
p. 137). Lippincott Williams and Wilkins.

21. Which of the following represents D D. T wave


ventricular repolarization?
● Ventricular repolarization is
A. P wave represented by the T wave.
B. QRS complex
C. U wave ● P wave – atrial depolarization
D. T wave
● QRS complex – Ventricular
depolarization

Reference:
Costanzo, L.S. (2018). BRS Physiology (7th ed.,
p. 137). Lippincott Williams and Wilkins.

22. Beta 1 receptor stimulation results B B. Phospholamban


in increased cardiac contractility
through the phosphorylation of ____ : Phospholamban phosphorylation →
A. Calmodulin increased activity of the Ca2+ ATPase →
B. Phospholamban increased Ca2+ stored in the
C. Tropomyosin
D. All of the above sarcoplasmic reticulum → more calcium
available for next cardiac beats.

References:
● Costanzo, L.S. (2018). BRS Physiology (7th
ed., p. 148). Lippincott Williams and Wilkins.
● Le, T., Bhushan, V., & Sochat, M. (2023).
First Aid for the USMLE Step 1 2022, Thirty
Third Edition (33rd ed., p. 289). McGraw-Hill
Education.

23. Myocardial oxygen demand is B B. Reduced afterload


increased by all of the following,
EXCEPT? Factors that increase myocardial
oxygen demand are the following:
A. Increased cardiac contractility ● Increased cardiac contractility
B. Reduced afterload ● Increased afterload
C. Increased HR ● Increased HR
D. Increased ventricular diameter ● Increased ventricular diameter
from increased wall tension
Reference:
Le, T., Bhushan, V., & Sochat, M. (2023). First
Aid for the USMLE Step 1 2022, Thirty Third
Edition (33rd ed., p. 289). McGraw-Hill
Education.

75
24. Prolactin secretion is tonically A A. Dopamine
inhibited by which of the following?
Dopamine tonically inhibits prolactin
A. Dopamine secretion from the anterior pituitary.
B. Dobutamine
C. Somatostatin This is the reason why dopamine
D. ACTH antagonists (which removes the tonic
inhibition of prolactin), such as
antipsychotics, can cause
hyperprolactinemia leading to
galactorrhea.

References:
● Costanzo, L.S. (2018). BRS Physiology (7th
ed., p. 440). Lippincott Williams and Wilkins.
● Le, T., Bhushan, V., & Sochat, M. (2023).
First Aid for the USMLE Step 1 2022, Thirty
Third Edition (33rd ed., p. 334). McGraw-Hill
Education.

25. The following anterior pituitary D D. ACTH


hormones have the same alpha
subunits, EXCEPT? TSH, FSH, and LH have the same
alpha subunits because they belong in
A. TSH the same glycoprotein family, but they
B. FSH have different beta subunits which
C. LH accounts for the unique biologic activity
D. ACTH of each of them.

ACTH is derived from


proopiomelanocortin or POMC.

Reference:
Costanzo, L.S. (2018). BRS Physiology (7th ed.,
p. 438). Lippincott Williams and Wilkins.

26. Lesions of which of the following B B. Substantia nigra


parts of the basal ganglia may result in
Parkinson disease? Lesions of the substantia nigra may
result in Parkinson disease.
A. Striatum
B. Substantia nigra Lesions of the striatum may occur in
C. Subthalamic nucleus Huntington disease.
D. Any of the above
Lesions of the subthalamic nucleus
may give rise to hemiballismus.

Reference:
Costanzo, L.S. (2018). BRS Physiology (7th ed.,
p. 109). Lippincott Williams and Wilkins.

27. Damage to what area may result in A A. Broca area


the inability to write, but with intact
understanding? Damage to the Broca area, such as
from an MCA stroke, may result in
A. Broca area motor aphasia, where understanding is

76
B. Wernicke area intact, but speech and writing is
C. Occipital area impaired.
D. Hippocampus
Damage to the Wernicke area, such as
from an MCA stroke, may result in
sensory aphasia, where language
comprehension is impaired.

References:
● Costanzo, L.S. (2018). BRS Physiology (7th
ed., p. 111). Lippincott Williams and Wilkins.
● Le, T., Bhushan, V., & Sochat, M. (2023).
First Aid for the USMLE Step 1 2022, Thirty
Third Edition (33rd ed., p. 531). McGraw-Hill
Education.

28. Calcitonin is produced by ___: B B. Parafollicular cells

A. Follicular cells Parafollicular cells (C cells) of the


B. Parafollicular cells thyroid produce calcitonin.
C. PTH
D. All of the above Reference:
Costanzo, L.S. (2018). BRS Physiology (7th ed.,
p. 477). Lippincott Williams and Wilkins.

29. Aldosterone is produced in which of A A. Zona glomerulosa


the following structures?
Aldosterone – zona glomerulosa
A. Zona glomerulosa
B. Zona fasciculata Cortisol – zona fasciculata
C. Zona reticularis
D. Adrenal medulla Androgens – zona reticularis

Reference:
Costanzo, L.S. (2018). BRS Physiology (7th ed.,
p. 453). Lippincott Williams and Wilkins.

30. Cortisol has all the following D D. None of the above


effects, EXCEPT?
Cortisol has all the following effects to
A. Increased protein catabolism stimulate gluconeogenesis:
B. Decreased glucose utilization ● Increased protein catabolism
C. Increased lipolysis ● Decreased glucose utilization
D. None of the above ● Increased lipolysis

Reference:
Costanzo, L.S. (2018). BRS Physiology (7th ed.,
p. 455). Lippincott Williams and Wilkins

31. Which of the following muscles A A. Diaphragm


is/are used during quiet inspiration?
The external intercostals and
A. Diaphragm accessory muscles are not used
B. External intercostals during quiet inspiration.
C. Accessory muscles

77
D. All of the above They are only used during exercise or
respiratory distress.

Reference:
Costanzo, L.S. (2018). BRS Physiology (7th ed.,
p. 220) Lippincott Williams and Wilkins

32. The following are characteristics of C C. High sodium


the saliva, EXCEPT?
The characteristics of the saliva are the
A. High volume following:
B. High potassium ● High volume
C. High sodium ● High potassium and bicarbonate
D. Hypotonic ● Low sodium and chloride
● Hypotonic
● Has amylase, lingual lipase and
kallikrein

Reference:
Costanzo, L.S. (2018). BRS Physiology (7th ed.,
p. 387) Lippincott Williams and Wilkins.

33. The vomiting center in the medulla C C. 4th ventricle


receives information from the
chemoreceptor trigger zone (CTZ) Activation of the CTZ found in the area
found in the ____ : postrema located in the 4th ventricle
results to vomiting.
A. Lateral ventricles
B. 3rd ventricle Reference:
C. 4th ventricle Costanzo, L.S. (2018). BRS Physiology (7th ed.,
p. 385) Lippincott Williams and Wilkins.
D. Cerebral cortex

34. Vitamin A is stored in the liver in B B. Hepatic stellate cells


what type of cells?
Hepatic stellate cells or Ito cells store
A. Kupffer cells Vitamin A in the liver.
B. Hepatic stellate cells
C. Sinusoidal cells Kupffer cells function by clearing
D. Hepatocytes bacteria and damaged or old RBCs.
They reside in the sinusoids.

Reference:
Le, T., Bhushan, V., & Sochat, M. (2023). First
Aid for the USMLE Step 1 2022, Thirty Third
Edition (33rd ed., p. 374). McGraw-Hill
Education.

35. Antidiuretic hormone (ADH) is B B. Supraoptic nuclei


synthesized in the ____:
ADH is sythesized in the supraoptic
A. Posterior pituitary nuclei of the hypothalamus and are
B. Supraoptic nuclei stored and released in the posterior
C. Anterior pituitary pituitary.
D. All of the above
References:

78
● Costanzo, L.S. (2018). BRS Physiology (7th
ed., p. 444). Lippincott Williams and Wilkins.
● Le, T., Bhushan, V., & Sochat, M. (2023).
First Aid for the USMLE Step 1 2022, Thirty
Third Edition (33rd ed., p. 509). McGraw-Hill
Education.

36. ADH increases water permeability A A. cAMP


of the late distal tubule and collecting
ducts using what signaling pathway? ADH increases water permeability of
the late distal tubule and collecting
A. cAMP ducts via a V2 receptor using the cAMP
B. cGMP signaling pathway.
C. IP3
D. Tyrosine kinase The IP3 pathway is used via a V1
receptor for vasoconstriction by ADH.

References:
● Costanzo, L.S. (2018). BRS Physiology (7th
ed., p. 444). Lippincott Williams and Wilkins.
● Le, T., Bhushan, V., & Sochat, M. (2023).
First Aid for the USMLE Step 1 2022, Thirty
Third Edition (33rd ed., p. 341). McGraw-Hill
Education.

37. When blood in the lungs becomes A A. Haldane effect


oxygenated, hydrogen dissociates from
hemoglobin to form CO2 which is then When blood in the lungs becomes
exhaled. This process is called the oxygenated, hydrogen dissociates from
____ : hemoglobin to form CO2 which is then
exhaled. This process is called the
A. Haldane effect Haldane effect.
B. Bohr effect
C. Cushing effect The Bohr effect occurs at the cellular
D. Left shift
level where there’s increased hydrogen
and CO2 (from tissue metabolism) →
increased unloading of oxygen at the
tissues.

References:
● Costanzo, L.S. (2018). BRS Physiology (7th
ed., p. 689). Lippincott Williams and Wilkins.
● Le, T., Bhushan, V., & Sochat, M. (2023).
First Aid for the USMLE Step 1 2022, Thirty
Third Edition (33rd ed., p. 709). McGraw-Hill
Education.

38. Examples of primary active D D. Na+ - Ca2+ exchange


transport are the following, EXCEPT?
Na+ - Ca2+ exchange is an example of
+ +
A. Na K ATPase secondary active transport where
B. Ca2+ ATPase energy is indirectly provided from the
C. H+ K+ ATPase Na+ gradient that is present across cell
D. Na+ - Ca2+ exchange membranes.

79
The rest are examples of primary
active transport which require direct
input of ATP.

Reference:
Costanzo, L.S. (2018). BRS Physiology (7th ed.,
p. 18) Lippincott Williams and Wilkins.

39. The Na+ – glucose cotransport B B. Secondary active transport


present in the proximal tubules of the
kidney is an example of ___ : In this example, Na+ is transported
“downhill” while glucose is transported
A. Primary active transport “uphill.”
B. Secondary active transport
C. Osmosis Therefore, it is an example of
D. Diffusion secondary active transport.
Reference:
Costanzo, L.S. (2018). BRS Physiology (7th ed.,
p. 18) Lippincott Williams and Wilkins.

40. Which of the following glial cells A A. Oligodendrocytes


synthesize myelin in the CNS?
Oligodendrocytes synthesize myelin in
A. Oligodendrocytes the CNS. Schwann cells produce
B. Schwann cells myelin in the PNS.
C. Astrocytes
D. Microglia Astrocytes provide:
● Physical support
● Repair functions
● Removal of excess
neurotransmitters

References:
● Costanzo, L.S. (2018). BRS Physiology (7th
ed., p. 79). Lippincott Williams and Wilkins.
● Le, T., Bhushan, V., & Sochat, M. (2023).
First Aid for the USMLE Step 1 2022, Thirty
Third Edition (33rd ed., p. 503). McGraw-Hill
Education.

41. Which of the following vessels B B. Arterioles


provide the highest resistance in the
cardiovascular system? Arterioles provide the highest
resistance in the cardiovascular
A. Arteries system.
B. Arterioles
C. Capillaries Reference:
D. Venules Costanzo, L.S. (2018). BRS Physiology (7th ed.,
p. 130) Lippincott Williams and Wilkins.

42. Which of the following has the D D. Veins


highest proportion of blood in the
cardiovascular system? Arterioles provide the highest
resistance in the cardiovascular
A. Arteries system.
B. Capillaries

80
C. Venules Capillaries comprise the largest total
D. Veins cross-sectional area.

Veins contain the highest proportion of


blood in the CV system.

Reference:
Costanzo, L.S. (2018). BRS Physiology (7th ed.,
p. 130) Lippincott Williams and Wilkins.

43. Na+-K+-2Cl+ transport in the thick A A. Cotransport


ascending limb of the loop of Henle is
an example of ___: Na+-K+-2Cl+ transport in the thick
ascending limb of the loop of Henle is
A. Cotransport an example of cotransport (secondary
B. Countertransport active transport).
C. Antiport
D. Primary active transport Na+ - Ca2+ exchange is an example of
countertransport or antiport.

Reference:
Costanzo, L.S. (2018). BRS Physiology (7th ed.,
p. 18) Lippincott Williams and Wilkins.

44. Efflux of K+ occurs in which phase D D. Phase D


of the SA node action potential?
SA node action potential:
A. Phase 0
B. Phase 1 Phase 0:
C. Phase 2 ● Upstroke – opening of voltage-
D. Phase 3 gated Ca2+ channels
Phase 3:
● Repolarization
● Inactivation of Ca2+ channels
● Increased K+ efflux
Phase 4:
● Slow depolarization
● Caused by inward Na+ current
called If (funny current)
● Accounts for automaticity of the
SA and AV node

There is no Phase 1 and Phase 2 in


pacemaker action potential.

References:
● Costanzo, L.S. (2018). BRS Physiology (7th
ed., p. 139). Lippincott Williams and Wilkins.
● Le, T., Bhushan, V., & Sochat, M. (2023).
First Aid for the USMLE Step 1 2022, Thirty
Third Edition (33rd ed., p. 297). McGraw-Hill
Education.

45. Which of the following phases of D D. Phase 4

81
the pacemaker action potential
accounts for SA node automaticity? SA node action potential:

A. Phase 0 Phase 0:
B. Phase 2 ● Upstroke – opening of voltage-
C. Phase 3 gated Ca2+ channels
D. Phase 4
Phase 3:
● Repolarization
● Inactivation of Ca2+ channels
● Increased K+ efflux
Phase 4:
● Slow depolarization
● Caused by inward Na+ current
called If (funny current)
● Accounts for automaticity of the
SA and AV node

There is no Phase 1 and Phase 2 in


pacemaker action potential.

References:
● Costanzo, L.S. (2018). BRS Physiology (7th
ed., p. 139). Lippincott Williams and Wilkins.
● Le, T., Bhushan, V., & Sochat, M. (2023).
First Aid for the USMLE Step 1 2022, Thirty
Third Edition (33rd ed., p. 297). McGraw-Hill
Education.

46. Which of the following controls C C. Submucosal plexus


secretion and blood flow in the GI
tract? Myenteric plexus controls GI smooth
muscle motility.
A. Vagus nerve
B. Myenteric plexus Submucosal plexus control secretion
C. Submucosal plexus and blood flow in the GI tract.
D. All of the above
Reference:
Costanzo, L.S. (2018). BRS Physiology (7th ed.,
p. 372) Lippincott Williams and Wilkins.

47. The following can increase stroke A A. Increased afterload


volume, EXCEPT?
Increased afterload results in a
A. Increased afterload decrease in stroke volume.
B. Increased venous return
C. Increased preload The rest of the choices result to an
D. None of the above increase in stroke volume.

Reference:
Costanzo, L.S. (2018). BRS Physiology (7th ed.,
p. 152) Lippincott Williams and Wilkins.

48. Accesory sex organs such as the A A. 5 alpha-reductase


prostate convert testosterone to

82
dihydrotestosterone using what Accesory sex organs such as the
enzyme? prostate convert testosterone to
dihydrotestosterone using 5 alpha-
A. 5 alpha-reductase reductase.
B. Tyrosinase
C. Phosphodiesterase Reference:
Costanzo, L.S. (2018). BRS Physiology (7th ed.,
D. None of the above p. 480) Lippincott Williams and Wilkins.

49. Testosterone is produced by what A A. Leydig cells


cell?
Leydig cells secrete testosterone.
A. Leydig cells
B. Sertoli cells Reference:
C. Granulosa cells Costanzo, L.S. (2018). BRS Physiology (7th ed.,
p. 480) Lippincott Williams and Wilkins.
D. Spermatogonia

50. Humoral hypercalcemia of A A. PTH-related peptide


malignancy is caused by ___ :
PTH-rp released by certain tumors,
A. PTH-related peptide such breast cancers and squamous cell
B. Excess vitamin D carcinoma of the lung, increase bone
C. Calcitonin resorption leading to hypercalcemia.
D. Any of the above
Reference:
Costanzo, L.S. (2018). BRS Physiology (7th ed.,
p. 473) Lippincott Williams and Wilkins.

51. Cholecystokinin (CCK) is secreted B B. I cells


by which of the following cells?
S cells secrete secretin and acts to
A. S cells increase HCO3- secretion.
B. I cells
C. C cells CCK is secreted by I cells and
D. K cells increases secretion of amylase, lipases
and proteases.

Reference:
Costanzo, L.S. (2018). BRS Physiology (7th ed.,
p. 399) Lippincott Williams and Wilkins.

52. Which of the following substances D D. Vitamin B12


is absorbed in the ileum and requires
intrinsic factor? Vitamin B12 is absorbed in the ileum
and requires intrinsic factor.
A. Iron
B. Lipids Reference:
C. Calcium Costanzo, L.S. (2018). BRS Physiology (7th ed.,
p. 412) Lippincott Williams and Wilkins.
D. Vitamin B12

53. The following are transported from C C. Fructose


the intestinal lumen into cells by using
Na+-dependent cotransport, EXCEPT? Fructose is transported through
facilitated diffusion (GLUT5).
A. Glucose
B. Galactose Glucose and galactose are

83
C. Fructose transported using Na+-dependent
D. None of the above cotransport (SGLT1).

Reference:
Costanzo, L.S. (2018). BRS Physiology (7th ed.,
p. 405) Lippincott Williams and Wilkins.

54. What is the main regulator of A A. Glucose.


insulin release?
The main regulator of insulin release is
A. Glucose glucose.
B. Stomach acid
C. Amino acids Reference:
D. Vagus nerve Costanzo, L.S. (2018). BRS Physiology (7th ed.,
p. 338) Lippincott Williams and Wilkins.

55. The following hormones are D D. GHRH


derived from proopiomelanocortin
(POMC), EXCEPT? ACTH, MSH, Beta-lipotropin, and Beta
endorphin are derived from POMC.
A. ACTH
B. MSH Reference:
C. Beta-lipotropin Costanzo, L.S. (2018). BRS Physiology (7th ed.,
p. 438) Lippincott Williams and Wilkins.
D. GHRH

56. Growth hormone secretion is D D. Hypoglycemia


increased by which of the following?
Secretion of GH is increased by:
A. Somatostatin ● Sleep
B. Somatomedin ● Stress
C. Pregnancy ● Puberty hormones
D. Hypoglycemia ● Starvation
● Hypoglycemia
● Exercise

It is decreased by:
● Somatostatin
● Somatomedin
● Obesity
● Hyperglycemia
● Pregnancy
Reference:
Costanzo, L.S. (2018). BRS Physiology (7th ed.,
p. 438) Lippincott Williams and Wilkins.

57. Increased linear growth which A A. Insulin-like growth factor


happens during a pubertal growth spurt
is due to ___ : Increased linear growth which happens
during a pubertal growth spurt is due to
A. Insulin-like growth factor actions of growth hormone through
B. Growth hormone IGF-1, not a direct action of growth
C. ACTH hormone.
D. FSH
This makes B wrong.

84
Direct actions of GH include:
● Decreased glucose uptake
● Increased lipolysis
● Increased protein synthesis
● Increased production of IGF

Reference:
Costanzo, L.S. (2018). BRS Physiology (7th ed.,
p. 439) Lippincott Williams and Wilkins.

58. Which of the following lung A A. Vital capacity


capacities is the sum of tidal volume,
IRV, and ERV? ● VC is the sum of tidal volume (TV),
IRV and ERV.
A. Vital capacity
B. Functional residual capacity ● FRC is the volume remaining in the
C. Inspiratory capacity lungs after a TV is expired.
D. Total lung capacity
● IC is the sum of TV and IRV.

● TLC is the sum of all 4 lung


volumes, namely: IRV, TV, ERV,
and RV.

Reference:
Costanzo, L.S. (2018). BRS Physiology (7th ed.,
p. 218) Lippincott Williams and Wilkins.

59. Lung surfactant is made up B B. Dipalmitoylphosphatidylcholine


primarily of ___:
Lung surfactant is made up primarily of
A. Water dipalmitoylphosphatidylcholine (DPPC)
B. Dipalmitoylphosphatidylcholine by type II alveolar cells.
C. Bisphosphoglycerate
D. None of the above Reference:
Costanzo, L.S. (2018). BRS Physiology (7th ed.,
p. 224) Lippincott Williams and Wilkins.

60. In most organs in the body, hypoxia B B. Lungs


causes vasodilation. In which organ
does hypoxia cause vasoconstriction? Hypoxia causes vasoconstriction of the
pulmonary vessels.
A. Heart
B. Lungs In most organs in the body, hypoxia
C. Kidneys causes vasodilation.
D. Liver
Reference:
Le, T., Bhushan, V., & Sochat, M. (2023). First
Aid for the USMLE Step 1 2022, Thirty Third
Edition (33rd ed., p. 300). McGraw-Hill
Education.

61. Which of the following dilates the A A. Increased prostaglandins


afferent arteriole?
Prostaglandins dilate the afferent
A. Increased prostaglandins arteriole while angiotensin II constricts
B. Decreased prostaglandins the efferent arteriole.

85
C. Increased angiotensin II
D. Decreased angiotensin II Reference:
Costanzo, L.S. (2018). BRS Physiology (7th ed.,
p. 603) Lippincott Williams and Wilkins.

62. The following anterior pituitary D D. GH


hormones are produced by basophils,
EXCEPT? ● GH and prolactin are secreted by
acidophils.
A. FSH
B. ACTH ● FSH, LH, TSH, ACTH are secreted
C. TSH by basophils.
D. GH
Reference:
Costanzo, L.S. (2018). BRS Physiology (7th ed.,
p. 331) Lippincott Williams and Wilkins.

63. Renal blood flow is how many B B. 25%


percent of the cardiac output?
Renal blood flow is 25% of the cardiac
A. 20% output.
B. 25%
C. 30% Reference:
D. 50% Costanzo, L.S. (2018). BRS Physiology (7th ed.,
p. 285) Lippincott Williams and Wilkins.

64. Volume of the conducting airways A A. Anatomic dead space


is called ___:
The volume of the conducting airways
A. Anatomic dead space is called anatomic dead space.
B. Residual volume
C. Functional residual capacity RV is air in the lungs after maximal
D. None of the above expiration.

FRC is air in the lungs after expiration


of tidal volume.

Reference:
Costanzo, L.S. (2018). BRS Physiology (7th ed.,
pp. 217-218) Lippincott Williams and Wilkins.

65. Excretion of glucose in the urine B B. 250


happens when the plasma glucose
reaches what level (in mg/dL)? Excretion of glucose in the urine
happens when the plasma glucose
A. 180 reaches 250 mg/dL. This is also the
B. 250 called threshold.
C. 500
D. 1000 Reference:
Costanzo, L.S. (2018). BRS Physiology (7th ed.,
p. 295) Lippincott Williams and Wilkins.

66. Which of the following cells in the A A. Principal cells


late distal tubule secretes K+?
Principal cells secrete K+ and

86
A. Principal cells reabsorb Na+.
B. Alpha intercalated cells
C. Beta intercalated cells Alpha-intercalated cells rebasorb K+.
D. All of the above
Reference:
Costanzo, L.S. (2018). BRS Physiology (7th ed.,
pp. 302-303) Lippincott Williams and Wilkins

67. Components of the Cushing reflex C C


include the following, EXCEPT?
The Cushing reflex occurs in patients
A. Hypertension with increased intracranial pressure.
B. Bradycardia
C. Tachycardia Components of the Cushing reflex
D. Respiratory depression include:
● Hypertension
● Bradycardia
● Respiratory depression
Reference:
Le, T., Bhushan, V., & Sochat, M. (2023). First
Aid for the USMLE Step 1 2022, Thirty Third
Edition (33rd ed., p. 299). McGraw-Hill.

68. Which of the following will create A A. Reflection coefficient of 1


an osmotic pressure that will drive
water flow? Reflection coefficient refers to the
ease with which a solute passes
A. Reflection coefficient of 1 through a membrane.
B. Urea
C. Reflection coefficient of 0 If the reflection coefficient is 1 (such
D. Incomplete data as albumin), the solute does not pass
through the membrane. This creates an
osmotic pressure that drives water flow.

If the reflection coefficient is 0 (such


as urea), the solute can pass through
the membrane and therefore does not
create osmotic pressure and does not
cause water flow.

Reference:
Costanzo, L.S. (2018). BRS Physiology (7th ed.,
p. 22) Lippincott Williams and Wilkins.

69. Sphincter muscle contraction in the A A. α1


bladder is a result of of stimulation of
the which of the following? Sphincter muscle contraction in the
bladder is a result of of stimulation of
A. α1 α1 adrenergic receptors.
B. α2
C. β1 ● Stimulation of α2 adrenergic
D. β2
receptors → decreased

sympathetic outflow

87
● Stimulation of β1 adrenergic
receptors → increased HR and
cardiac contractility
● Stimulation of β2 adrenergic
receptors → vasodilation,

bronchodilation

Reference:
Le, T., Bhushan, V., & Sochat, M. (2023). First
Aid for the USMLE Step 1 2022, Thirty Third
Edition (33rd ed., p. 237). McGraw-Hill.

70. Nonselective beta blockers can D D. β2


exacerbate reactive airway diseases
like asthma due to inhibition of which ● Stimulation of β1 adrenergic
receptors?
receptors → increased HR and
A. α1 cardiac contractility
B. α2
C. β1 ● Stimulation of β2 adrenergic
D. β2 receptors → vasodilation,

bronchodilation
● Inhibition of β2 adrenergic
receptors by using beta blockers will
result in bronchoconstriction →
exacerbation of asthma.

Reference:
Le, T., Bhushan, V., & Sochat, M. (2023). First
Aid for the USMLE Step 1 2022, Thirty Third
Edition (33rd ed., p. 237). McGraw-Hill.

71. The following are characteristics of D D. None of the above


the parasympathetic nervous system,
EXCEPT? Characteristics of the parasympathetic
nervous system:
A. Long preganglionic nerve axon ● Long preganglionic nerve axon
B. Has nicotinic receptors in the ● Has nicotinic receptors in the
ganglion ganglion
C. Has muscarinic receptors in ● Short postganglionic nerve axon
effector organs ● Has muscarinic receptors in
D. None of the above effector organs

Characteristics of the sympathetic


nervous system:
● Short preganglionic nerve axon
● Has nicotinic receptors in the
ganglion
● Long postganglionic nerve axon

88
● Has alpha and beta receptors in
effector organs
Reference:
Le, T., Bhushan, V., & Sochat, M. (2023). First
Aid for the USMLE Step 1 2022, Thirty Third
Edition (33rd ed., p. 73). McGraw-Hill.

72. The origin of preganglionic nerves D D. Spinal cord segment T1-T12


of the sympathetic nervous system is in
the ____ : The origin of preganglionic nerves of
the sympathetic nervous system is in
A. Pelvic splanchnic nerves spinal cord segments T1-T12 and L1-
B. Cranial nerve III, VII, IX and X L3.
C. S2-S4 spinal cord segment
D. Spinal cord segment T1-T12 The origin of preganglionic nerves of
the parasympathetic nervous system
is in cranial nerves III, VII, IX and X and
S2-S4 spinal cord segment.

Reference:
Costanzo, L.S. (2018). BRS Physiology (7th ed.,
p. 73) Lippincott Williams and Wilkins.

73. Which of the following is TRUE of B B. Innervated by preganglionic


the adrenal medulla? sympathetic fibers

A. Innervated by the The adrenal medulla is innervated by


parasympathetic nervous preganglionic sympathetic fibers.
system
B. Innervated by preganglionic Reference:
sympathetic fibers Costanzo, L.S. (2018). BRS Physiology (7th ed.,
p. 74) Lippincott Williams and Wilkins.
C. Innervated by postganglionic
sympathetic fibers
D. None of the above

74. Muscarinic antagonists in the C C. M3


bladder promote relaxation of the
detrusor muscle. Muscarinic antagonists in the bladder
promote relaxation of the detrusor
They block which of the following muscle by blocking M3 cholinergic
receptors? receptors.

A. M1 M1 receptors mediate higher cognitive


B. M2 functions.
C. M3
D. M4 M2 receptors are for decreasing HR
and contractility of the heart.

Reference:
Le, T., Bhushan, V., & Sochat, M. (2023). First
Aid for the USMLE Step 1 2022, Thirty Third
Edition (33rd ed., p. 237). McGraw-Hill.

75. Which of the following is absorbed D D. Bile acids

89
in the ileum?
The site of absorption of bile acids and
A. Carbohydrates Vitamin B12 is in the ileum.
B. Fat-soluble vitamins
C. Iron Reference:
Costanzo, L.S. (2018). BRS Physiology (7th ed.,
D. Bile acids
p. 404) Lippincott Williams and Wilkins.

76. Which of the following transports A A. Transferrin


iron in the blood?
Iron is transported in the blood bound to
A. Transferrin transferrin.
B. Hepcidin
C. Apoferritin Ferritin is the iron storage protein of
D. Ferritin the body.

Hepcidin is produced in the liver in the


presence of inflammation. It decreases
the release of iron from macrophages
and its absorption from the GIT.

References:
● Costanzo, L.S. (2018). BRS Physiology (7th
ed., p. 412). Lippincott Williams and Wilkins.
● Le, T., Bhushan, V., & Sochat, M. (2023).
First Aid for the USMLE Step 1 2022, Thirty
Third Edition (33rd ed., p. 427). McGraw-Hill
Education.

77. What is the active form of Vitamin A A. 1,25 dihydroxycholecalciferol


D?
1,25 dihydroxycholecalciferol is the
A. 1,25 dihydroxycholecalciferol active form of Vitamin D.
B. Ergocalciferol
C. Cholecalciferol References:
D. 7- dehydrocholesterol ● Costanzo, L.S. (2018). BRS Physiology (7th
ed., p. 475). Lippincott Williams and Wilkins.
● Le, T., Bhushan, V., & Sochat, M. (2023).
First Aid for the USMLE Step 1 2022, Thirty
Third Edition (33rd ed., p. 68). McGraw-Hill
Education.

78. Which of the following B B. Dorsal column system


somatosensory pathways is
responsible for sensations of fine touch The dorsal column receives the
and vibration? following sensory information:
● Fine touch
A. Anterior system ● Vibration
B. Dorsal column system ● Pressure
C. Lateral system ● 2-point discrimination
D. All of the above ● Proprioception

The anterolateral system receives


information on:
● Temperature
● Pain

90
● Light touch

Reference:
Costanzo, L.S. (2018). BRS Physiology (7th ed.,
p. 83) Lippincott Williams and Wilkins.

79. All of the following increases the A A. Decreased serum osmolality


secretion of ADH, EXCEPT?
Increased serum osmolality increases
A. Decreased serum osmolality ADH secretion, not decreased
B. Nicotine osmolality.
C. Pain
D. Nausea Other factors that increase ADH
secretion are:
● Volume contraction
● Pain
● Nausea
● Hypoglycemia
● Nicotine
● Opiates
● Antineoplastic drugs

Reference:
Costanzo, L.S. (2018). BRS Physiology (7th ed.,
p. 446) Lippincott Williams and Wilkins.

80. Which of the following is NOT D D. None of the above


derived from tyrosine?
Thyroxine, dopamine, and melanin
A. Thyroxine are all derived from tyrosine.
B. Dopamine
C. Melanin Reference:
D. None of the above Le, T., Bhushan, V., & Sochat, M. (2023). First
Aid for the USMLE Step 1 2022, Thirty Third
Edition (33rd ed., p. 82). McGraw-Hill Education.

81. The limbic system, which is D D. None of the above


involved in processing emotions and
long-term memory, is composed of the All are part of the limbic system.
following structures, EXCEPT?
The limbic system, which is involved
A. Hippocampus in processing emotions and long-term
B. Amygdala memory, is composed of the following
C. Mammillary bodies structures:
D. None of the above ● Hippocampus
● Amygdala
● Mammillary bodies
● Anterior thalamic nuclei
● Cingulate gyrus
● Entorhinal cortex

Reference:
Le, T., Bhushan, V., & Sochat, M. (2023). First
Aid for the USMLE Step 1 2022, Thirty Third
Edition (33rd ed., p. 510). McGraw-Hill
Education.

91
82. Which of the following C C. Nigrostriatal
dopaminergic pathways is responsible
for motor control and is involved in Dopaminergic pathways:
Parkinson disease?
● Mesocortical and mesolimbic:
A. Mesocortical motivation and reward
B. Mesolimbic ● Nigrostriatal: motor control
C. Nigrostriatal ● Tuberoinfundibular: regulates
D. Tuberoinfundibular prolactin secretion

Reference:
Le, T., Bhushan, V., & Sochat, M. (2023). First
Aid for the USMLE Step 1 2022, Thirty Third
Edition (33rd ed., p. 510). McGraw-Hill
Education.

83. Which of the following B B. PGE2


prostaglandins is responsible for
increasing the set-point temperature in PGE2 is responsible for increasing the
the hypothalamus to produce a fever? set-point temperature in the
hypothalamus to produce a fever.
A. PGD2
B. PGE2 Reference:
C. PGI2 Costanzo, L.S. (2018). BRS Physiology (7th ed.,
p. 114) Lippincott Williams and Wilkins.
D. None of the above

84. The following prostaglandins are C C. Thromboxane


vasodilators, EXCEP?
Thromboxane A2 is a vasoconstrictor.
A. Prostacyclin Reference:
B. E-series prostaglandins
Costanzo, L.S. (2018). BRS Physiology (7th ed.,
C. Thromboxane p. 180) Lippincott Williams and Wilkins.
D. None of the above

85. An increased PCO2 in the cerebral B B. Vasodilation


circulation causes _____:
An increased PCO2 in the cerebral
A. Vasoconstriction circulation causes vasodilation.
B. Vasodilation
C. No change in vessel diameter Reference:
D. None of the above Costanzo, L.S. (2018). BRS Physiology (7th ed.,
p. 180) Lippincott Williams and Wilkins.

86. What is the primary ion secreted C C. Chloride


into the intestinal lumen?
The primary ion secreted in the
A. Potassium intestinal lumen is chloride.
B. Hydrogen
C. Chloride Reference:
D. All of the above Costanzo, L.S. (2018). BRS Physiology (7th ed.,
p. 411) Lippincott Williams and Wilkins.

87. Calcium is absorbed in the intestine C C. 1,25 dihydroxycholecalciferol


in the presence of ____:
1,25 dihydroxycholecalciferol is the

92
A. PTH active form of Vitamin D and is
B. PTH-related peptide necessary in calcium absorption in the
C. 1,25 dihydroxycholecalciferol small intestine.
D. 24,25 dihydroxycholecalciferol
Reference:
Costanzo, L.S. (2018). BRS Physiology (7th ed.,
p. 412) Lippincott Williams and Wilkins.

88. Proton pump inhibitors such as A A. Primary active transport


omeprazole inhibit a transport pump
that is classified as an example of Proton pump (H+ K+ ATPase)
____: inhibitors such as omeprazole inhibit a
transport pump that is classified as an
A. Primary active transport example of primary active transport.
B. Secondary active transport
C. Osmosis Reference:
D. Diffusion Costanzo, L.S. (2018). BRS Physiology (7th ed.,
p. 17) Lippincott Williams and Wilkins.

89. Which of the following is secreted A A. Gastrin


by G cells that increase gastric acid
secretion? Gastrin is secreted by G cells in the
stomach antrum and increases gastric
A. Gastrin acid secretion.
B. Somatostatin
C. CCK Somatostatin is secreted by D cells
D. Secretin and decreases gastric acid and
pepsinogen secretion.

CCK is secreted by I cells and


increases bile and pancreatic
bicarbonate secretion.

Secretin is secreted by S cells and also


increases bile and pancreatic
bicarbonate secretion.

Reference:
Le, T., Bhushan, V., & Sochat, M. (2023). First
Aid for the USMLE Step 1 2022, Thirty Third
Edition (33rd ed., p. 378). McGraw-Hill
Education.

90. What hypothalamic nucleus is A A. Ventromedial nucleus


stimulated by ghrelin?
Lateral nucleus is stimulated by
A. Lateral nucleus ghrelin which is responsible for the
B. Ventromedial nucleus feeling of hunger.
C. Anterior nucleus
D. Supraoptic nucleus Ventromedial nucleus is stimulated by
leptin responsible for satiety.

Anterior nucleus is involved in heat


loss mechanisms during a fever.

Supraoptic and paraventricular

93
nuclei produce ADH and oxytocin.

Reference:
Le, T., Bhushan, V., & Sochat, M. (2023). First
Aid for the USMLE Step 1 2022, Thirty Third
Edition (33rd ed., p. 509). McGraw-Hill
Education.

91. Patients with thyroid storm are A A. Wolff Chaikoff effect


given potassium iodide to decrease the
release of thyroid hormones. What is Patients with thyroid storm are given
this effect called? potassium iodide to decrease the
synthesis of thyroid hormones. This is
A. Wolff Chaikoff effect called the Wolff Chaikoff effect.
B. Jod Basedow phenomenon
C. Graves disease Jod Basedow phenomenon occurs in
D. None of the above patients who are iodine-deficient and
are given iodine. This results in an
iodine-induced hyperthyroidism.
Reference:
Le, T., Bhushan, V., & Sochat, M. (2023). First
Aid for the USMLE Step 1 2022, Thirty Third
Edition (33rd ed., p. 346). McGraw-Hill
Education.

92. What is the primary regulator of A A. Ionized calcium


PTH release?
Ionized calcium is the primary
A. Ionized calcium regulator of PTH.
B. Phosphate levels
C. Vagus nerve Reference:
Le, T., Bhushan, V., & Sochat, M. (2023). First
D. TSH Aid for the USMLE Step 1 2022, Thirty Third
Edition (33rd ed., p. 337). McGraw-Hill
Education.

93. Which of the following is a multiunit D D. Ciliary muscle of the lens


smooth muscle?

A. Uterus Examples of multiunit smooth


B. Ureter muscles:
C. Bladder ● Iris
D. Ciliary muscle of the lens ● Ciliary muscle of the lens
● Vas deferens

Examples of unitary smooth


muscles:
● Uterus
● GI tract
● Ureter
● Bladder

Reference:
Costanzo, L.S. (2018). BRS Physiology (7th ed.,
p. 49) Lippincott Williams and Wilkins.

94. Exchange of nutrients, water and B B. Capillaries

94
gases occur in which of the following?
Exchange of nutrients, water and gases
A. Arterioles occur in the capillaries.
B. Capillaries
C. Venules Reference:
D. Lymphatics Costanzo, L.S. (2018). BRS Physiology (7th ed.,
p. 130) Lippincott Williams and Wilkins.

95. Which of the following is B B. Thick ascending limb


impermeable to water?
Of the options, only the thick ascending
A. Proximal tubule limb of the loop of Henle is
B. Thick ascending limb impermeable to water.
C. Late distal tubule
D. Collecting ducts
Reference:
Costanzo, L.S. (2018). BRS Physiology (7th ed.,
p. 324) Lippincott Williams and Wilkins.

96. Taste sensation on the posterior ⅓ C C. CN IX


of the tongue is innervated by ____:
CN VII: Taste on anterior ⅔ of the
A. CN V tongue
B. CN VII
C. CN IX CN IX: Taste on posterior ⅓ of the
D. CN XII tongue

Reference:
Costanzo, L.S. (2018). BRS Physiology (7th ed.,
p. 98) Lippincott Williams and Wilkins.

97. Gastric emptying is inhibited by A A. CCK


____:
Presence of fat stimulates the release
A. CCK of CCK which slows gastric emptying.
B. Gastrin
C. Secretin Gastrin increases gastric motility.
D. All of the above
Reference:
Costanzo, L.S. (2018). BRS Physiology (7th ed.,
p. 399) Lippincott Williams and Wilkins.

98. Ovulation occurs ____: B B. 14 days before menses

A. 14 days after menses Ovulation occurs 14 days before


B. 14 days before menses menses.
C. 10 days after menses
D. 10 days before menses Reference:
Costanzo, L.S. (2018). BRS Physiology (7th ed.,
p. 488) Lippincott Williams and Wilkins.

99. Ovulation occurs due to ___: C C. Surge of LH

A. Surge of progesterone Ovulation occurs from estrogen-


B. Surge of FSH induced surge of LH.
C. Surge of LH
D. Surge of prolacitn Reference:

95
Costanzo, L.S. (2018). BRS Physiology (7th ed.,
p. 488) Lippincott Williams and Wilkins.

100. Among the following thyroid A A. T3


hormones, which is the most potent? ● T3 is 3-4x more potent than T4.
● rT3 is inactive.
A. T3
B. T4 Reference:
C. rT3 Costanzo, L.S. (2018). BRS Physiology (7th ed.,
p. 450) Lippincott Williams and Wilkins.
D. They have the same potency

96
MICROBIOLOGY

97
QUESTION ANSWER EXPLANATION

1. The following are TRUE C C. The drug of choice is iodoquinol


regarding Giardiasis EXCEPT:
The drug of choice for Giardiasis is
A. The mode of transmission is Metronidazole, not Iodoquinol.
through ingestion of mature
cysts Drugs of choice for other intestinal
B. Associated with villous protozoa:
flattening and crypt
hypertrophy Intestinal Drug of choice
C. The drug of choice is protozoa
iodoquinol
D. Trophozoite is described as Entamoeba Iodoquinol or
heart-shaped histolytica paromomycin
(Metronidazole for
mild, moderate,
severe intestinal
disease)

Cyclospora Trimethoprim
sulfamethoxazole

Cryptosporidium Nitazoxanide for


immunocompetent

Reference:
Riedel, S., Morse, S., Mietzner, T., & Miller, S.
(2019). Jawetz Melnick & Adelbergs Medical
Microbiology. (28th ed., p. 723, p. 727). McGraw-Hill
Education.

2. A patient was brought to the ER C C. Ingestion of eggs


for a 3-month history of headache,
nausea, vomiting, and repeated Taenia solium:
episodes of fever. He was
diagnosed with neurocysticercosis. Ingestion of larvae → intestinal tapeworm
He most likely acquired the
infecting organism through what
method of transmission? Ingestion of eggs → cysticercosis or
neurocysticercosis
A. Ingestion of trophozoites
B. Ingestion of larvae References:
● Le, T., Bhushan, V., & Sochat, M. (2023). First
C. Ingestion of eggs Aid for the USMLE Step 1 2022, Thirty Third
D. Any of the above Edition (33rd ed., p. 157). McGraw-Hill
Education.
● Riedel, S., Morse, S., Mietzner, T., & Miller, S.
(2019). Jawetz Melnick & Adelbergs Medical
Microbiology. (28th ed., p. 748). McGraw-Hill
Education.

98
3. Microcytic anemia is seen in D D. Necator americanus
infections by which of the following
parasites? Hookworm disease (Necator americanus
and Ancylostoma duodenale) is
A. Strongyloides stercoralis characterized by microcytic anemia and
B. Diphyllobothrium latum iron deficiency.
C. Trichinella spiralis
D. Necator americanus D. latum → can cause Vit. B12 deficiency

T. spiralis → can cause myositis or


Trichinosis manifested by fever, vomiting,
periorbital edema, myalgia

S. stercoralis → accelerated autoinfection


in the immunocompromised can cause
hyperinfection.

References:
● Le, T., Bhushan, V., & Sochat, M. (2023). First
Aid for the USMLE Step 1 2022, Thirty Third
Edition (33rd ed., p. 156). McGraw-Hill
Education.
● Riedel, S., Morse, S., Mietzner, T., & Miller, S.
(2019). Jawetz Melnick & Adelbergs Medical
Microbiology. (28th ed., p. 746, p.749). McGraw-
Hill Education.

4. Chocolate agar contains B B. Haemophilus influenzae


hematin and NAD+ which are
required for the isolation of which of ● Identification of Haemophilus
the following organisms? influenzae involves using Factors V
(NAD+) and X (hemin).
A. Serratia marcescens ● Serratia is a lactose-fermenting
B. Haemophilus influenzae enteric bacteria that grows pink
C. Corynebacterium colonies on MacConkey agar.
diphtheriae ● Corynebacterium diphtheriae
D. Bordetella pertussis shows gram (+) rods with
metachromatic granules.
● Bordetella pertussis is isolated
using Bordet-Gengou agar or
Regan-Lowe medium

References:
● Le, T., Bhushan, V., & Sochat, M. (2023). First
Aid for the USMLE Step 1 2022, Thirty Third
Edition (33rd ed., p. 124, p. 137). McGraw-Hill
Education.
● Riedel, S., Morse, S., Mietzner, T., & Miller, S.
(2019). Jawetz Melnick & Adelbergs Medical
Microbiology. (28th ed., p. 275). McGraw-Hill
Education.

99
5. In osteomyelitis caused by A A. Metaphysis
caused by S. aureus, the primary
focus is found in which part of the In osteomyelitis caused by caused by S.
bone? aureus, the primary focus of growth is
found in a terminal blood vessel of the long
A. Metaphysis bone’s metaphysis.
B. Diaphysis
C. Epiphysis Reference:
Riedel, S., Morse, S., Mietzner, T., & Miller, S.
D. No particular area (2019). Jawetz Melnick & Adelbergs Medical
Microbiology. (28th ed., p. 209). McGraw-Hill
Education.

6. Which of the following toxins is A A. Panton-Valentine Leukocidin


an important virulence factor in CA-
MRSA? Panton-Valentine Leukocidin (PVL) is
mobile phage-encoded S. aureus toxin
A. Panton-Valentine which can kill white blood cells of both
Leukocidin humans and rabbits.
B. Hyaluronidase
C. Clumping factor It is an important virulence factor in CA-
D. Hemolysin MRSA.

Other S. aureus enzymes and toxins:

Enzymes and Function


toxins

Catalase ● Converts hydrogen


peroxide into water
and oxygen.
● Differentiates
staphylococci
(positive) from
streptococci
(negative).

Coagulase Synonymous with


invasive pathogenic
potential

Clumping Responsible for the


factor adherence of S. aureus
to fibrinogen and fibrin.

Hyaluronidase ● Aka “spreading


factor”
● Staphylokinase
which results in
fibrinolysis

100
Hemolysins Lyses white blood cells
through pore formation

References:
● Le, T., Bhushan, V., & Sochat, M. (2023). First
Aid for the USMLE Step 1 2022, Thirty Third
Edition (33rd ed., p. 133). McGraw-Hill
Education.
● Riedel, S., Morse, S., Mietzner, T., & Miller, S.
(2019). Jawetz Melnick & Adelbergs Medical
Microbiology. (28th ed., pp. 207-208). McGraw-
Hill Education.

7. A throat swab was done to a D D. Streptococcus pyogenes


patient which showed Gram (+)
cocci in pairs which are catalase A is incorrect since S. aureus is catalase
negative. It showed complete positive.
hemolysis and was susceptible to
bacitracin. The organism identified B is incorrect because Streptococcus
is most likely: agalactiae is not inhibited by bacitracin.

A. Staphylococcus aureus C is incorrect because Viridans streptococci


B. Streptococcus agalactiae are alpha hemolytic (showing partial
C. Viridans streptococci hemolysis), not beta hemolytic.
D. Streptococcus pyogenes
A concise algorithm for the laboratory
identification of Gram-positive organisms
may be accessed in USMLE First Aid 2023,
33rd ed (p. 132).

Reference:
Le, T., Bhushan, V., & Sochat, M. (2023). First Aid for
the USMLE Step 1 2022, Thirty Third Edition (33rd
ed., p. 132). McGraw-Hill Education.

8. The following HIV medications C C. Enfuvirtide


are nucleoside/nucleotide reverse
transcriptase inhibitors (NRTIs), Enfuvirtide is a fusion inhibitor.
EXCEPT:
The rest of the options are considered
A. Didanosine NRTIs.
B. Tenofovir
C. Enfuvirtide Other NRTIs:
D. Zidovudine ● Abacavir
● Emtricitabine
● Lamivudine
● Stavudine
Reference:
Riedel, S., Morse, S., Mietzner, T., & Miller, S.
(2019). Jawetz Melnick & Adelbergs Medical
Microbiology. (28th ed., p. 667). McGraw-Hill
Education.

101
9. In neonates, the classic triad of C C. Toxoplasma gondii
chorioretinitis, hydrocephalus, and
intracranial calcifications is Congenital toxoplasmosis is
associated with: characterized by the triad of
● Chorioretinitis
A. Rubella ● Hydrocephalus, and
B. CMV ● Intracranial calcifications
C. Toxoplasma gondii
D. HIV Aside from the above, it can also lead to
stillbirths, psychomotor disturbances or
microcephaly.

CMV is associated with chorioretinitis and


periventricular calcifications, not
intracranial.

References:
● Le, T., Bhushan, V., & Sochat, M. (2023). First
Aid for the USMLE Step 1 2022, Thirty Third
Edition (33rd ed., p. 181). McGraw-Hill
Education.
● Riedel, S., Morse, S., Mietzner, T., & Miller, S.
(2019). Jawetz Melnick & Adelbergs Medical
Microbiology. (28th ed., p. 739). McGraw-Hill
Education.

10. What is the intermediate host of B B. Pigs


Taenia solium?
Pigs are the intermediate host of T. solium.
A. Rats
B. Pigs But humans can also be intermediate hosts
C. Cattle if they ingest T. solium eggs leading to
D. Horse cysticercosis.

Reference:
Riedel, S., Morse, S., Mietzner, T., & Miller, S.
(2019). Jawetz Melnick & Adelbergs Medical
Microbiology. (28th ed., p. 748). McGraw-Hill
Education.

11. Which of the following S. A A. Pyrogenic exotoxin


pyogenes toxins has been
associated with streptococcal toxic Pyrogenic exotoxins (aka erythrogenic
shock syndrome? toxin A) are produced by group A
streptococci that carry a lysogenic phage.
A. Pyrogenic exotoxin
B. Streptokinase They are associated with the development
C. Deoxyribonuclease of streptococcal toxic shock syndrome and
D. Hyaluronidase scarlet fever.

This is different from staphylococcal toxic


shock syndrome which is promoted by
staphylococcal toxic shock syndrome toxin-
1 or TSST-1).

102
Other toxins and enzymes produced by S.
pyogenes are:

Streptokinase
● Allows escape from blood clots by
fibrin digestion.
● Given IV for treatment of pulmonary
emboli, coronary artery, and venous
thromboses.
Deoxyribonuclease
● Helps in the spread of streptococci
by liquefying pus.
Hyaluronidase
● “Spreading factor” that splits
hyaluronic acid in connective tissue.
Reference:
Riedel, S., Morse, S., Mietzner, T., & Miller, S.
(2019). Jawetz Melnick & Adelbergs Medical
Microbiology. (28th ed., pp. 218-219). McGraw-Hill
Education.

12. Lowenstein-Jensen medium is D D. Mycobacterium tuberculosis


used to culture which of the
following organisms? Mycobacterium tuberculosis is isolated
using:
A. Escherichia coli ● Lowenstein-Jensen medium
B. Mycoplasma pneumoniae ● Middlebrook medium
C. Neisseria gonorrhoeae ● Rapid automated broth cultures.
D. Mycobacterium
tuberculosis Eosin-methylene blue (EMB): isolation of
E. coli.

Eaton agar: Mycoplasma pneumoniae

Thayer-Martin agar: isolation of Neisseria


gonorrhoeae and meningitidis.

Reference:
Le, T., Bhushan, V., & Sochat, M. (2023). First Aid for
the USMLE Step 1 2022, Thirty Third Edition (33rd
ed., p. 124). McGraw-Hill Education.

13. All of the following are TRUE D D. None of the above


regarding mycobacterial cell walls,
EXCEPT? Tubercle bacilli virulence is correlated with
formation of serpentine cords by cord
A. The presence of cord factor.
formation is correlated with
virulence Cord factor is associated with:
B. Cord factor is responsible ● Inhibition of WBC migration
for the inhibition of WBC ● Formation of chronic granulomas
migration
Reference:
Riedel, S., Morse, S., Mietzner, T., & Miller, S.

103
C. Cord factor is associated (2019). Jawetz Melnick & Adelbergs Medical
with formation of chronic Microbiology. (28th ed., p. 326). McGraw-Hill
Education.
granulomas
D. None of the above

14. A 60-year-old female presents C C. Varicella zoster


to the clinic with painful vesicular
lesions on her forehead and the tip This is a case of herpes zoster
of her nose. On examination, the ophthalmicus.
lesions are present in CN V1
distribution. This is caused by reactivation of varicella
zoster virus in the ophthalmic division of the
Which of the following is the trigeminal nerve.
possible etiology of her condition?
References:
A. Herpes simplex 1 ● Le, T., Bhushan, V., & Sochat, M. (2023). First
Aid for the USMLE Step 1 2022, Thirty Third
B. Herpes simplex 2 Edition (33rd ed., p. 162). McGraw-Hill
C. Varicella zoster Education.
D. HIV ● Riedel, S., Morse, S., Mietzner, T., & Miller, S.
(2019). Jawetz Melnick & Adelbergs Medical
Microbiology. (28th ed., pp. 482-484). McGraw-
Hill Education.

15. Destruction of the vasa C C. Tertiary


vasorum occurs at which stage of
syphilis? Stages of Syphilis and Clinical
Manifestations
A. Primary
B. Secondary Stage Clinical manifestations
C. Tertiary
D. None of the above Primary Painless chancre

Secondary ● Constitutional
symptoms
● Maculopapular rash
● Condylomata lata,
● Lymphadenopathy
● Patchy hair loss

Tertiary ● Gummas
● Aortitis
● Neurosyphilis
● Argyll-Robertson pupil

Congenital syphilis
● Facial abnormalities
o Rhagades
o Snuffles
o Saddle nose
o Hutchinson teeth
o Mulberry molars
o Short maxilla

104
o Saber shins
● CN VIII deafness

References:
● Le, T., Bhushan, V., & Sochat, M. (2023). First
Aid for the USMLE Step 1 2022, Thirty Third
Edition (33rd ed., p. 145). McGraw-Hill
Education.
● Riedel, S., Morse, S., Mietzner, T., & Miller, S.
(2019). Jawetz Melnick & Adelbergs Medical
Microbiology. (28th ed., pp. 340-342). McGraw-
Hill Education

16. A 24-year-old medical student B B. Entamoeba histolytica


presents to the clinic with fever and
right upper quadrant pain. A This is most likely a case of amebic liver
diagnosis was confirmed, and abscess caused by Entamoeba
surgical drainage was performed, histolytica.
revealing an abscess with an
“anchovy paste” consistency. An “anchovy paste” characteristic of the
Which of the following agents is the abscess contents is seen upon surgical
most likely cause of these findings? drainage. The contents are described as
necrotic and bacteriologically sterile active
A. Giardia lamblia amebae being confined to the walls.
B. Entamoeba histolytica
C. Vibrio cholerae Giardiasis: presents with non-bloody, foul-
D. Shigella smelling, fatty diarrhea

Vibrio cholerae: rice-water stools

Shigella: bloody, mucoid stools (bacillary


dysentery)

References:
● Le, T., Bhushan, V., & Sochat, M. (2023). First
Aid for the USMLE Step 1 2022, Thirty Third
Edition (33rd ed., p. 152). McGraw-Hill
Education.
● Riedel, S., Morse, S., Mietzner, T., & Miller, S.
(2019). Jawetz Melnick & Adelbergs Medical
Microbiology. (28th ed., p. 728). McGraw-Hill
Education

17. Crypt hypertrophy and villous B B. Giardia lamblia


atrophy or flattening is associated
with which of the following? G. lamblia: crypt hypertrophy and villous
atrophy or flattening.
A. Entamoeba histolytica
B. Giardia lamblia E. histolytica: colon biopsy will show
C. Trichuris trichura formation of a flask-shaped ulcer.
D. Ascaris lumbricoides
Trichuris trichiura: rectal prolapse in
children.

105
Ascaris lumbricoides: ileocecal valve and
biliary obstruction, intestinal perforation,
Loeffler syndrome.

References:
● Le, T., Bhushan, V., & Sochat, M. (2023). First
Aid for the USMLE Step 1 2022, Thirty Third
Edition (33rd ed., p. 152, p. 156). McGraw-Hill
Education.
● Riedel, S., Morse, S., Mietzner, T., & Miller, S.
(2019). Jawetz Melnick & Adelbergs Medical
Microbiology. (28th ed., p. 727). McGraw-Hill
Education

18. Visceral larva migrans may lead A A. Toxocara canis


to signs and symptoms of
inflammation of the liver, visual Visceral larva migrans may lead to signs
impairment, seizures, coma, and and symptoms of inflammation of the liver,
even myocarditis. visual impairment, seizures, coma, and
even myocarditis.
What nematode is responsible for
this disease? This is caused by Toxocara canis.

A. Toxocara canis A. lumbricoides associated with Loeffler


B. Ascaris lumbricoides syndrome manifested by cough,
C. Onchocerca volvulus eosinophilia and pulmonary infiltrates.
D. Necator americanus
O. volvulus is associated with “river
blindness” and black skin nodules.

Hookworms (Ancylostoma spp. and


Necator americanus) are associated with
cutaneous larva migrans (itchy, serpiginous
rash), not visceral larva migrans.

References:
● Le, T., Bhushan, V., & Sochat, M. (2023). First
Aid for the USMLE Step 1 2022, Thirty Third
Edition (33rd ed., p. 156). McGraw-Hill
Education.
● Riedel, S., Morse, S., Mietzner, T., & Miller, S.
(2019). Jawetz Melnick & Adelbergs Medical
Microbiology. (28th ed., p. 746). McGraw-Hill
Education

19. A patient who underwent total C. S. pneumoniae


splenectomy is at risk for C
developing infections caused by Encapsulated bacteria are opsonized and
which of the following? cleared by the spleen. Patients without
spleens are at an increased risk for
A. S. aureus developing severe infections by
B. S. lugdunensis encapsulated bacteria and thus require
C. S. pneumoniae vaccination against the following:
D. S. dysenteriae ● N. meningitidis
● S. pneumoniae
● H. influenzae

106
References:
● Le, T., Bhushan, V., & Sochat, M. (2023). First
Aid for the USMLE Step 1 2022, Thirty Third
Edition (33rd ed., p. 96, p. 134). McGraw-Hill
Education.
● Riedel, S., Morse, S., Mietzner, T., & Miller, S.
(2019). Jawetz Melnick & Adelbergs Medical
Microbiology. (28th ed., p. 303). McGraw-Hill
Education.

20. Lymphatic filariasis is caused D D. Onchocerca volvulus


by the following, EXCEPT?
Lymphatic filariasis is caused by the
A. Wuchereria bancrofti following:
B. Brugia malayi ● Wuchereria bancrofti
C. Brugia timori ● Brugia malayi
D. Onchocerca volvulus ● Brugia timori

Reference:
Riedel, S., Morse, S., Mietzner, T., & Miller, S.
(2019). Jawetz Melnick & Adelbergs Medical
Microbiology. (28th ed., p. 749). McGraw-Hill
Education.

21. An immigrant from the C C. Diagnosis is based on the visualization


Caribbean is diagnosed with of elementary bodies in tissue smears or
granuloma inguinale. The following biopsy specimens.
is true regarding the condition,
EXCEPT? Granuloma inguinale is a chronic genital
ulcer caused by Klebsiella granulomatis
A. Treatment is azithromycin (formerly Calymmatobacterium
B. Caused by Klebsiella granulomatis).
granulomatis
C. Diagnosis is based on the On microscopy, there is the appearance of
visualization of elementary cytoplasmic Donovan bodies.
bodies in tissue smears or
biopsy specimens. The recommended treatment is
D. Mostly occurs in tropical azithromycin 1 g orally once per week
regions for at least three weeks or until all lesions
are healed.

An elementary body is an environmentally


stable infectious particle seen in the
developmental cycle of Chlamydia.

References:
● Le, T., Bhushan, V., & Sochat, M. (2023). First
Aid for the USMLE Step 1 2022, Thirty Third
Edition (33rd ed., p. 180). McGraw-Hill
Education.
● Riedel, S., Morse, S., Mietzner, T., & Miller, S.
(2019). Jawetz Melnick & Adelbergs Medical
Microbiology. (28th ed., p. 240, p. 367).
McGraw-Hill Education.

107
22. The characteristic red pigment C C. S. marcescens
prodigiosin is produced by which of
the following: Serratia marcescens produce prodigiosin
which gives it its characteristic red pigment.
A. Actinomyces israelii
B. S. aureus Actinomyces israelii: yellow sulfur
C. S. marcescens granules
D. P. aeruginosa
Staphylococcus aureus: forms gray to
deep golden yellow pigment

Pseudomonas aeruginosa: pyocyanin


and pyoverdine (blue and yellow green
pigment, respectively) → combination of
pyocyanin and pyoverdine produce the
bright green color characteristic of this
organism.

Reference:
Riedel, S., Morse, S., Mietzner, T., & Miller, S.
(2019). Jawetz Melnick & Adelbergs Medical
Microbiology. (28th ed., p. 241, p. 253). McGraw-Hill
Education.

23. The following ribosomal D D. 80S


subunits are present in prokaryotes,
EXCEPT? The ribosomal subunits present in
prokaryotes are:
A. 30S ● 30S
B. 50S ● 50S
C. 70S ● 70S
D. 80S
The ribosomal subunits present in
eukaryotes are:
● 30S
● 50S
● 70S

References:
● Le, T., Bhushan, V., & Sochat, M. (2023). First
Aid for the USMLE Step 1 2022, Thirty Third
Edition (33rd ed., p. 43). McGraw-Hill Education.
● Riedel, S., Morse, S., Mietzner, T., & Miller, S.
(2019). Jawetz Melnick & Adelbergs Medical
Microbiology. (28th ed., p. 117). McGraw-Hill
Education.

24. A hungry medical student eats C C. A heat-stable enterotoxin


the fried rice she has reheated for
the third time this week and is now The bacteria most likely responsible for this
suffering from nausea and vomiting. patient’s signs and symptoms is Bacillus
This happened 3 hours after eating cereus, a Gram (+) rod that causes food

108
the said fried rice. poisoning (reheated rice syndrome).

Which of the following is Its spores are not destroyed by cooking.


responsible for the patient’s
symptoms? It has 2 types:
● Emetic
A. A heat-labile enterotoxin ○ Nausea, vomiting within 1-5 h
B. Proliferation of a strain that ● Diarrheal
produces toxins A and B ○ Occurs within 8-18 h
C. A heat-stable enterotoxin ○ Non-bloody, watery diarrhea
D. Production of a delta toxin
Reference:
Le, T., Bhushan, V., & Sochat, M. (2023). First Aid for
the USMLE Step 1 2022, Thirty Third Edition (33rd
ed., p. 136). McGraw-Hill Education.

25. The agents responsible for B B. Proteins


causing spongiform
encephalopathies are made up of The agents responsible for causing
____: spongiform encephalopathies are made up
of proteins called prions.
A. Carbohydrates
B. Proteins These prions are transmissible through
C. Nucleic acids CNS-related tissues or BSE (bovine
D. All of the above spongiform encephalopathy)-contaminated
food.

Prions cause disease by causing


conformation changes in the proteins of
hosts leading to aggregation and
dysfunction.

Reference:
Riedel, S., Morse, S., Mietzner, T., & Miller, S.
(2019). Jawetz Melnick & Adelbergs Medical
Microbiology. (28th ed., p. 443). McGraw-Hill
Education.

26. Biliary tract disease and B B. Clonorchis sinensis


cholangiocarcinoma are associated
with which of the following? Clonorchis sinensis (Chinese liver fluke)
excyst in the intestine and migrate up to the
A. Schistosoma haematobium common bile duct.
B. Clonorchis sinensis
C. Echinococcus granulosus Mechanical irritation may result in fibrosis
D. Hookworms and hyperplasia.

It may also lead to chronic cholangitis may


result to atrophy of liver parenchyma, portal
fibrosis, jaundice due to biliary obstruction,

109
and liver cirrhosis.

It is also linked to cholangiocarcinoma.

The rest of the choices are not linked to


cholangiocarcinoma.

Reference:
Riedel, S., Morse, S., Mietzner, T., & Miller, S.
(2019). Jawetz Melnick & Adelbergs Medical
Microbiology. (28th ed., p. 752). McGraw-Hill
Education

27. Which of the following A A. Paragonimus westermani


trematodes can be acquired by
eating raw crabs and other Paragonimus westermani is acquired by
freshwater crustaceans? eating raw crabs and other freshwater
crustaceans.
A. Paragonimus westermani
B. Clonorchis sinensis Other trematode tissue infections:
C. Fasciola hepatica ● Clonorchis sinensis: acquired by
D. All of the above eating undercooked freshwater fish
● Fasciola hepatica: acquired by
eating watercress or aquatic
vegetation.

Reference:
Riedel, S., Morse, S., Mietzner, T., & Miller, S.
(2019). Jawetz Melnick & Adelbergs Medical
Microbiology. (28th ed., p. 743). McGraw-Hill
Education.

28. A 70/F was brought to the clinic C C. Candida vulvovaginitis


for complaints of reddened vulva
with thick white discharge. On The presence of thick, white, “cottage
microscopy, there is presence of cheese” discharge in the vulvar area points
pseudohyphae. to Candida vulvovaginitis.

What is the most likely diagnosis? Bacterial vaginosis: thin, white discharge
with fishy odor with no inflammation.
A. Bacterial vaginosis
B. Trichomoniasis Trichomoniasis: Frothy, yellow-green,
C. Candida vulvovaginitis foul-smelling discharge with inflammation of
D. Syphilis the cervix known as strawberry cervix.

Reference:
Le, T., Bhushan, V., & Sochat, M. (2023). First Aid for
the USMLE Step 1 2022, Thirty Third Edition (33rd
ed., p. 179). McGraw-Hill Education.

29. Which of the following C C. Aflatoxin


substances is found as a
contaminant of peanuts and corn Aflatoxin is a substance found as a

110
and is considered a cofactor for contaminant of peanuts and corn and is
hepatocellular carcinoma? considered a cofactor for hepatocellular
carcinoma.
A. Vinyl chloride
B. Beryllium Vinyl chloride is found in PVC pipes and
C. Aflatoxin may cause hepatic angiosarcoma.
D. Arsenic
Arsenic may cause lung cancer and
squamous cell carcinoma, not
hepatocellular carcinoma.

Beryllium exposure may cause lung


cancer.

References:
● Le, T., Bhushan, V., & Sochat, M. (2023). First
Aid for the USMLE Step 1 2022, Thirty Third
Edition (33rd ed., p. 221). McGraw-Hill
Education.
● Riedel, S., Morse, S., Mietzner, T., & Miller, S.
(2019). Jawetz Melnick & Adelbergs Medical
Microbiology. (28th ed., p. 645). McGraw-Hill
Education.

30. An elderly patient in the ICU A A. Clostridium difficile


was being treated with multiple
antibiotics for several days. He was A patient presenting with watery diarrhea
referred to you today for watery and was treated with Clindamycin makes
diarrhea. Pseudomembranous colitis the most
likely diagnosis. This is caused by
On further history, he was given Clostridium difficile.
Clindamycin for an anaerobic
infection. Shigella and E. histolytica both present
with bloody diarrhea, not watery diarrhea.
What is the most likely cause of the
patient’s signs and symptoms? References:
● Le, T., Bhushan, V., & Sochat, M. (2023). First
Aid for the USMLE Step 1 2022, Thirty Third
A. Clostridium difficile Edition (33rd ed., p. 248). McGraw-Hill
B. Shigella Education.
C. Entamoeba histolytica ● Riedel, S., Morse, S., Mietzner, T., & Miller, S.
D. Any of the above (2019). Jawetz Melnick & Adelbergs Medical
Microbiology. (28th ed., p. 191). McGraw-Hill
Education.

31. Which of the following can be B B. Metronidazole


used to treat the patient case
above? Vancomycin and metronidazole can be
used in the treatment of C. difficile.
A. Cephalosporins
B. Metronidazole Reference:
C. Fluoroquinolones Le, T., Bhushan, V., & Sochat, M. (2023). First Aid for
the USMLE Step 1 2022, Thirty Third Edition (33rd
D. Any of the above ed., p. 192). McGraw-Hill Education.

111
32. An infant who was being given A A. Malassezia
total parenteral nutrition while in the
ICU develops an infection not In infants being given TPN, an infection
responding to current antibiotics. with Malassezia can rarely occur because
of potential contamination of the lipid
Which of the following is the most emulsion.
likely etiologic organism causing
the infection? The other organisms are not associated
with contaminated TPN.
A. Malassezia
B. Rotavirus Reference:
Riedel, S., Morse, S., Mietzner, T., & Miller, S.
C. E. coli (2019). Jawetz Melnick & Adelbergs Medical
D. Staphylococcus aureus Microbiology. (28th ed., p. 681). McGraw-Hill
Education.

33. A pregnant patient was B B. Amoxicillin


diagnosed with Lyme disease.
Which of the following can be given Treatment of Lyme disease is with
for treatment? Doxycycline as the first line.

A. Doxycycline But this patient is pregnant so doxycycline


B. Amoxicillin use is discouraged.
C. Vancomycin
D. Clindamycin Amoxicillin can be given for pregnant
patients or children <8 years old.

Ceftriaxone can also be given if IV therapy


is required.

References:
● Le, T., Bhushan, V., & Sochat, M. (2023). First
Aid for the USMLE Step 1 2022, Thirty Third
Edition (33rd ed., p. 144). McGraw-Hill
Education.
● Riedel, S., Morse, S., Mietzner, T., & Miller, S.
(2019). Jawetz Melnick & Adelbergs Medical
Microbiology. (28th ed., p. 346). McGraw-Hill
Education.

34. A patient presents with flesh- D D. Poxvirus


colored umbilicated papules. The
causative agent of this skin lesions Flesh-colored umbilicated papules are
belongs to ___: caused by Molluscum contagiosum which is
classified as a Poxvirus.
A. Adenovirus
B. Papillomavirus References:
C. Polyomavirus ● Le, T., Bhushan, V., & Sochat, M. (2023). First
Aid for the USMLE Step 1 2022, Thirty Third
D. Poxvirus Edition (33rd ed., p. 161). McGraw-Hill
Education.
● Riedel, S., Morse, S., Mietzner, T., & Miller, S.
(2019). Jawetz Melnick & Adelbergs Medical
Microbiology. (28th ed., p. 508). McGraw-Hill
Education.

112
35. Which of the following receptors C C. ACE2
is used by SARS-CoV-2 for entry
into cells? ACE2 receptor is used by SARS-CoV-2 for
entry into cells.
A. P antigen on RBCs
B. Nicotinic acetylcholine Virus: receptor pairs:
receptor ● Parvovirus B19: P antigen on
C. ACE2 RBCs
D. CD21 ● Rabies: Nicotinic acetylcholine
receptor
● EBV: CD21

Reference:
Le, T., Bhushan, V., & Sochat, M. (2023). First Aid for
the USMLE Step 1 2022, Thirty Third Edition (33rd
ed., p. 163). McGraw-Hill Education.

36. Which of the following provides A A. Homozygous CCR5 mutation


protection (immunity) from HIV
infection? Homozygous CCR5 mutation →
A. Homozygous CCR5 protection (immunity)
mutation
B. Heterozygous CCR5 Heterozygous CCR5 mutation →
mutation
C. Homozygous CXCR4 slower course
mutation
D. Heterozygous CXCR4 References:
mutation ● Le, T., Bhushan, V., & Sochat, M. (2023). First
Aid for the USMLE Step 1 2022, Thirty Third
Edition (33rd ed., p. 173). McGraw-Hill
Education.
● Riedel, S., Morse, S., Mietzner, T., & Miller, S.
(2019). Jawetz Melnick & Adelbergs Medical
Microbiology. (28th ed., p. 659). McGraw-Hill
Education.

37. Which of the following B B. S. haematobium


Schistosoma species can cause
urinary symptoms? S. haematobium infections can cause
urinary symptoms such as dysuria, urinary
A. S. japonicum frequency, and hematuria.
B. S. haematobium
C. S. mansoni It is also linked to development of
D. All of the above squamous cell carcinoma of the bladder.

Reference:
Riedel, S., Morse, S., Mietzner, T., & Miller, S.
(2019). Jawetz Melnick & Adelbergs Medical
Microbiology. (28th ed., p. 753). McGraw-Hill
Education.

38. Which of the following B B. S. haematobium


Schistosoma species is identified
as egg with terminal spine? Identification of Schistosoma species:

113
A. S. japonicum ● S. japonicum: egg with barely
B. S. haematobium visible nubby spine
C. S. mansoni ● S. haematobium: egg with terminal
D. All of the above spine
● S. mansoni: egg with lateral spine

References:
● Le, T., Bhushan, V., & Sochat, M. (2023). First
Aid for the USMLE Step 1 2022, Thirty Third
Edition (33rd ed., p. 157). McGraw-Hill
Education.
● Riedel, S., Morse, S., Mietzner, T., & Miller, S.
(2019). Jawetz Melnick & Adelbergs Medical
Microbiology. (28th ed., p. 753). McGraw-Hill
Education.

39. Which of the following is A A. Hepatitis A


transmitted in a fecal-oral manner?
The transmission of Hepatitis A and
A. Hepatitis A Hepatitis E is fecal-oral.
B. Hepatitis B
C. Hepatitis C Hepatitis B, C, and D are transmitted
D. Hepatitis D parenterally.

Reference:
Riedel, S., Morse, S., Mietzner, T., & Miller, S.
(2019). Jawetz Melnick & Adelbergs Medical
Microbiology. (28th ed., p. 512). McGraw-Hill
Education.

40. Severe leptospirosis, referred to D D. Pulmonary edema


as Weil’s disease, is characterized
by the following, EXCEPT? Severe leptospirosis, referred to as
Weil’s disease, is characterized by the
A. Jaundice following:
B. Renal impairment ● Jaundice
C. Hemorrhage ● Renal impairment
D. Pulmonary edema ● Hemorrhage

Reference:
Loscalzo J, & Fauci A, & Kasper D, & Hauser S, &
Longo D, & Jameson J. (2022). Leptospirosis. In
J.F.P. Wagenaar & M.G.A. Goris (Eds.), Harrison's
Principles of Internal Medicine (21st ed., Vol. 1, p.
6171). New York, NY: McGraw Hill.

41. Rocky Mountain spotted fever is A A. Rickettsia ricketsii


caused by ___:
Rickettsia ricketsii: Rocky Mountain
A. Rickettsia ricketsii spotted fever
B. Rickettsia prowazekii
C. Rickettsia typhi Rickettsia prowazekii: Epidemic typhus
D. All of the above
Rickettsia typhi: Endemic typhus

114
References
● Le, T., Bhushan, V., & Sochat, M. (2023). First
Aid for the USMLE Step 1 2022, Thirty Third
Edition (33rd ed., p. 147). McGraw-Hill
Education.
● Riedel, S., Morse, S., Mietzner, T., & Miller, S.
(2019). Jawetz Melnick & Adelbergs Medical
Microbiology. (28th ed., p. 357). McGraw-Hill
Education.

42. Adenovirus can cause all of the D D None of the above


following, EXCEPT?
Adenovirus can cause all of the following:
A. Hemorrhagic cystitis ● Hemorrhagic cystitis
B. Pneumonia ● Pneumonia
C. Myocarditis ● Myocarditis
D. None of the above ● Conjunctivitis
● Gastroenteritis

References:
● Le, T., Bhushan, V., & Sochat, M. (2023). First
Aid for the USMLE Step 1 2022, Thirty Third
Edition (33rd ed., p. 161). McGraw-Hill
Education.
● Riedel, S., Morse, S., Mietzner, T., & Miller, S.
(2019). Jawetz Melnick & Adelbergs Medical
Microbiology. (28th ed., p. 417). McGraw-Hill
Education.

43. Which of the following vaccines B B. Rotavirus


has intussusception as one of its
adverse effects? Rotavirus vaccine has been associated
with uncommon occurrence of
A. MMR intussusception.
B. Rotavirus
C. Tdap In 1998, an oral live attenuated rotavirus
D. Hep B was withdrawn a year after being licensed
because of reports of intussusception as an
adverse effect.

Intussusception has also been associated


with recent viral infection with adenovirus.

References:
● Le, T., Bhushan, V., & Sochat, M. (2023). First
Aid for the USMLE Step 1 2022, Thirty Third
Edition (33rd ed., p. 382). McGraw-Hill
Education.
● Riedel, S., Morse, S., Mietzner, T., & Miller, S.
(2019). Jawetz Melnick & Adelbergs Medical
Microbiology. (28th ed., p. 551). McGraw-Hill
Education.

44. Which of the following is the A A. Coxsackie A


cause of hand-foot-and-mouth

115
disease (HFMD)? HFMD is caused by Coxsackievirus A16
and enterovirus 71. It presents with:
A. Coxsackie A ● Oral and pharyngeal ulcerations
B. HHV 6 ● Vesicular rash of the palms and
C. HHV 8 soles that may spread to the arms
D. Parvovirus B19 and legs.
Reference:
Riedel, S., Morse, S., Mietzner, T., & Miller, S.
(2019). Jawetz Melnick & Adelbergs Medical
Microbiology. (28th ed., p. 539). McGraw-Hill
Education.

45. Which of the following is the C C. Parvovirus


smallest DNA virus?
The smallest DNA viruses are the
A. Poxvirus Parvoviruses.
B. Papillomavirus
C. Parvovirus Reference:
D. Picornavirus Riedel, S., Morse, S., Mietzner, T., & Miller, S.
(2019). Jawetz Melnick & Adelbergs Medical
Microbiology. (28th ed., p. 457). McGraw-Hill
Education.

46. Infection with JC virus in an HIV C C. Progressive multifocal


patient may result in which of the leukoencephalopathy
following?
Infection with JC virus in an HIV patient
A. Bovine spongiform (immunocompromised) may result in
encephalopathy development of progressive multifocal
B. Alzheimer disease leukoencephalopathy or PML.
C. Progressive multifocal
leukoencephalopathy JC virus is a Polyomavirus.
D. Aplastic crisis
Reference:
Le, T., Bhushan, V., & Sochat, M. (2023). First Aid for
the USMLE Step 1 2022, Thirty Third Edition (33rd
ed., p. 161). McGraw-Hill Education.

47. A child with beta thalassemia B B. Parvovirus B19


was rushed to the ER for pallor. On
workup, he was found to have an Parvovirus B19 can cause transient
aplastic crisis. aplastic crisis in patients with:
● Sickle cell disease
Which of the following agents must ● Thalassemias
have precipitated his episode of ● Acquired hemolytic anemia in adults
aplastic crisis?
References:
A. Polyomavirus ● Le, T., Bhushan, V., & Sochat, M. (2023). First
Aid for the USMLE Step 1 2022, Thirty Third
B. Parvovirus B19 Edition (33rd ed., p. 425). McGraw-Hill
C. Coxsackievirus A16 Education.
D. Hepatitis virus ● Riedel, S., Morse, S., Mietzner, T., & Miller, S.
(2019). Jawetz Melnick & Adelbergs Medical
Microbiology. (28th ed., p. 460). McGraw-Hill
Education.

116
48. Which of the following B B. Ancylostoma
nematodes is not acquired by the
ingestion of its eggs? The route of infection of the following
nematodes is through the skin:
A. Enterobius ● Strongyloides
B. Ancylostoma ● Ancylostoma
C. Trichuris ● Necator
D. Trichinella
The route of infection of the following
nematodes is through ingestion of eggs:
● Enterobius
● Ascaris
● Toxocara
● Trichinella
● Trichuris

References:
● Le, T., Bhushan, V., & Sochat, M. (2023). First
Aid for the USMLE Step 1 2022, Thirty Third
Edition (33rd ed., p. 155). McGraw-Hill
Education.
● Riedel, S., Morse, S., Mietzner, T., & Miller, S.
(2019). Jawetz Melnick & Adelbergs Medical
Microbiology. (28th ed., p. 742). McGraw-Hill
Education.

49. All of the following are TRUE C C. Treatment is with Praziquantel


about Trichuris trichiura, EXCEPT?

A. Involves the cecum Treatment of Trichuris trichiura is with


B. Acquired through ingestion mebendazole, albendazole or pyrantel
of eggs pamoate.
C. Treatment is with
Praziquantel Reference:
D. None of the above Riedel, S., Morse, S., Mietzner, T., & Miller, S.
(2019). Jawetz Melnick & Adelbergs Medical
Microbiology. (28th ed., p. 742). McGraw-Hill
Education.

50. What is the most probable B B. Staphylococcus aureus


infecting organism in a patient with
a history of IV drug abuse who
developed infective endocarditis? In patients with history of IV drug abuse,
infective endocarditis can develop from the
A. Staphylococcus epidermidis following organisms:
B. Staphylococcus aureus ● S. aureus
C. Coxiella ● Pseudomonas
D. Enterococcus ● Candida

References:
● Le, T., Bhushan, V., & Sochat, M. (2023). First
Aid for the USMLE Step 1 2022, Thirty Third
Edition (33rd ed., p. 318). McGraw-Hill
Education.
● Riedel, S., Morse, S., Mietzner, T., & Miller, S.
(2019). Jawetz Melnick & Adelbergs Medical
Microbiology. (28th ed., p. 796). McGraw-Hill

117
Education.

51. Culture-negative endocarditis C C. Coxiella


can be caused by ____:
Among the choices, culture-negative
A. Staphylococcus epidermidis endocarditis can be caused by Coxiella.
B. Staphylococcus aureus
C. Coxiella Reference:
D. Enterococcus ● Le, T., Bhushan, V., & Sochat, M. (2023). First
Aid for the USMLE Step 1 2022, Thirty Third
Edition (33rd ed., p. 318). McGraw-Hill
Education.
● Loscalzo J, & Fauci A, & Kasper D, & Hauser S,
& Longo D, & Jameson J. (2022). Bartonella
infections, including cat-scratch disease. In M.
Giladi & M. Ephros (Eds.), Harrison's Principles
of Internal Medicine (21st ed., Vol. 2, p. 772).
New York, NY: McGraw Hill.

52. Chronic enlargement of the A A. Adult worms


limbs, breasts, and genitalia, known
as elephantiasis, is caused by what The main cause of the inflammatory and
form of the causative filariid fibrotic reactions which lead to the signs
nematodes? and symptoms of elephantiasis is the adult
worms residing in lymph tissues.
A. Adult worms
B. Microfilariae Reference:
C. Eggs Riedel, S., Morse, S., Mietzner, T., & Miller, S.
(2019). Jawetz Melnick & Adelbergs Medical
D. Larvae Microbiology. (28th ed., p. 750). McGraw-Hill
Education.

53. Struvite stone formation in the C C. Proteus spp.


kidney is associated with which of
the following organisms: Struvite stone formation is associated with
Proteus due to the presence of urease
A. Clostridium difficile which splits urea into ammonia and CO2,
B. Pseudomonas spp. thereby increasing urine pH and risk for
C. Proteus spp. stone formation.
D. Serratia marcescens
The rest of the options are not urease-
positive organisms and are not associated
with struvite stone formation.

References:
● Le, T., Bhushan, V., & Sochat, M. (2023). First
Aid for the USMLE Step 1 2022, Thirty Third
Edition (33rd ed., p. 125). McGraw-Hill
Education.
● Riedel, S., Morse, S., Mietzner, T., & Miller, S.
(2019). Jawetz Melnick & Adelbergs Medical
Microbiology. (28th ed., p. 241). McGraw-Hill
Education.

54. Which of the following B B. Clostridium perfringens


organisms possesses a toxin that

118
can cause myonecrosis and C. perfringens secretes alpha toxin which
hemolysis? can cause myonecrosis in the form of gas
gangrene and hemolysis.
A. Clostridium difficile
B. Clostridium perfringens References:
C. Streptococcus pyogenes ● Le, T., Bhushan, V., & Sochat, M. (2023). First
Aid for the USMLE Step 1 2022, Thirty Third
D. Staphylococcus aureus Edition (33rd ed., p. 131). McGraw-Hill
Education.
● Riedel, S., Morse, S., Mietzner, T., & Miller, S.
(2019). Jawetz Melnick & Adelbergs Medical
Microbiology. (28th ed., p. 162). McGraw-Hill
Education.

55. Patients with sickle cell disease C C. Salmonella


are susceptible to bacteremia from
what organism? Patients with sickle cell disease are
susceptible to bacteremia from Salmonella
A. Streptococcus and S. aureus.
B. Neisseria
C. Salmonella References:
D. Haemophilus ● Le, T., Bhushan, V., & Sochat, M. (2023). First
Aid for the USMLE Step 1 2022, Thirty Third
Edition (33rd ed., p. 177). McGraw-Hill
Education.
● Riedel, S., Morse, S., Mietzner, T., & Miller, S.
(2019). Jawetz Melnick & Adelbergs Medical
Microbiology. (28th ed., p. 249). McGraw-Hill
Education.

56. What is the most prevalent of all B B. Tinea pedis


dermatophytoses?
The most prevalent of all dermatophytoses
A. Tinea corporis is tinea pedis or athlete’s foot.
B. Tinea pedis
C. Tinea cruris
D. Tinea capitis Reference:
Riedel, S., Morse, S., Mietzner, T., & Miller, S.
(2019). Jawetz Melnick & Adelbergs Medical
Microbiology. (28th ed., p. 683). McGraw-Hill
Education.

57. What hemorrhagic and necrotic B B. Ecthyma gangrenosum


skin lesion develops in patients with
sepsis from P. aeruginosa? Ecthyma gangrenosum is a hemorrhagic
and necrotic skin lesion that develops in
A. Pyoderma gangrenosum patients with sepsis from P. aeruginosa.
B. Ecthyma gangrenosum
C. Gas gangrene Pyoderma gangrenosum occurs in
D. All of the above ulcerative colitis and Crohn disease.

Gas gangrene is caused by C. perfringens.

References:
● Le, T., Bhushan, V., & Sochat, M. (2023). First
Aid for the USMLE Step 1 2022, Thirty Third
Edition (33rd ed., p. 389). McGraw-Hill

119
Education.
● Riedel, S., Morse, S., Mietzner, T., & Miller, S.
(2019). Jawetz Melnick & Adelbergs Medical
Microbiology. (28th ed., p. 255). McGraw-Hill
Education.

58. Tetanus causes spastic A A. Inhibiting release of GABA and glycine


paralysis by ____:
Tetanus causes spastic paralysis by
A. Inhibiting release of GABA inhibiting the release of inhibitory
and glycine neurotransmitters like GABA and glycine.
B. Cleaving SNARE proteins
C. Releasing alpha toxin C. botulinum causes botulism by cleaving
D. Inactivating 60S ribosome SNARE proteins and decreasing
acetylcholine release.

C. perfringens releases alpha toxin.

Shiga toxin inactivates 60S ribosome to


cause hemolytic uremic syndrome.

References:
● Le, T., Bhushan, V., & Sochat, M. (2023). First
Aid for the USMLE Step 1 2022, Thirty Third
Edition (33rd ed., p. 711). McGraw-Hill
Education.
● Riedel, S., Morse, S., Mietzner, T., & Miller, S.
(2019). Jawetz Melnick & Adelbergs Medical
Microbiology. (28th ed., p. 162). McGraw-Hill
Education.

59. Which of the following is NOT A A. Produces a heat-stable toxin


true of C. botulinum?
The toxin released by C. botulinum is heat-
A. Produces a heat-stable labile, not heat-stable.
toxin
B. Causes floppy baby C. botulinum causes floppy baby
syndrome syndrome.
C. Toxin decreases
acetylcholine at the C. botulinum causes botulism by cleaving
neuromuscular junction SNARE proteins and decreasing
D. None of the above acetylcholine release.

References:
● Le, T., Bhushan, V., & Sochat, M. (2023). First
Aid for the USMLE Step 1 2022, Thirty Third
Edition (33rd ed., p. 136). McGraw-Hill
Education.
● Riedel, S., Morse, S., Mietzner, T., & Miller, S.
(2019). Jawetz Melnick & Adelbergs Medical
Microbiology. (28th ed., p. 800). McGraw-Hill
Education.

60. Which infection by the following A A. S. japonicum


organisms may result in portal

120
hypertension? S. japonicum eggs may cause
granulomatous reaction and fibrosis of the
A. S. japonicum liver leading to portal hypertension.
B. S. haematobium
C. Paragonimus westermani Paragonimus affects the lung which leads
D. All of the above to pulmonary symptoms such as cough and
hemoptysis.

Reference:
Riedel, S., Morse, S., Mietzner, T., & Miller, S.
(2019). Jawetz Melnick & Adelbergs Medical
Microbiology. (28th ed., pp. 752-753). McGraw-Hill
Education.

61. Which of the following should C C. Paragonimus westermani


be considered as an important
differential diagnosis when Paragonimus affects the lung which leads
considering pulmonary TB? to pulmonary symptoms such as cough and
hemoptysis. These symptoms are similar to
A. S. japonicum patients with PTB and therefore, should be
B. S. haematobium considered as an important differential
C. Paragonimus westermani diagnosis when considering PTB.
D. Fasciola
Reference:
Riedel, S., Morse, S., Mietzner, T., & Miller, S.
(2019). Jawetz Melnick & Adelbergs Medical
Microbiology. (28th ed., p. 752). McGraw-Hill
Education.

62. What diagnostic test for C C. Bone marrow culture


Typhoid fever is considered the
most sensitive? The most sensitive test for Typhoid fever is
bone marrow culture but are clinically less
A. Urine culture practical than other tests.
B. Stool culture
C. Bone marrow culture Reference:
D. Serologic tests Riedel, S., Morse, S., Mietzner, T., & Miller, S.
(2019). Jawetz Melnick & Adelbergs Medical
Microbiology. (28th ed., p. 247). McGraw-Hill
Education.

63. What is the most common B B. Enterocolitis


manifestation of salmonella
infection? Enterocolitis is the most common
manifestation of salmonella infection.
A. Bacteremia
B. Enterocolitis Reference:
C. Meningitis Riedel, S., Morse, S., Mietzner, T., & Miller, S.
(2019). Jawetz Melnick & Adelbergs Medical
D. Osteomyelitis Microbiology. (28th ed., p. 247). McGraw-Hill
Education.

64. What is the main cause of B B. Trichophyton tonsurans


“black dot” tinea capitis?
Trichophyton tonsurans produces spores
A. Microsporum inside the hair shaft (endothrix) and is the
B. Trichophyton tonsurans main cause of black dot tinea capitis.
C. Trichophyton rubrum

121
D. All of the above Reference:
Riedel, S., Morse, S., Mietzner, T., & Miller, S.
(2019). Jawetz Melnick & Adelbergs Medical
Microbiology. (28th ed., p. 684). McGraw-Hill
Education.

65. A patient with HIV develops A A. EBV


wart-like growth on the tongue. On
examination, oral hairy leukoplakia Oral hairy leukoplakia is a focus of EBV
was the most likely diagnosis. replication and is described as wart-like
growth on the tongue.
What is the causative agent of this
lesion? Reference:
Riedel, S., Morse, S., Mietzner, T., & Miller, S.
(2019). Jawetz Melnick & Adelbergs Medical
A. EBV Microbiology. (28th ed., p. 487). McGraw-Hill
B. CMV Education.
C. HSV
D. VZV

66. EBV is associated with the D D. None of the above


following malignancies, EXCEPT?
EBV is associated with the following
A. Hodgkin lymphoma malignancies:
B. Non-Hodgkin lymphoma ● Burkitt lymphoma
C. Nasopharyngeal carcinoma ● Hodgkin lymphoma
D. None of the above ● Non-Hodgkin lymphoma
● Nasopharyngeal carcinoma
● Gastric carcinoma

Reference:
Riedel, S., Morse, S., Mietzner, T., & Miller, S.
(2019). Jawetz Melnick & Adelbergs Medical
Microbiology. (28th ed., p. 487). McGraw-Hill
Education.

67. In patients with infectious A A. Amoxicillin


mononucleosis, use of which of the
following can cause a In patients with infectious mononucleosis,
maculopapular rash? use of Amoxicillin (for a presumed
streptococcal pharyngitis) can cause a
A. Amoxicillin maculopapular rash.
B. Doxycycline
C. Azithromycin Reference:
D. All of the above Le, T., Bhushan, V., & Sochat, M. (2023). First Aid for
the USMLE Step 1 2022, Thirty Third Edition (33rd
ed., p. 162). McGraw-Hill Education.

68. The bite of an Anopheles B B. Sporozoite


mosquito onto a human will inject
which of the following forms of the The bite of an Anopheles mosquito will
malarial parasite? inject sporozoites into the bloodstream of a
human.
A. Merozoite
B. Sporozoite Gametocytes are the form of the malarial
C. Gametocyte parasite that is taken up and ingested by
D. Schizont bloodsucking female Anopheles.

122
Reference:
Riedel, S., Morse, S., Mietzner, T., & Miller, S.
(2019). Jawetz Melnick & Adelbergs Medical
Microbiology. (28th ed., p. 735). McGraw-Hill
Education.

69. A thick and thin blood smear B B. P. falciparum


from a malaria patient from
Palawan reveals crescent-shaped On blood smear, P. falciparum infection will
gametocytes and trophozoite rings show:
within RBCs. ● Crescent-shaped gametocytes, and
● Trophozoite rings within RBCs
This is characteristic of which of the
following? P. malariae: Trophozoite rings within RBCs
with round or oval gametocytes
A. P. malariae
P. vivax/ovale: Trophozoites and Schuffner
B. P. falciparum stippling within RBC cytoplasm
C. P. ovale
D. P. vivax References:
● Le, T., Bhushan, V., & Sochat, M. (2023). First
Aid for the USMLE Step 1 2022, Thirty Third
Edition (33rd ed., p. 154). McGraw-Hill
Education.
● Riedel, S., Morse, S., Mietzner, T., & Miller, S.
(2019). Jawetz Melnick & Adelbergs Medical
Microbiology. (28th ed., p. 736). McGraw-Hill
Education.

70. Which of the following malarial B B. P. falciparum


parasites is associated with
cerebral malaria? P. falciparum infections are more severe
and has a higher rate of fatal complications
A. P. malariae such as cerebral malaria.
B. P. falciparum
C. P. ovale Reference:
D. P. vivax Riedel, S., Morse, S., Mietzner, T., & Miller, S.
(2019). Jawetz Melnick & Adelbergs Medical
Microbiology. (28th ed., p. 736). McGraw-Hill
Education.

72. What is the treatment of choice B Pyrimethamine + sulfadiazine is the


for Toxoplasmosis? treatment of choice for toxoplasmosis.

A. Fluconazole Toxoplasma infection in a pregnant


B. Pyrimethamine plus woman may lead to congenital
sulfadiazine toxoplasmosis.
C. Amphotericin B
D. Metronidazole Congenital toxoplasmosis is
characterized by the triad of
● Chorioretinitis
● Hydrocephalus, and
● Intracranial calcifications

Aside from the above, it can also lead to

123
stillbirths, psychomotor disturbances, or
microcephaly.

Reference:
Riedel, S., Morse, S., Mietzner, T., & Miller, S.
(2019). Jawetz Melnick & Adelbergs Medical
Microbiology. (28th ed., p. 725). McGraw-Hill
Education.

73. The classic triad of B B. Rubella


sensorineural deafness, cataracts,
and patent ductus arteriosus is The classic triad of sensorineural deafness,
seen in which of the following? cataracts, and patent ductus arteriosus is
seen in patients with congenital rubella.
A. CMV
B. Rubella Congenital toxoplasmosis is
C. Rubeola characterized by the triad of
D. Toxoplasma ● Chorioretinitis
● Hydrocephalus, and
● Intracranial calcifications

Aside from the above, it can also lead to


stillbirths, psychomotor disturbances, or
microcephaly.

References:
● Le, T., Bhushan, V., & Sochat, M. (2023). First
Aid for the USMLE Step 1 2022, Thirty Third
Edition (33rd ed., p. 166, p. 181). McGraw-Hill
Education.
● Riedel, S., Morse, S., Mietzner, T., & Miller, S.
(2019). Jawetz Melnick & Adelbergs Medical
Microbiology. (28th ed., p. 725, p. 739).
McGraw-Hill Education.

74. A patient presents to the clinic A A. Ascaris lumbricoides


with a 2-month history of cough. On
workup, CBC showed eosinophilia. The patient’s symptoms of cough, CBC
CXR was also done which showed which showed eosinophilia and CXR
pulmonary infiltrates. showing pulmonary infiltrates point to a
possible case of Loeffler syndrome from
What is the most likely cause of this A. lumbricoides.
patient’s symptoms?
B and C cause microcytic anemia from iron
A. Ascaris lumbricoides deficiency.
B. Ancylostoma duodenale
C. Necator americanus D causes rectal prolapse in children.
D. Trichuris trichiura
Reference:
Riedel, S., Morse, S., Mietzner, T., & Miller, S.
(2019). Jawetz Melnick & Adelbergs Medical
Microbiology. (28th ed., p. 746). McGraw-Hill
Education.

75. Which of the following C C. Strongyloides stercoralis


trematodes can sustain an infection
for many years and can lead to a Strongyloides stercoralis can sustain an

124
hyperinfection syndrome in the infection for many years and can lead to a
immunocompromised state? hyperinfection syndrome when the host
becomes immunocompromised. The
A. Ascaris lumbricoides hyperinfection syndrome can lead to an
B. Trichuris trichiura accelerated and fatal infection.
C. Strongyloides stercoralis
D. Trichinella spiralis

References:
● Le, T., Bhushan, V., & Sochat, M. (2023). First
Aid for the USMLE Step 1 2022, Thirty Third
Edition (33rd ed., p. 156). McGraw-Hill
Education.
● Riedel, S., Morse, S., Mietzner, T., & Miller, S.
(2019). Jawetz Melnick & Adelbergs Medical
Microbiology. (28th ed., p. 747). McGraw-Hill
Education.

76. A female patient comes to the B This is a case of bacterial vaginosis


clinic complaining of a grayish caused by Gardnerella vaginalis.
vaginal discharge with a fishy smell.
She denies any other symptoms. The patient presents with thin, whitish
vaginal discharge that has a fishy odor.
Microscopy showed “clue cells”.
On microscopy, there are “clue cells” which
What is the suggested treatment for are vaginal epithelial cells covered with
her condition? Gardnerella.

A. Azithromycin Treatment is with metronidazole or


B. Metronidazole clindamycin.
C. Ceftriaxone
D. Cefixime References:
● Le, T., Bhushan, V., & Sochat, M. (2023). First
Aid for the USMLE Step 1 2022, Thirty Third
Edition (33rd ed., p. 147). McGraw-Hill
Education.
● Riedel, S., Morse, S., Mietzner, T., & Miller, S.
(2019). Jawetz Melnick & Adelbergs Medical
Microbiology. (28th ed., p. 309). McGraw-Hill
Education.

76. Trichinellosis is acquired by B B. Eating undercooked pork


____:
Trichinellosis, caused by Trichinella spiralis,
A. Eating undercooked fish is acquired by eating undercooked pork.
B. Eating undercooked pork
C. Eating raw crabs Reference:
D. Eating aquatic vegetation Riedel, S., Morse, S., Mietzner, T., & Miller, S.
(2019). Jawetz Melnick & Adelbergs Medical
Microbiology. (28th ed., p. 742). McGraw-Hill
Education.

77. Which of the following D D. Amphotericin B


antifungals binds to ergosterol in
fungal cell membrane and creates Amphotericin B binds to ergosterol in
pores? fungal cell membranes and creates
membrane pores that lead to leakage of

125
A. Fluconazole ions and small molecules.
B. Terbinafine
C. Echinocandin Terbinafine works by inhibiting squalene
D. Amphotericin B epoxidase which is a fungal enzyme.

Echinocandin functions to inhibit beta


glucan synthesis.

Fluconazole works by inhibiting fungal


sterol synthesis.

References:
● Le, T., Bhushan, V., & Sochat, M. (2023). First
Aid for the USMLE Step 1 2022, Thirty Third
Edition (33rd ed., p. 196). McGraw-Hill
Education.
● Riedel, S., Morse, S., Mietzner, T., & Miller, S.
(2019). Jawetz Melnick & Adelbergs Medical
Microbiology. (28th ed., p. 712). McGraw-Hill
Education.

78. A 31-year-old horticulturist C C. Cigar-shaped yeast


presents with an oozing ulcer and
nodules on the dorsum of the left This is a case of sporotrichosis.
hand and arm, forming an
ascending lymphangitic pattern. The initial lesion is granulomatous nodule
that eventually forms a necrotic or
Which of the following findings ulcerative lesion with ascending
would you expect to see on lymphangitis.
microscopy?
The causative agent is Sporothrix
A. Spaghetti and meatballs schenckii. It appears as a cigar-shaped
appearance yeast on microscopy.
B. Soap bubble appearance
C. Cigar-shaped yeast Malassezia: Spaghetti and meatballs
D. None of the above
Cryptococcus: Soap bubble appearance
on MRI

References:
● Le, T., Bhushan, V., & Sochat, M. (2023). First
Aid for the USMLE Step 1 2022, Thirty Third
Edition (33rd ed., p. 151). McGraw-Hill
Education.
● Riedel, S., Morse, S., Mietzner, T., & Miller, S.
(2019). Jawetz Melnick & Adelbergs Medical
Microbiology. (28th ed., p. 686). McGraw-Hill
Education.

79. A lumbar puncture was done to A A. Cryptococcus neoformans


an HIV patient which revealed
encapsulated yeasts on India ink. Cryptococcus neoformans is
What is the most likely organism characterized by encapsulated yeasts on
identified? India ink.
Cryptosporidium spp is found in stool (not
A. Cryptococcus neoformans CSF) as acid-fast oocysts.

126
B. Cryptosporidium spp
C. EBV In patients with HIV, EBV is responsible for
D. CMV causing CNS lymphoma while CMV is
associated with linear ulcers on endoscopy
and cotton-wool spots on fundoscopy.

References:
Le, T., Bhushan, V., & Sochat, M. (2023). First Aid for
the USMLE Step 1 2022, Thirty Third Edition (33rd
ed., p. 174). McGraw-Hill Education.

80. An HIV patient presented with D D. Bronchoalveolar lavage


symptoms of fever, dyspnea and
cough for the past few days. Chest This is most likely a case of Pneumocystis
imaging revealed diffuse, bilateral pneumonia caused by Pneumocystic
ground-glass opacities with some jiroveci. A bronchoalveolar lavage, lung
pneumatoceles. biopsy, or pulmonary secretions with
metheneamine silver stain are used to
Which of the following should be establish the diagnosis.
used to establish the diagnosis?
Chest imaging will reveal diffuse, bilateral
A. Chest Xray ground-glass opacities with some
B. Blood culture pneumatoceles in these patients.
C. CBC
D. Bronchoalveolar lavage References:
● Le, T., Bhushan, V., & Sochat, M. (2023). First
Aid for the USMLE Step 1 2022, Thirty Third
Edition (33rd ed., p. 151). McGraw-Hill
Education.
● Riedel, S., Morse, S., Mietzner, T., & Miller, S.
(2019). Jawetz Melnick & Adelbergs Medical
Microbiology. (28th ed., p. 709). McGraw-Hill
Education.

81. Most cases of Pneumocystis D D. < 200/mm3


pneumonia occur at what level of
CD4 cell count? Most cases of Pneumocystis pneumonia
occur at CD4 cell count of < 200/mm3
A. < 500/mm3
B. < 400/mm3 References:
C. < 300/mm3 ● Le, T., Bhushan, V., & Sochat, M. (2023). First
Aid for the USMLE Step 1 2022, Thirty Third
D. < 200/mm3 Edition (33rd ed., p. 174). McGraw-Hill
Education.
● Loscalzo J, & Fauci A, & Kasper D, & Hauser S,
& Longo D, & Jameson J. (2022). Pneumocystis
infections. In A. Morris & H. Masu (Eds.),
Harrison's Principles of Internal Medicine (21st
ed., Vol. 2, p. 7339). New York, NY: McGraw
Hill.

82. Which of the following is NOT B B. Staphyloccal enterotoxin can be


true about Staphylococcus aureus? destroyed by cooking.

A. Staphyloccal enterotoxin Staphyloccal enterotoxin can NOT be


causes the most common destroyed by cooking. It is heat stable.

127
form of food poisoning
B. Staphyloccal enterotoxin Symptoms can appear 2-6 hours after
can be destroyed by ingestion of the preformed toxin.
cooking
C. Symptoms can appear 2-6 Staphylococcal food poisoning by
hours after ingestion of the staphycoccal enterotoxin is the most
toxin common form of food poisoning.
D. None of the above

References:
● Le, T., Bhushan, V., & Sochat, M. (2023). First
Aid for the USMLE Step 1 2022, Thirty Third
Edition (33rd ed., p. 133). McGraw-Hill
Education.
● Riedel, S., Morse, S., Mietzner, T., & Miller, S.
(2019). Jawetz Melnick & Adelbergs Medical
Microbiology. (28th ed., p. 163, p. 205).
McGraw-Hill Education.

83. Which of the following is A A. Low-affinity penicillin-binding proteins


responsible for the resistance of
MRSA? Low-affinity penicillin-binding proteins
(PBP2a) are responsible for conferring
A. Low-affinity penicillin- resistance to MRSA.
binding protein
B. Developing efflux pumps Efflux pumps and mutations in the DNA
C. Mutations in DNA gyrase gyrase are the mechanisms of resistance of
D. All of the above fluoroquinolones.

References:
● Le, T., Bhushan, V., & Sochat, M. (2023). First
Aid for the USMLE Step 1 2022, Thirty Third
Edition (33rd ed., p. 133, p. 192). McGraw-Hill
Education.
● Riedel, S., Morse, S., Mietzner, T., & Miller, S.
(2019). Jawetz Melnick & Adelbergs Medical
Microbiology. (28th ed., p. 206). McGraw-Hill
Education.

84. A 60/M with uncontrolled Type A A. Group A streptococci


2 diabetes mellitus comes to the
ER with erythematous, swollen and The patients signs and symptoms are
tender right lower extremity with consistent with necrotizing fasciitis.
necrotic areas and crepitus on
palpation which progressed over It can be caused by group A streptococci
the last few hours. which is sometimes termed “flesh-eating
bacteria.”
The resident mentioned that it is
most likely due to an infection with References:
“flesh-eating bacteria.” Which of the ● Le, T., Bhushan, V., & Sochat, M. (2023). First
Aid for the USMLE Step 1 2022, Thirty Third
following bacteria is the resident Edition (33rd ed., p. 487). McGraw-Hill
most likely referring to? Education.
● Riedel, S., Morse, S., Mietzner, T., & Miller, S.
A. Group A streptococci (2019). Jawetz Melnick & Adelbergs Medical
Microbiology. (28th ed., p. 219). McGraw-Hill
B. Staphylococcus aureus

128
C. Clostridium tetani Education.
D. Clostridium botulinum

85. Which of the following is the C C. Bacillus anthracis


only bacteria that has a polypeptide
capsule? The only bacteria that has a polypeptide
capsule (poly D-glutamate) is Bacillus
A. Clostridium tetani anthracis.
B. Bacillus cereus
C. Bacillus anthracis References:
D. Clostridium botulinum ● Le, T., Bhushan, V., & Sochat, M. (2023). First
Aid for the USMLE Step 1 2022, Thirty Third
Edition (33rd ed., p. 135). McGraw-Hill
Education.
● Riedel, S., Morse, S., Mietzner, T., & Miller, S.
(2019). Jawetz Melnick & Adelbergs Medical
Microbiology. (28th ed., p. 32). McGraw-Hill
Education.

86. What anthrax toxin facilitates A A. Protective antigen


the entry of the other 2 toxins into
the cell? The 3 anthrax toxins are:
● Protective antigen
A. Protective antigen ● Lethal factor
B. Lethal factor ● Edema factor
C. Edema factor
D. All of the above Protective antigen facilitates the entry of the
other 2 toxins into the cell.

Reference:
Riedel, S., Morse, S., Mietzner, T., & Miller, S.
(2019). Jawetz Melnick & Adelbergs Medical
Microbiology. (28th ed., p. 184). McGraw-Hill
Education.

87. An infant was brought to the D D. HHV-6


clinic for high fevers which occurred
for the past several days. This was The infant most likely has roseola infantum
then followed by the appearance of which is characterized by high fevers for
a diffuse macular rash. What is the several days which is then followed by the
etiologic agent of the most likely appearance of a diffuse macular rash.
diagnosis?
The causative agent is HHV-6.
A. HHV-3
B. HHV-4 HHV-3: Varicella-zoster
C. HHV-5
D. HHV-6 HHV-4: Epstein Barr

HHV-5: Cytomegalovirus

References:
● Le, T., Bhushan, V., & Sochat, M. (2023). First
Aid for the USMLE Step 1 2022, Thirty Third
Edition (33rd ed., p. 162). McGraw-Hill
Education.
● Riedel, S., Morse, S., Mietzner, T., & Miller, S.

129
(2019). Jawetz Melnick & Adelbergs Medical
Microbiology. (28th ed., p. 493). McGraw-Hill
Education.

88. A pediatric patient was brought C C. Inactivation of inhibitory G subunit and


to the clinic due to excessive activation of adenylate cyclase
coughing with a characteristic
“whoop” on inspiration. This is a case of whooping cough caused
by Bordetella pertussis.
Which of the following is the
mechanism of action of the toxin The pertussis toxin acts by inactivating
produced by causative agent in this
case? the inhibitory G subunit → activation of
adenylate cyclase → increased cAMP.
A. Inactivation of elongation
factor (EF-2)
Inactivation of elongation factor (EF-2) is
B. Overactivation of adenylate
the mechanism of diphtheria toxin.
cyclase by permanently
activating Gs
Overactivation of adenylate cyclase by
C. Inactivation of inhibitory G permanently activating Gs is the
subunit and activation of
mechanism of cholera toxin.
adenylate cyclase
D. None of the above Reference:
Le, T., Bhushan, V., & Sochat, M. (2023). First Aid for
the USMLE Step 1 2022, Thirty Third Edition (33rd
ed., p. 130). McGraw-Hill Education.

89. Kala-azar is caused by which of B Kala-azar is another term for visceral


the following parasites? leishmaniasis, it is Hindi for “black fever”.

A. Trypanosoma Trypanosoma cruzi causes Chagas


B. Leishmania disease.
C. Trichomonas
D. Plasmodium Trichomonas vaginalis causes
trichomoniasis.

Plasmodium causes malaria.

Reference:
Riedel, S., Morse, S., Mietzner, T., & Miller, S.
(2019). Jawetz Melnick & Adelbergs Medical
Microbiology. (28th ed., p. 733). McGraw-Hill
Education.

90. In patients with Typhoid fever B B. Gallbladder


who become chronic carriers, the
organisms persist in what organ? In patients with Typhoid fever who become
chronic carriers, the organisms persist in
A. Liver the gallbladder and in the biliary tract.
B. Gallbladder
C. Colon Reference:
D. Pancreas Riedel, S., Morse, S., Mietzner, T., & Miller, S.
(2019). Jawetz Melnick & Adelbergs Medical
Microbiology. (28th ed., p. 249). McGraw-Hill

130
Education.

91. Which of the following is C C. HDV


described as a defective virus that
requires the HbsAg coat for HDV requires the HbsAg coat for
transmission? transmission and is described as a
defective virus.
A. HAV
B. HCV Reference:
C. HDV Riedel, S., Morse, S., Mietzner, T., & Miller, S.
(2019). Jawetz Melnick & Adelbergs Medical
D. HEV Microbiology. (28th ed., p. 516). McGraw-Hill
Education.

92. Which of the following Gram- A A. Campylobacter jejuni


negative curved rods that are
oxidase (+) grows at 42 degrees C. jejuni is a Gram-negative curved rod
Celsius? that is oxidase (+) and grows in 42 degrees
Celsius.
A. Campylobacter jejuni
B. Vibrio cholerae Vibrio cholerae is also a Gram-negative
C. Helicobacter pylori curved rod that is oxidase (+), but it grows
D. Citrobacter in alkaline media.

Helicobacter pylori is a Gram-negative


curved rod that is oxidase (+) and produces
urease.

Check out First Aid for a complete picture


of the Gram-negative lab algorithm (p. 139).

Reference:
Le, T., Bhushan, V., & Sochat, M. (2023). First Aid for
the USMLE Step 1 2022, Thirty Third Edition (33rd
ed., p. 139). McGraw-Hill Education.

93. A patient was rushed to the ER D D. Neisseria meningitidis


due to severe headache, vomiting,
photophobia and stiff neck. He was The patient’s signs and symptoms are
also noted to have high fever with a characteristic of fulminant
hemorrhagic rash. After several meningococcemia with high fever,
hours, he became hypotensive and hemorrhagic rash, shock and DIC.
developed DIC.
Weil disease, a severe form of
What is the most likely etiologic leptospirosis, is characterized by a triad of
agent in this patient case? jaundice, renal impairment, and
hemorrhages. None of these are present in
A. Leptospira interrogans the patient.
B. Haemophilus ducreyi
C. Treponema pallidum Option B causes chancroid.
D. Neisseria meningitidis

131
Option C causes syphilis.

Reference:
Riedel, S., Morse, S., Mietzner, T., & Miller, S.
(2019). Jawetz Melnick & Adelbergs Medical
Microbiology. (28th ed., p. 302). McGraw-Hill
Education.

94. Patients with Streptococcus C C. Colon cancer


gallolyticus (formerly S. bovis)
endocarditis are at an increased Patients with Streptococcus gallolyticus
risk for developing which of the (formerly S. bovis) endocarditis are at an
following malignancies? increased risk for developing colon cancer.

A. Esophageal cancer References:


● Le, T., Bhushan, V., & Sochat, M. (2023). First
B. Endometrial cancer Aid for the USMLE Step 1 2022, Thirty Third
C. Colon cancer Edition (33rd ed., p. 135). McGraw-Hill
D. Breast cancer Education.
● Riedel, S., Morse, S., Mietzner, T., & Miller, S.
(2019). Jawetz Melnick & Adelbergs Medical
Microbiology. (28th ed., p. 223). McGraw-Hill
Education.

95. The causative agent of the A A. ssRNA


COVID-19 pandemic is a ___:
The causative agent of the COVID-19
A. ssRNA pandemic is a (+) single-stranded RNA
B. ssDNA virus.
C. dsRNA
D. dsDNA

Reference:
Le, T., Bhushan, V., & Sochat, M. (2023). First Aid for
the USMLE Step 1 2022, Thirty Third Edition (33rd
ed., p. 170). McGraw-Hill Education.

96. Which of the following is the B B. Age


strongest risk factor for severe
COVID-19 illness? Advanced age and medical comorbidities
are the strongest risk factors for severe
A. Sex illness or death from COVID-19.
B. Age
C. Socioeconomic status Reference:
D. All of the above Le, T., Bhushan, V., & Sochat, M. (2023). First Aid for
the USMLE Step 1 2022, Thirty Third Edition (33rd
ed., p. 170). McGraw-Hill Education.

97. The vector for the agent that D D. Tsetse fly


causes African Sleeping Sickness
is ___: Infective trypanosomes of Trypanosoma
brucei are introduced through the bite of a
A. Sandfly tsetse fly.
B. Kissing bug
C. Ixodes tick Sandfly is the vector for Leishmania spp.
D. Tsetse fly
Kissing bug is the vector for Trypanosoma

132
cruzi.

Ixodes deer tick transmits Borrelia


burgdorferi which causes Lyme disease.

Reference:
Riedel, S., Morse, S., Mietzner, T., & Miller, S.
(2019). Jawetz Melnick & Adelbergs Medical
Microbiology. (28th ed., p. 731). McGraw-Hill
Education.

98. Viruses from the major group of B B. ICAM-1


human rhinoviruses use which of
the following receptors? Human rhinoviruses can be divided into
major and minor receptor groups.
A. P antigen
B. ICAM-1 ● Major group viruses: use
C. CCR5 intracellular adhesion molecule-1
D. CD21 (ICAM-1) as receptor
● Minor group viruses: bind
members of the low-density
lipoprotein receptor (LDLR) family

P antigen: Parvovirus B19

CCR5: HIV

CD21: EBV

References:
● Le, T., Bhushan, V., & Sochat, M. (2023). First
Aid for the USMLE Step 1 2022, Thirty Third
Edition (33rd ed., p. 163). McGraw-Hill
Education.
● Riedel, S., Morse, S., Mietzner, T., & Miller, S.
(2019). Jawetz Melnick & Adelbergs Medical
Microbiology. (28th ed., p. 542). McGraw-Hill
Education.

99. Oseltamivir is used as a A A. Inhibiting neuraminidase


treatment for influenza and works
by ____:
Oseltamivir is used as a treatment for
A. Inhibiting neuraminidase influenza and works by inhibiting influenza
B. Inhibiting viral RNA- neuraminidase.
dependent RNA polymerase
C. Acting as a guanosine Reference:
analog Le, T., Bhushan, V., & Sochat, M. (2023). First Aid for
the USMLE Step 1 2022, Thirty Third Edition (33rd
D. All of the above ed., p. 197). McGraw-Hill Education.

100. Chronic carriers of HBV are C C. More than 6 months


patients in whom HBsAg persists
for how long? Chronic carriers of HBV are patients in
whom HBsAg persists for > 6 months.
A. More than 6 weeks

133
B. More than 3 months Reference:
C. More than 6 months Riedel, S., Morse, S., Mietzner, T., & Miller, S.
(2019). Jawetz Melnick & Adelbergs Medical
D. More than 12 months Microbiology. (28th ed., p. 519). McGraw-Hill
Education.

134
PATHOLOGY

135
QUESTION ANSWER EXPLANATION

1. A 30/M presents to the clinic D D. Fusobacterium necrophorum


with sore throat associated with
rigors, neck pain, and neck Lemierre syndrome is an infection of the
swelling. lateral pharyngeal space with septic
thrombophlebitis of the internal jugular vein.
On further examination, there is
involvement of the lateral The diagnosis should be considered in young
pharyngeal space. adult patients with neck pain and swelling with
associated rigors.
A diagnosis of Lemierre
syndrome is highly considered. Prevotella is a common anaerobe found in
What is the most likely cause of the mouth and pharynx.
this disease?
Clostridium spp. may cause abdominal
abscesses, not neck abscesses.
A. Staphylococcus aureus
B. Prevotella spp. Reference:
C. Clostridium spp. Kumar, V., Abbas, A. & Aster, J. (2020). Infectious
diseases. In K. Frank & A.J. McAdam (Eds.), Robbins
D. Fusobacterium and Cotran Pathologic Basis of Diseases (10th ed., p.
necrophorum 329). Amsterdam: Elsevier

2. Lemierre syndrome involves A A. Jugular vein


which of the following neck
structures? Lemierre syndrome is an infection of the
lateral pharyngeal space with septic
A. Jugular vein thrombophlebitis of the internal jugular vein.
B. Carotid artery
C. Sternocleidomastoid Reference:
muscle Kumar, V., Abbas, A. & Aster, J. (2020). Infectious
diseases. In K. Frank & A.J. McAdam (Eds.), Robbins
D. Vagus nerve and Cotran Pathologic Basis of Diseases (10th ed., p.
379). Amsterdam: Elsevier

3. A patient was referred to you D D. Spongiform nodule


for evaluation of a thyroid nodule
that was incidentally discovered Thyroid ultrasound features that are
in annual PE. suggestive of malignancy are the ff:
● Hypoechoic solid nodule with
What finding in the thyroid infiltrative borders
ultrasound suggests that the ● Microcalcifications
nodule is benign?
Thyroid ultrasound features that suggest a
A. Microcalcifications benign nodule:
B. Hypoechoic nodule ● Spongiform nodule
C. Nodule with infiltrative ○ Multiple small internal cystic
borders areas
D. Spongiform nodule ● Simple cysts

(This question was asked in Pathology but I


can’t seem to find the answer in Robbins. But
I found it in Harrison’s.)

Reference:

136
Loscalzo J, & Fauci A, & Kasper D, & Hauser S, &
Longo D, & Jameson J. (2022). Prion diseases. In S.B.
Pruisner & M. Geschwind (Eds.), Harrison's Principles
of Internal Medicine (21st ed., Vol.2, p. 12414). New
York, NY: McGraw Hill.

4. A 40/F woman was brought to A A. Fibroma


the OPD for a 3-month history of
enlarging abdominal girth. On The triad of ovarian fibroma, ascites, and
ultrasound, there was a note of hydrothorax (usually on the right) is consistent
an ovarian mass on the right with the diagnosis of Meigs syndrome.
with associated ascites.
CXR was also done which Sertoli-Leydig cell tumors are clinically
showed hydrothorax on the right. present with masculinization or
defeminization.
What is the most likely nature of
the ovarian mass? Granulosa-cell tumors usually occur in
postmenopausal women and are not
A. Fibroma associated with the other components of the
B. Sertoli-Leydig tumor triad like hydrothorax and ascites.
C. Granulosa cell tumor
D. Krukenberg tumor Krukenberg tumor presents as bilateral
ovarian metastases from gastric carcinoma.
The cancer cells have a signet-ring
appearance.

Reference:
Kumar, V., Abbas, A. & Aster, J. (2020). The female
genital tract. In L.H. Ellenson & E.C. Pirog (Eds.),
Robbins and Cotran Pathologic Basis of Diseases (10th
ed., p. 1027-1029). Amsterdam: Elsevier

5. A patient diagnosed with PTB A A. Neutrophilic pleocytosis


a week ago now presents to the
ER with headache, confusion, CSF findings consistent with TB meningitis:
and acute-onset vomiting. ● Mononuclear pleocytosis (or a mixture
A lumbar puncture was done. of neutrophils and mononuclear cells)
● Increased CSF protein
Which of the following is NOT ● Moderately reduced or normal glucose
consistent with TB meningitis as
a possible cause of his Reference:
neurologic symptoms? Kumar, V., Abbas, A. & Aster, J. (2020). The central
nervous system. In M. Margeta & A. Perry (Eds.),
Robbins and Cotran Pathologic Basis of Diseases (10th
A. Neutrophilic pleocytosis ed., pp. 1263-1264). Amsterdam: Elsevier.
B. Mononuclear pleocytosis
C. Increased CSF protein
D. Normal glucose

6. Neurosyphilis is a C C. Tertiary
manifestation of what stage of
syphilis? The tertiary stage of syphilis included
neurosyphilis.
A. Primary
B. Secondary STAGES of SYPHILIS
C. Tertiary
D. Quaternary

137
Stage Clinical manifestations

Primary Painless chancre (localized)

Secondary ● Maculopapular rash


● Condyloma lata
● Hair loss

Tertiary ● Neurosyphilis
● Gummas
● Aortitis
● Argyll Robertson pupil

Reference:
Kumar, V., Abbas, A. & Aster, J. (2020). The central
nervous system. In M. Margeta & A. Perry (Eds.),
Robbins and Cotran Pathologic Basis of Diseases (10th
ed., p. 1264). Amsterdam: Elsevier

7. Which of the following toxins A. A. Tetanus toxin


can cause spastic paralysis by
blocking GABA release in the Tetanus toxin blocks the release of GABA in
spinal cord? the spinal cord to cause violent, spastic
paralysis.
A. Tetanus toxin
B. Botulinum toxin Botulinum toxin inhibits acetylcholine
C. Alpha toxin release at the neuromuscular junction that
D. Any of the above causes flaccid paralysis (not spastic
paralysis).

Alpha toxin is secreted by C. perfringens and


causes myonecrosis leading to gas gangrene

References:
● Kumar, V., Abbas, A. & Aster, J. (2020). Infectious
diseases. In K. Frank & A.J. McAdam (Eds.),
Robbins and Cotran Pathologic Basis of Diseases
(10th ed., p. 380). Amsterdam: Elsevier
● Le, T., Bhushan, V., & Sochat, M. (2023). First Aid
for the USMLE Step 1 2022, Thirty Third Edition
(33rd ed., p. 136). McGraw-Hill Education.

8. Which of the following is true D D. High protein concentration


of exudates?
Exudate is characterized by the following:
A. Low protein content ● High protein concentration
B. Little cellular material ● Contains cellular debris
C. Low specific gravity Transudate is characterized by the following:
D. High protein ● Low protein content
concentration ● Little or no cellular material
● Low specific gravity

Reference:
Kumar, V., Abbas, A. & Aster, J. (2020). Inflammation
and repair. In Robbins and Cotran Pathologic Basis of

138
Diseases (10th ed., p. 75). Amsterdam: Elsevier

9. Which of the following is an X- A A. Hunter syndrome


linked recessive disorder?
B and C are X-linked dominant diseases.
A. Hunter syndrome
B. Fragile X syndrome X-linked recessive diseases:
C. Alport syndrome ● Bruton agammaglobulinemia
D. All of the above ● Duchenne and Becker muscular
dystrophies
● Fabry disease
● G6PD deficiency
● Hemophilia A and B
● Hunter syndrome
● Lesch-Nyhan syndrome
● Ocular albinism
● Ornithine transcarbamylase deficiency
● Wiskott-Aldrich syndrome

Reference:
Le, T., Bhushan, V., & Sochat, M. (2023). First Aid for
the USMLE Step 1 2022, Thirty Third Edition (33rd ed.,
p. 59). McGraw-Hill Education.

10. Which of the following is A A. It is mostly X-linked recessive


NOT TRUE of Ehlers-Danlos
syndrome (EDS)? The mode of inheritance of EDS type I, II, III,
IV, VIIa, and VIIb is autosomal dominant.
A. It is mostly X-linked
recessive EDS type VI and VIIc are both autosomal
B. The vascular type of EDS recessive.
results from defective
type 3 collagen Reference:
C. Classic EDS presents Kumar, V., Abbas, A. & Aster, J. (2020). Genetic
disorders. In Robbins and Cotran Pathologic Basis of
with skin and joint Diseases (10th ed., p. 150). Amsterdam: Elsevier
hypermobility
D. NOTA

11. Mixed connective tissue C C. Anti-U1 RNP (ribonucleoprotein)


disease is associated with what
autoantibody? Antihistone is for drug-induced lupus

A. Antihistone Anticardiolipin is for SLE and


B. Anticardiolipin antiphospholipid syndrome
C. Anti-U1 RNP
D. Rheumatoid factor RF is for Rheumatoid arthritis

Reference:
Le, T., Bhushan, V., & Sochat, M. (2023). First Aid for
the USMLE Step 1 2022, Thirty Third Edition (33rd ed.,
p. 113). McGraw-Hill Education

12. This disease is characterized B B. MCTD


by clinical features that overlap Polyarteritis nodosa is characterized by

139
with those of polymyositis, necrotizing inflammation of the blood vessel
systemic sclerosis, and SLE: walls.

A. Polyarteritis nodosa IgG4-related disease is characterized by


B. Mixed connective tissue tissue infiltrates with IgG4 antibody-producing
disease plasma cells and T cells, fibrosis, and
C. IgG4-related disease obliterative phlebitis.
D. Scleroderma
Scleroderma is also known as systemic
sclerosis and involves fibrosis of the skin,
GIT, and other tissues.

Reference:
Kumar, V., Abbas, A. & Aster, J. (2020). Diseases of the
immune system. In Robbins and Cotran Pathologic
Basis of Diseases (10th ed., p. 234). Amsterdam:
Elsevier

13. Which of the following can A A. Esophageal strictures


be found in patients with
systemic sclerosis? Barrett esophagus and strictures may result
from LES dysfunction associated with
A. Esophageal strictures systemic sclerosis (SSc).
B. Esophageal
hyperflexibility In SSc, there is also progressive atrophy and
C. Progressive hypertrophy fibrous replacement of the muscularis layer
of the esophagus that is most severe in the esophagus.
D. All of the above
The lower ⅔ of the esophagus may also
develop rubber-hose-like inflexibility.

Reference:
Kumar, V., Abbas, A. & Aster, J. (2020). Diseases of the
immune system. In Robbins and Cotran Pathologic
Basis of Diseases (10th ed., p. 232). Amsterdam:
Elsevier

14. Barrett esophagus is a result B. B. Metaplasia


of the replacement of the
squamous epithelium of the Metaplasia is the replacement of one cell
esophagus by glandular
epithelium. This process is type with another type. For example,
called: squamous epithelium → glandular epithelium
A. Dysplasia in Barrett esophagus.
B. Metaplasia
C. Carcinoma in situ Dysplasia is disordered cell growth
D. Any of the above characterized by pleomorphism with large
hyperchromatic nuclei with a high nuclear-to-
cytoplasmic ratio.

Carcinoma in situ happens when the


dysplasia is severe and involves the full
thickness of the epithelium without
penetrating the basement membrane.

140
Reference:
Le, T., Bhushan, V., & Sochat, M. (2023). First Aid for
the USMLE Step 1 2022, Thirty Third Edition (33rd ed.,
pp. 202, 215). McGraw-Hill Education

15. Which of the following D D. Mesothelioma


cancers is associated with
asbestos? Vinyl chloride - Hepatic angiosarcoma

A. Hepatic angiosarcoma Nickel compounds - oropharyngeal carcinoma


B. Oropharyngeal
carcinoma Cadmium - Prostate carcinoma
C. Prostate carcinoma
D. Mesothelioma Reference:
Kumar, V., Abbas, A. and Aster, J. (2020). Infectious
diseases. In K. Frank & A.J. McAdam (Eds.), Robbins
and Cotran Pathologic Basis of Diseases (10th ed., p.
280). Amsterdam: Elsevier.

16. Majority of the cause of C C. Unknown


congenital anomalies are:
The exact cause of congenital anomalies
A. Due to genetic causes remains unknown in 40-60% of cases.
B. Due to environmental
factors The common known causes are grouped into:
C. Unknown genetic, environmental, and multifactorial.
D. Multifactorial
Reference:
Kumar, V., Abbas, A. and Aster, J. (2020). Infectious
diseases. In K. Frank & A.J. McAdam (Eds.), Robbins
and Cotran Pathologic Basis of Diseases (10th ed., p.
455). Amsterdam: Elsevier

17. A patient presenting with A A. WT1


Wilms tumor, aniridia,
genitourinary anomalies, and This patient has WAGR syndrome: Wilms
intellectual disability has tumor, Aniridia, Genitourinary anomalies, and
deletions in which of the Intellectual disability (formerly called mental
following? Retardation).

A. WT1 WAGR syndrome is associated with WT1


B. WT2 deletion.
C. WT3
D. WT4 Beckwith-Wiedemann syndrome is
associated with WT2 mutation. This is
characterized by Wilms tumor, macroglossia,
organomegaly, hemihyperplasia, and
omphalocele.

Denys-Drash syndrome is associated with


WT1 mutation. This is characterized by Wilms
tumor, diffuse mesangial sclerosis, and
dysgenesis of the gonads.

141
References:
● Le, T., Bhushan, V., & Sochat, M. (2023). First Aid
for the USMLE Step 1 2022, Thirty Third Edition
(33rd ed., p. 626). McGraw-Hill Education
● Kumar, V., Abbas, A. & Aster, J. (2020). Diseases
of infancy and childhood. In A.H. Husain & S.C.
Koo, Robbins and Cotran Pathologic Basis of
Diseases (10th ed., p. 481). Amsterdam: Elsevier

18. Which of the following is the C C. Trisomy 21 (Down syndrome)


most common of the
chromosomal disorders? Down syndrome, aside from being the most
common of the chromosomal disorders, is
A. Trisomy 13 also a major cause of intellectual disability.
B. Trisomy 18
C. Trisomy 21 Trisomy 13 is Patau syndrome
D. Klinefelter syndrome Trisomy 18 is Edwards syndrome

Klinefelter syndrome (47, XXY) – One of the


most common causes of male hypogonadism.

Reference:
Kumar, V., Abbas, A. & Aster, J. (2020). Genetic
disorders. In Robbins and Cotran Pathologic Basis of
Diseases (10th ed., p. 166). Amsterdam: Elsevier

19. Which of the following does C C. Klinefelter syndrome


not cause intellectual disability?
The cognitive abilities of patients with
A. Prader-Willi syndrome Klinefelter syndrome range from average to
B. Angelman syndrome below average only.
C. Klinefelter syndrome
D. None of the above Prader-Willi syndrome is caused by the
deletion of the paternally derived
chromosome 15. It is characterized by
intellectual disability, hypotonia, short stature,
obesity, hyperphagia, small hands and feet,
and hypogonadism.

Angelman syndrome is caused by the


deletion of the maternally derived
chromosome 15. This also has an intellectual
disability. It also presents with microcephaly,
ataxic gait, seizures, and inappropriate
laughter.

Reference:
Kumar, V., Abbas, A. & Aster, J. (2020). Genetic
disorders. In Robbins and Cotran Pathologic Basis of
Diseases (10th ed., pp. 171-178). Amsterdam: Elsevier

20. Wiskott-Aldrich syndrome is A A. Immunodeficiency disorder


classified as a/an ___:
Wiskott-Aldrich syndrome is characterized by
A. Immunodeficiency thrombocytopenia, eczema, and recurrent
disorder infections.

142
B. Metabolic disorder
C. Blood disorder Mnemonic:
D. Musculoskeletal disorder WATER: Wiskott-Aldrich:
Thrombocytopenia, Eczema,
Recurrent (pyogenic)
infections

References:
● Kumar, V., Abbas, A. & Aster, J. (2020). Diseases
of the immune system. In Robbins and Cotran
Pathologic Basis of Diseases (10th ed., p. 246).
Amsterdam: Elsevier
● Le, T., Bhushan, V., & Sochat, M. (2023). First Aid
for the USMLE Step 1 2022, Thirty Third Edition
(33rd ed., p. 115). McGraw-Hill Education.

21. Which of the following B B. Subdural hematoma


causes of intracranial
hemorrhage is associated with Epidural hematoma
the rupture of bridging veins? ● Rupture of the middle meningeal
artery commonly from skull fracture
A. Epidural hematoma involving the pterion (thinnest region
B. Subdural hematoma of the lateral skull).
C. Subarachnoid ● On CT: a biconvex, hyperdense
hemorrhage collection of blood not crossing suture
D. Aneurysmal rupture lines.

Subdural hematoma
● Rupture of bridging veins
● On CT: crescent-shaped hemorrhage
that crosses suture lines.

References:
● Le, T., Bhushan, V., & Sochat, M. (2023). First Aid
for the USMLE Step 1 2022, Thirty Third Edition
(33rd ed., p. 530). McGraw-Hill Education.
● Kumar, V., Abbas, A. & Aster, J. (2020). The
central nervous system. In M. Margeta & A. Perry
(Eds.), Robbins and Cotran Pathologic Basis of
Diseases (10th ed., pp. 1251). Amsterdam:
Elsevier.

22. Which of the following B B. Liquefactive necrosis


morphological changes occurs in
the brain of a patient who just Hypoxic cell death in the CNS results in
suffered an acute ischemic liquefactive necrosis.
stroke?
Reference:
A. Coagulative necrosis Kumar, V., Abbas, A. & Aster, J. (2020). Cell injury, cell
death, and adaptations. In S.A. Oakes (Ed.), Robbins
B. Liquefactive necrosis and Cotran Pathologic Basis of Diseases (10th ed., p.
C. Gangrenous necrosis 40). Amsterdam: Elsevier
D. Caseous necrosis

23. Which of the following D D. Fat necrosis


morphological changes occurs in
the pancreas of a patient who is Fat necrosis results from the release of
affected with acute pancreatitis? activated pancreatic lipases into the pancreas

143
and peritoneal cavity.
A. Coagulative necrosis
B. Liquefactive necrosis Reference:
C. Gangrenous necrosis Kumar, V., Abbas, A. & Aster, J. (2020). Cell injury, cell
death, and adaptations. In S.A. Oakes (Ed.), Robbins
D. Fat necrosis and Cotran Pathologic Basis of Diseases (10th ed., p.
41). Amsterdam: Elsevier

24. Which of the following C C. Mitochondrial or intrinsic pathway


pathways of apoptosis is
triggered by DNA damage? The mitochondrial pathway or intrinsic
pathway is triggered by DNA damage, loss of
A. Death receptor pathway survival signals, and accumulation of
B. Extrinsic pathway misfolded proteins (ER stress).
C. Mitochondrial pathway
D. All of the above Death receptor (extrinsic pathway) is initiated
by the engagement of death receptors.

Reference:
Kumar, V., Abbas, A. & Aster, J. (2020). Cell injury, cell
death, and adaptations. In S.A. Oakes (Ed.), Robbins
and Cotran Pathologic Basis of Diseases (10th ed., p.
46). Amsterdam: Elsevier

25. Which of the following small A A. Granulomatosis with polyangiitis


vessel vasculitis presents with
the triad of focal necrotizing Granulomatosis with polyangiitis
vasculitis, necrotizing ● Previously called Wegener
granulomas in the lung, and granulomatosis
focal necrotizing ● Characterized by the triad of:
glomerulonephritis? ○ Necrotizing vasculitis
○ Necrotizing granulomas in the lung or
A. Granulomatosis with the URT
polyangiitis ○ Focal necrotizing, often crescentic,
B. Churg-Strauss syndrome glomerulonephritis
C. Microscopic polyangiitis ● Associated with PR3-ANCA/c-ANCA.
D. Buerger disease
Eosinophilic granulomatosis with
polyangiitis
● Formerly called Churg-Strauss syndrome
● It presents with asthma, sinusitis, skin
nodules/purpura, and peripheral
neuropathy
● Associated with MPO-ANCA/p-ANCA.

Microscopic polyangiitis is also associated


with MPO-ANCA/p-ANCA.

Buerger disease occurs exclusively in heavy


smokers.

Reference:
Kumar, V., Abbas, A. & Aster, J. (2020). Blood vessels.
In R.N. Mitchell & M.K. Halusha (Eds.), Robbins and
Cotran Pathologic Basis of Diseases (10th ed., p. 515).
Amsterdam: Elsevier

144
26. Histamine is an inflammatory B B. Platelets
mediator that promotes
vasodilation and increased Sources of histamine include:
vascular permeability. What cell ● Mast cells
type produces this substance? ● Basophils
● Platelets
A. Macrophages
B. Platelets Reference:
C. Endothelial cells Kumar, V., Abbas, A. & Aster, J. (2020). Inflammation
and repair. In Robbins and Cotran Pathologic Basis of
D. Activated macrophages Diseases (10th ed., p. 85). Amsterdam: Elsevier

27. In Carcinoid heart disease, A A. Right heart valves


which of the following is primarily
affected? Since the right side of the heart is the first to
be bathed by the mediators released by GI
A. Right heart valves carcinoid tumors, the endocardium and valves
B. Left heart valves of the right heart are primarily affected.
C. Pulmonary veins The left side of the heart is protected due to
D. Left atrium the degradation of mediators by the
pulmonary vascular bed.
Reference:
Kumar, V., Abbas, A. & Aster, J. (2020). The heart. In
R.N. Mitchell & A.J. Connolly (Eds.), Robbins and
Cotran Pathologic Basis of Diseases (10th ed., p. 565).
Amsterdam: Elsevier.

28. A female patient comes to A A. Carcinoid syndrome


the clinic complaining of
episodes of cutaneous flushing, Carcinoid syndrome consists of the
bronchospasm, diarrhea, and an following signs and symptoms:
incidental finding of a heart ● cutaneous flushing
murmur. She reports not having ● bronchospasm
allergies to any food or drugs in ● diarrhea
the past. ● heart murmur

Which of the following is the References:


most likely diagnosis? ● Le, T., Bhushan, V., & Sochat, M. (2023). First Aid
for the USMLE Step 1 2022, Thirty Third Edition
(33rd ed., p. 705). McGraw-Hill Education.
A. Carcinoid syndrome ● Kumar, V., Abbas, A. & Aster, J. (2020). The heart.
B. Plummer-Vinson In R.N. Mitchell & A.J. Connolly (Eds.), Robbins
syndrome and Cotran Pathologic Basis of Diseases (10th ed.,
C. Bronchial asthma pp. 565). Amsterdam: Elsevier.
D. Anaphylaxis

29. What is the most common A A. Primary hyperparathyroidism


manifestation of MEN-1?
Multiple endocrine neoplasia type 1 (MEN-1)
A. Primary consists of abnormalities in the 3P’s:
hyperparathyroidism ● Pancreas - Pancreatic endocrine
B. Pancreatic tumor tumor (Zollinger-Elllison, insulinomas)
C. Prolactinoma ● Parathyroid - Primary
D. Zollinger-Ellison hyperparathyroidism
syndrome ● Pituitary - anterior pituitary tumor

145
(most frequently a prolactinoma)
Reference:
Kumar, V., Abbas, A. & Aster, J. (2020). The endocrine
system. In A. Maitra (Ed.), Robbins and Cotran
Pathologic Basis of Diseases (10th ed., pp. 1129).
Amsterdam: Elsevier.

30. Which of the following is C C. MEN 2B


characterized by medullary
thyroid carcinoma, MEN1
pheochromocytoma, and ● Primary tumors (prolactin or GH)
mucosal neuromas? ● Pancreatic endocrine tumors
o Zollinger-Ellison syndrome
A. MEN1 o Insulinomas
B. MEN2A o VIPomas
C. MEN2B o Glucagonomas (rare)
D. MEN3 ● Parathyroid adenomas
● Associated mutation of MEN1
o Tumor suppressor gene
o Codes for menin
o Chromosome 1 angiofibromas
o Collagenomas
o Meningiomas
MEN2A
● Parathyroid hyperplasia
● Medullary thyroid carcinoma –
neoplasm of parafollicular C cells,
secretes calcitonin; prophylactic
thyroidectomy required
● Pheochromocytoma – secretes
catecholamines
● Associated with mutation in RET
o Protooncogene
o Codes for receptor tyrosine
kinase
o Chromosome 10
MEN2B
● Medullary thyroid carcinoma
● Pheochromocytoma
● Mucosal neuromas A
(oral/intestinal ganglioneuromatosis)
● Associated with marfanoid habitus;
mutation in RET gene
Reference:
Le, T., Bhushan, V., & Sochat, M. (2023). First Aid for
the USMLE Step 1 2022, Thirty Third Edition (33rd ed.,
p. 356). McGraw-Hill Education.

31. Which of the following is D. D. Composed of signet ring cells


TRUE of diffuse gastric cancer?
A, B, and C are found in intestinal-type gastric
A. Associated with H. pylori adenocarcinoma
B. Commonly found in the
lesser curvature

146
C. Has raised margins TYPES OF
D. Composed of signet-ring GASTRIC ADENOCARCINOMA
cells (Based on morphology)

Intestinal Diffuse

Associated with H. Not associated with


pylori H. pylori

● Dietary E-cadherin mutation


nitrosamines (most cases)
(smoked foods
common in East
Asian countries)
● Tobacco smoking
● Achlorhydria
● Chronic gastritis

● Commonly in the ● Signet ring cells


lesser curvature (mucin-filled cells
● Looks like an with peripheral
ulcer with raised nuclei
margins ● Stomach wall is
grossly thickened
and leathery
(linitis plastica)

References:
● Le, T., Bhushan, V., & Sochat, M. (2023). First Aid
for the USMLE Step 1 2022, Thirty Third Edition
(33rd ed., p. 386). McGraw-Hill Education.
● Kumar, V., Abbas, A. & Aster, J. (2020). The
gastrointestinal tract. In Robbins and Cotran
Pathologic Basis of Diseases (10th ed., pp. 775-
776). Amsterdam: Elsevier.

32. Which of the following occurs C C. Miliary pulmonary disease


when a patient develops
individual lesions that are either Miliary pulmonary disease - presence of
microscopic or small, visible individual lesions that are either microscopic
(2mm) foci of yellow-white or small, visible (2mm) foci of yellow-white
consolidation scattered through consolidation scattered through the lung
the lung parenchyma? parenchyma (miliary resembling millet seeds)

A. Progressive pulmonary Primary tuberculosis - may have a Ghon


tuberculosis focus which is a 1 to 1.5cm area of gray-white
B. Secondary tuberculosis inflammation with consolidation
C. Miliary pulmonary
disease Secondary tuberculosis - initial lesion is a
D. Primary tuberculosis small focus of consolidation less than 2 cm in
diameter and within 1-2cm of the apical
pleura

Reference:
Kumar, V., Abbas, A. & Aster, J. (2020). Infectious
diseases. In K. Frank & A.J. McAdam (Eds.), Robbins
and Cotran Pathologic Basis of Diseases (10th ed., p.

147
371-373). Amsterdam: Elsevier

33. A Ghon focus can usually be D D. Primary tuberculosis


found in which of the following?
Miliary pulmonary disease - presence of
A. Progressive pulmonary individual lesions that are either microscopic
tuberculosis or small, visible (2mm) foci of yellow-white
B. Secondary tuberculosis consolidation scattered through the lung
C. Miliary pulmonary parenchyma (miliary resembling millet seeds)
disease
D. Primary tuberculosis Primary tuberculosis - may have a Ghon
focus which is a 1 to 1.5cm area of gray-white
inflammation with consolidation

Secondary tuberculosis - initial lesion is a


small focus of consolidation less than 2 cm in
diameter and within 1-2cm of the apical
pleura

Reference:
Kumar, V., Abbas, A. & Aster, J. (2020). Infectious
diseases. In K. Frank & A.J. McAdam (Eds.), Robbins
and Cotran Pathologic Basis of Diseases (10th ed., p.
371-373). Amsterdam: Elsevier

34. What is the most common D D. GIST


mesenchymal tumor of the
abdomen? GIST is the most common mesenchymal
tumor of the abdomen and most often occurs
A. Zollinger-Ellison in the stomach. It develops from Cajal or
syndrome pacemaker cells.
B. Carcinoid tumors
C. MALTomas The most useful diagnostic marker is KIT
D. Gastrointestinal stromal (CD117)
tumor
Reference:
Kumar, V., Abbas, A. & Aster, J. (2020). The
gastrointestinal tract. In Robbins and Cotran Pathologic
Basis of Diseases (10th ed., p. 780). Amsterdam:
Elsevier.

35. The majority of deaths in D D. Aortic dissection


Marfan syndrome are caused by:
Aortic dissection account for the majority of
A. Skeletal abnormalities deaths in Marfan syndrome followed by
B. Ocular changes cardiac failure.
C. Mitral valve prolapse
D. Aortic dissection Skeletal abnormalities are the most striking
feature.

Reference:
Kumar, V., Abbas, A. & Aster, J. (2020). Genetic
disorders. In Robbins and Cotran Pathologic Basis of
Diseases (10th ed., p. 149). Amsterdam: Elsevier

36. Which of the following A A. MVP


conditions that occur in Marfan

148
syndrome patients is described MVP is described histologically as
histologically as myxomatous myxomatous degeneration of the spongiosa
degeneration of the spongiosa layer of the mitral leaflet.
layer?
Reference:
A. Mitral valve prolapse Kumar, V., Abbas, A. & Aster, J. (2020). The heart. In
R.N. Mitchell & A.J. Connolly (Eds.), Robbins and
B. Ectopia lentis Cotran Pathologic Basis of Diseases (10th ed., p. 559).
C. Ascending aortic dilation Amsterdam: Elsevier
D. Joint hyperlaxity

37. A patient was referred to you A. A. Alport syndrome


for symptoms of hematuria
which eventually progressed to Alport syndrome is a mutation in type IV
chronic kidney disease. This is collagen. It presents with hematuria with
associated with deafness and progression to chronic kidney disease, nerve
lens dislocation. deafness, lens dislocation, posterior
cataracts, and corneal dystrophy.
What is the most likely Most commonly x-linked dominant.
diagnosis?
Thin basement membrane nephropathy is
A. Alport syndrome differentiated from Alport by the absence of
B. Marfan syndrome hearing loss, ocular abnormalities, and a
C. Homocystinuria family history of renal failure.
D. Thin basement
membrane nephropathy Reference:
Kumar, V., Abbas, A. & Aster, J. (2020). The kidney. In
A. Chang & Z.G. Laszik (Eds.), Robbins and Cotran
Pathologic Basis of Diseases (10th ed., p. 922).
Amsterdam: Elsevier.

38. A young patient with B B. Panacinar emphysema


emphysema was diagnosed with TYPES OF EMPHYSEMA
a1-antitrypsin deficiency.
On chest CT scan, there is more Centriacinar Panacinar
severe involvement of the bases.
Which of the following is the ● Affects respiratory ● Affects respiratory
most likely type present? bronchioles while bronchioles and
sparing distal alveoli
A. Centriacinar emphysema alveoli ● Associated with α1-
B. Panacinar emphysema ● Associated with antitrypsin
C. Distal acinar emphysema tobacco smoking deficiency
D. Any of the above ● Frequently in ● Frequently in lower
upper lobes lobes

MNEMONIC: smoke rises up (centriacinar)


Reference:
Kumar, V., Abbas, A. & Aster, J. (2020). The lung. In A.
Chang & Z.G. Laszik (Eds.), Robbins and Cotran
Pathologic Basis of Diseases (10th ed., pp. 679, 698).
Amsterdam: Elsevier.

39. A 30/F presented to the clinic C C. Type III


with nonscarring alopecia, malar
rash, oral ulcers, joint pains, and This is most likely a case of SLE – Type III
increasing creatinine levels. ANA hypersensitivity.

149
was (+).
TYPES OF HYPERSENSITIVITY REACTIONS
What type of hypersensitivity is
present in this patient? Type Mechanism Examples

A. Type I Type Anaphylactic ● Anaphylaxis (eg.,


B. Type II I and atopic food, drug, or bee
C. Type III sting allergies)
D. Type IV ● Allergic asthma

Type Antibodies bind ● Autoimmune


II to cell-surface hemolytic anemia
antigens or ECM (including drug-
leading to cellular
induced form)
inflammation/
dysfunction/ ● Immune
destruction. thrombocytopenia
● Transfusion
reactions
● Hemolytic disease
of the newborn
● Goodpasture
syndrome
● Rheumatic fever
● Hyperacute
transplant rejection
● Myasthenia gravis
● Graves’ disease
● Pemphigus vulgaris

Type Immune complex ● SLE


III ● Rheumatoid
(antigen-antibody
arthritis
complexes) → ● Reactive arthritis
activate ● Polyarteritis nodosa
● Poststreptococcal
complement
glomerulonephritis
● IgA vasculitis

Type T-cell mediated. ● Contact dermatitis


IV No antibodies are (e.g., poison ivy,
involved in the nickel allergy)
response. ● Drug reaction with
eosinophilia and
systemic symptoms
(DRESS)
● Graft-versus-host
disease

Reference:
Le, T., Bhushan, V., & Sochat, M. (2023). First Aid for
the USMLE Step 1 2022, Thirty Third Edition (33rd ed.,
pp. 110-111). McGraw-Hill Education.

40. The use of PPD skin test to D D. Type IV


detect latent tuberculosis is
classified as what type of TYPES OF HYPERSENSITIVITY REACTIONS
hypersensitivity?

150
Type Mechanism Examples
A. Type I
B. Type II Type Anaphylactic and ● Anaphylaxis (eg.,
C. Type III I atopic food, drug, or bee
D. Type IV sting allergies)
● Allergic asthma

Type Antibodies bind ● Autoimmune


II to cell-surface hemolytic anemia
antigens or ECM (including drug-
leading to cellular
induced form)
inflammation/
dysfunction/ ● Immune
destruction. thrombocytopenia
● Transfusion
reactions
● Hemolytic disease
of the newborn
● Goodpasture
syndrome
● Rheumatic fever
● Hyperacute
transplant rejection
● Myasthenia gravis
● Graves’ disease
● Pemphigus vulgaris

Type Immune complex ● SLE


III ● Rheumatoid
(antigen-antibody
arthritis
complexes) → ● Reactive arthritis
activate ● Polyarteritis nodosa
● Poststreptococcal
complement
glomerulonephritis
● IgA vasculitis

Type T-cell mediated. ● Contact dermatitis


IV No antibodies are (e.g., poison ivy,
involved in the nickel allergy)
response.
● Drug reaction with
eosinophilia and
systemic symptoms
(DRESS)
● Graft-versus-host
disease
Reference:
Le, T., Bhushan, V., & Sochat, M. (2023). First Aid for
the USMLE Step 1 2022, Thirty Third Edition (33rd ed.,
pp. 110-111). McGraw-Hill Education.

151
41. Which of the following D. D. Cystic fibrosis
diseases follow a pattern of
autosomal recessive Autosomal recessive diseases:
inheritance? ● Oculocutaneous albinism
● Phenylketonuria
A. Huntington disease ● Cystic fibrosis
B. Multiple endocrine ● Sickle cell disease
neoplasia ● Wilson disease
C. Tuberous sclerosis ● Sphingolipidoses (except Fabry disease)
D. Cystic fibrosis ● Hemochromatosis
● Glycogen storage diseases
● Thalassemia
● Mucopolysaccharidoses
(except Hunter syndrome)
● Friedreich ataxia
● Kartagener syndrome
● ARPKD

MNEMONIC
Oh, please! Can students who score high
grades tell me features of the kidney disorder
Autosomal Recessive Polycystic Kidney
Disease?

Reference:
Le, T., Bhushan, V., & Sochat, M. (2023). First Aid for
the USMLE Step 1 2022, Thirty Third Edition (33rd ed.,
p. 58). McGraw-Hill Education.

42. Sickle cell trait (in African A D. Malaria


populations) is protective against
___: Patients with sickle cell have resistance
specifically against falciparum malaria.
A. Schistosomiasis
B. Lyme disease References:
C. Dengue ● Le, T., Bhushan, V., & Sochat, M. (2023). First Aid
for the USMLE Step 1 2022, Thirty Third Edition
D. Malaria (33rd ed., p. 428). McGraw-Hill Education.
● Kumar, V., Abbas, A. & Aster, J. (2020). Red blood
cell and bleeding disorders. In Robbins and Cotran
Pathologic Basis of Diseases (10th ed., p. 641).
Amsterdam: Elsevier.

43. Which of the following is A. A. Low ferritin


TRUE in the diagnosis of iron-
deficiency anemia? In IDA, serum iron and ferritin are low and the
TIBC is high.
A. Low ferritin
B. Low TIBC Reference:
C. High ferritin Kumar, V., Abbas, A. & Aster, J. (2020). Red blood cell
and bleeding disorders. In Robbins and Cotran
D. High iron Pathologic Basis of Diseases (10th ed., p. 658).
Amsterdam: Elsevier.

44. A patient with leukemia was A A. Mucormycosis


brought to the clinic due to
symptoms of headache, and Mucormycosis usually occurs in

152
facial pain. On examination, immunocompromised patients such as this
there’s a note of a black, patient who has leukemia and most likely also
necrotic eschar on the face. decreased neutrophil count.

Microscopy showed: The right-angle branching with nonseptate


Nonseptate hyphae with hyphae is characteristic of mucormycetes.
frequent right-angle branching
In contrast, Aspergillus has septate filaments
What is the most likely with thick branching at acute angles
diagnosis? (45degrees).

A. Mucormycosis References:
B. Aspergillosis ● Le, T., Bhushan, V., & Sochat, M. (2023). First Aid
for the USMLE Step 1 2022, Thirty Third Edition
C. Malaria (33rd ed., p. 150). McGraw-Hill Education.
D. Pneumocystis ● Kumar, V., Abbas, A. & Aster, J. (2020). Infectious
diseases. In K. Frank & A.J. McAdam (Eds.),
Robbins and Cotran Pathologic Basis of Diseases
(10th ed., pp. 387-388). Amsterdam: Elsevier.

45. A 45/F came to your clinic for B B. Aspergillosis


a 1-month history of fever,
cough, and pleuritic chest pain Aspergillus has septate filaments with thick
which are now associated with branching at acute angles (45 degrees).
new-onset hemoptysis. On
further history, she had Mucormycosis usually occurs in
tuberculosis 3 years ago. immunocompromised patients such as this
patient who has leukemia and most likely also
On sputum microscopy: decreased neutrophil count. It is
(+) note of septate filaments that characterized by right-angle branching with
are 5-10 um thick and are nonseptate hyphae.
branching at acute angles
What is the most likely References:
diagnosis? ● Le, T., Bhushan, V., & Sochat, M. (2023). First Aid
for the USMLE Step 1 2022, Thirty Third Edition
(33rd ed., p. 150). McGraw-Hill Education.
A. Mucormycosis ● Kumar, V., Abbas, A. & Aster, J. (2020). Infectious
B. Aspergillosis diseases. In K. Frank & A.J. McAdam (Eds.),
C. Malaria Robbins and Cotran Pathologic Basis of Diseases
D. Pneumocystis (10th ed., pp. 387-388). Amsterdam: Elsevier.

46. Which of the following A. A. Haemophilus ducreyi


causes chancroid?
Haemophilus influenzae causes otitis
A. Haemophilus ducreyi media, bronchitis, and conjunctivitis. Not
B. Haemophilus influenzae chancroid.
C. Treponema pallidum Treponema pallidum causes syphilis.
D. Yersinia pestis
Yersinia pestis causes the plague.

Reference:
Kumar, V., Abbas, A. & Aster, J. (2020). Infectious
diseases. In K. Frank & A.J. McAdam (Eds.), Robbins
and Cotran Pathologic Basis of Diseases (10th ed., pp.

153
366-367). Amsterdam: Elsevier.

47. A 25/M heavy smoker C C. Goodpasture syndome


presents to the ER with fever,
DOB, and hemoptysis. His Goodpasture syndrome may present in
mother also stated that his urine patients in their 20s.
looks like blood.
What is the most likely Antibodies form and destroy the basement
diagnosis? membrane of pulmonary alveoli and renal
glomeruli leading to signs and symptoms of
A. Thin basement necrotizing hemorrhagic pneumonitis
membrane nephropathy (hemoptysis) and rapidly progressive
B. Alport syndrome glomerulonephritis (hematuria).
C. Goodpasture syndrome
D. IgA nephropathy Reference:
Kumar, V., Abbas, A. & Aster, J. (2020). The lung. In A.
Chang & Z.G. Laszik (Eds.), Robbins and Cotran
Pathologic Basis of Diseases (10th ed., pp. 704-705).
Amsterdam: Elsevier.

48. A 25/M heavy smoker C C. Linear deposits of immunoglobulins


presents to the ER with fever,
DOB, and hemoptysis. His In patients with Goodpasture syndrome, linear
mother also stated that his urine deposits of immunoglobulins and complement
looks like blood. are seen in the glomerular basement
membrane through immunofluorescence
Which of the following will most studies.
likely be seen in the
immunofluorescence studies of Reference:
the glomerular basement Kumar, V., Abbas, A. & Aster, J. (2020). The lung. In A.
Chang & Z.G. Laszik (Eds.), Robbins and Cotran
membrane of this patient? Pathologic Basis of Diseases (10th ed., pp. 704-705).
Amsterdam: Elsevier.
A. Wire loop deposits
B. Lumpy bumpy
appearance
C. Linear deposits of
immunoglobulins
D. All of the above

49. What is the most common C C. Aseptic meningitis


extrasalivary complication of
mumps? The most common extrasalivary complication
of mumps is aseptic meningitis.
A. Mumps orchitis
B. Mumps encephalitis Reference:
C. Aseptic meningitis Kumar, V., Abbas, A. & Aster, J. (2020). Infectious
diseases. In K. Frank & A.J. McAdam (Eds.), Robbins
D. Septic meningitis and Cotran Pathologic Basis of Diseases (10th ed., p.
351). Amsterdam: Elsevier.

50. In a 3-day-old neonate, what C C. Group B Streptococcus


is the most common cause of
early-onset bacterial meningitis? The most common cause of early-onset (first
7 days of life) sepsis and early-onset bacterial
A. E. coli meningitis is Group B Streptococcus.
B. Listeria

154
C. Group B Streptococcus Reference:
D. Neiserria meningitidis Kumar, V., Abbas, A. & Aster, J. (2020). Infectious
diseases. In K. Frank & A.J. McAdam (Eds.), Robbins
and Cotran Pathologic Basis of Diseases (10th ed., p.
462). Amsterdam: Elsevier.

51. Which of the following D D. None of the above


clinical features is NOT
associated with Trisomy 21? Atrioventricular septal defects are the most
common forms of congenital heart disease in
A. Atrioventricular septal Down syndrome.
defect
B. Acute lymphoblastic ALL and umbilical hernia are both associated
leukemia with Down syndrome.
C. Umbilical hernia
D. None of the above Reference:
Kumar, V., Abbas, A. & Aster, J. (2020). Genetic
disorders. In Robbins and Cotran Pathologic Basis of
Diseases (10th ed., p. 167). Amsterdam: Elsevier

52. Which of the following is D D. Maternal age


known to have a strong influence
on the incidence of Trisomy 21? The incidence of Trisomy 21 is influenced by
maternal age.
A. Vinyl chloride exposure
B. Lead exposure Vinyl chloride exposure predisposes to
C. Smoking hepatic angiosarcoma.
D. Maternal age
Lead exposure in children may lead to mental
deterioration.

Reference:
Kumar, V., Abbas, A. & Aster, J. (2020). Genetic
disorders. In Robbins and Cotran Pathologic Basis of
Diseases (10th ed., p. 166). Amsterdam: Elsevier

53. Which of the following can D D. Toxic megacolon


be found in patients with
Ulcerative colitis? D. Toxic megacolon is associated with UC

A. Noncaseating granuloma A, B, and C are found in Crohn's disease.


B. Transmural inflammation
C. Skip lesions Reference:
D. Toxic megacolon Kumar, V., Abbas, A. & Aster, J. (2020). The
gastrointestinal tract. In Robbins and Cotran Pathologic
Basis of Diseases (10th ed., pp. 800-802). Amsterdam:
Elsevier.

54. A 30/M consults the clinic for A A. IgA nephropathy


recurrent episodes of blood in
the urine whenever he IgA nephropathy commonly presents as
experiences an upper respiratory hematuria that occurs concurrently with upper
tract infection. respiratory tract infections.

Which of the following is the Reference:


most likely diagnosis? Kumar, V., Abbas, A. & Aster, J. (2020). The kidney. In
A. Chang & Z.G. Laszik (Eds.), Robbins and Cotran
Pathologic Basis of Diseases (10th ed., pp. 920-921).

155
A. IgA nephropathy Amsterdam: Elsevier.
B. Alport syndrome
C. Thin basement
membrane nephropathy
D. Goodpasture syndrome

55. What is characteristically C C. Mesangial deposition of IgA


found in the immunofluorescent
studies of patients in the above Mesangial deposition of IgA is the
case? characteristic finding in immunofluorescence
studies of patients with IgA nephropathy of
A. Linear deposition of IgG Berger’s disease
B. Lumpy bumpy deposits
C. Mesangial deposition of Linear deposits of immunoglobulin and
IgA complement are found in Goodpasture
D. Thinning of the basement syndrome.
membrane
Thinning of the GBM is found in Thin
basement membrane nephropathy.

Reference:
Kumar, V., Abbas, A. & Aster, J. (2020). The kidney. In
A. Chang & Z.G. Laszik (Eds.), Robbins and Cotran
Pathologic Basis of Diseases (10th ed., p. 921).
Amsterdam: Elsevier.

56. What is the most common A A. Nodular sclerosis


type of Hodgkin lymphoma that
is characterized by the Nodular sclerosis is the most common
deposition of collagen in bands subtype of Hodgkin lymphoma and is
that divide the lymph nodes into characterized by the deposition of collagen in
circumscribed nodules? bands that divide the lymph nodes into
circumscribed nodules.
A. Nodular sclerosis
B. Mixed-cellularity It is usually EBV (-).
C. Lymphocyte-rich
D. Lymphocyte depletion Reference:
Kumar, V., Abbas, A. & Aster, J. (2020). Diseases of
white blood cells, lymph nodes, spleen, and thymus. In
A. Chang & Z.G. Laszik (Eds.), Robbins and Cotran
Pathologic Basis of Diseases (10th ed., pp. 612-613).
Amsterdam: Elsevier.

57. Which of the following D D. Lymphocyte depletion


subtypes of Hodgkin lymphoma
has Reed-Sternberg cells that The least common subtype of Hodgkin
are EBV-infected 90% of the lymphoma is lymphocyte depletion and is
time? associated with Reed-Sternberg cells that are
EBV-infected 90% of the time.
A. Nodular sclerosis
B. Mixed-cellularity Reference:
C. Lymphocyte-rich Kumar, V., Abbas, A. & Aster, J. (2020). Diseases of
white blood cells, lymph nodes, spleen, and thymus. In
D. Lymphocyte depletion A. Chang & Z.G. Laszik (Eds.), Robbins and Cotran
Pathologic Basis of Diseases (10th ed., p. 614).
Amsterdam: Elsevier.

156
58. Which of the following is A A. 46, XX karyotype
found in a complete mole?
The rest of the options are associated with
A. 46, XX karyotype partial moles.
B. 69, XXY karyotype
C. Fetal tissues may be Reference:
present Kumar, V., Abbas, A. & Aster, J. (2020). The female
genital tract. In L.H. Ellenson & E.C. Pirog (Eds.),
D. Not associated with Robbins and Cotran Pathologic Basis of Diseases (10th
choriocarcinoma ed., p. 1103). Amsterdam: Elsevier.

59. A 70/M was brought to the C C. Mass in the distal colon


clinic because of a 3-month
history of constipation and Napkin-ring or apple-core constrictions are
abdominal cramping. usually found in the distal colon and are
suggestive of colonic malignancy.
On abdominal x-ray, there was a
note of “napkin-ring” Masses in the proximal colon rarely cause
constrictions. obstruction and usually grow as polypoid,
exophytic masses.
What is the most likely
diagnosis? Reference:
Kumar, V., Abbas, A. & Aster, J. (2020). The
gastrointestinal tract. In Robbins and Cotran Pathologic
A. Sigmoid volvulus Basis of Diseases (10th ed., p. 816). Amsterdam:
B. Mass in the ascending Elsevier.
colon
C. Mass in the distal colon
D. Intussusception

60. In patients with colorectal B B. Depth of invasion


carcinoma, which of the
following is considered to be the Aside from the depth of invasion, the
most important prognostic presence of lymph node metastases is also
factor? one of the two most important prognostic
factors in colorectal carcinoma.
A. CEA levels
B. Depth of invasion Reference:
C. Age Kumar, V., Abbas, A. & Aster, J. (2020). The
gastrointestinal tract. In Robbins and Cotran Pathologic
D. Comorbidities Basis of Diseases (10th ed., p. 816). Amsterdam:
Elsevier.

61. A 2-year-old patient was A A. Schiller-Duval bodies


referred to you for co-
management of an endodermal The most common testicular tumor in infants
sinus tumor. and children up to 3 years old is an
endodermal sinus tumor (aka yolk sac tumor).
Which of the following may be
seen histologically in these kinds Histologically, Schiller-Duval bodies
of tumors? (structures resembling endodermal sinuses),
may be seen.
A. Schiller-Duval bodies
B. Call-Exner bodies Call-Exner bodies are found in granulosa
C. Reinke crystalloids cell tumors.
D. None of the above
Reinke crystalloids are found in pure Leydig

157
cell tumors (Hilus cell tumors).

Reference:
Kumar, V., Abbas, A. & Aster, J. (2020).The lower
urinary tract and male genital system. In G.J. Netto &
M.B. Amin, Robbins and Cotran Pathologic Basis of
Diseases (10th ed., pp. 972, 1026, 1028). Amsterdam:
Elsevier.

62. Which of the following neck B B. Thyroglossal duct cyst


conditions is described as
arising from incomplete descent Branchial cysts originate from the 2nd
of the thyroid analog from the branchial pouch.
foramen cecum? Paraganglioma comes from neuroendocrine
cells associated with the sympathetic and
A. Branchial cysts parasympathetic NS.
B. Thyroglossal duct cyst
C. Paraganglioma Reference:
D. Mucocele Kumar, V., Abbas, A. & Aster, J. (2020).Head and neck.
In M.W. Lingen & N.A. Cipriani, Robbins and Cotran
Pathologic Basis of Diseases (10th ed., p. 746).
Amsterdam: Elsevier.

63. Cancer of what organ is B B. Pancreas


associated with the finding of
obstructive jaundice with an The enlarged and non-tender gallbladder with
enlarged non-tender associated painless jaundice is what’s known
gallbladder? as the Courvoisier sign.

A. Liver This happens in patients with pancreatic


B. Pancreas cancer.
C. Duodenum
D. Stomach Reference:
Kumar, V., Abbas, A. & Aster, J. (2020). The pancreas.
In A. Maitra, Robbins and Cotran Pathologic Basis of
Diseases (10th ed., p. 893). Amsterdam: Elsevier.

64. What is the most common A A. Papillary


type of thyroid cancer?
The most common type of thyroid cancer is
A. Papillary papillary thyroid carcinoma.
B. Anaplastic
C. Medullary Reference:
D. Follicular Kumar, V., Abbas, A. & Aster, J. (2020). The endocrine
system. In A. Maitra, Robbins and Cotran Pathologic
Basis of Diseases (10th ed., p. 1088). Amsterdam:
Elsevier.

65. A biopsy was done on a A A. Papillary


patient who is highly suspected
of having thyroid carcinoma. The description on microscopy is consistent
with Orphan Annie eye nuclei which are found
Microscopy showed: in papillary thyroid carcinoma.
Empty-appearing nuclei with
dispersed chromatin.
What is the most likely Reference:
diagnosis? Kumar, V., Abbas, A. & Aster, J. (2020). The endocrine
system. In A. Maitra, Robbins and Cotran Pathologic
Basis of Diseases (10th ed., p. 1088). Amsterdam:

158
A. Papillary Elsevier.
B. Anaplastic
C. Medullary
D. Follicular

66. An 80/M came to the clinic C C. Peripheral


for a 3-month history of lower
urinary tract symptoms, The age of the patient, lower urinary tract
hematuria, weight loss, symptoms, hematuria, weight loss,
generalized weakness, and back generalized weakness, and back pain point to
pain. a most likely diagnosis of prostate cancer.

The most likely pathology is Most cases (70%) of prostate carcinoma can
found in which zone of the be found in the peripheral zone.
prostate gland?
Reference:
A. Central Kumar, V., Abbas, A. & Aster, J. (2020).The lower
urinary tract and male genital system. In G.J. Netto &
B. Midcentral M.B. Amin, Robbins and Cotran Pathologic Basis of
C. Peripheral Diseases (10th ed., pp. 979). Amsterdam: Elsevier.
D. Transition

67. What is considered as the A A. Serology


mainstay of diagnosis of
Syphilis? The mainstay of diagnosis of syphilis is
serology (i.e., nontreponemal and
A. Serology antitreponemal antibody tests).
B. Dark-field microscopy
C. Biopsy Reference:
D. Culture Kumar, V., Abbas, A. & Aster, J. (2020). Infectious
diseases. In K. Frank & A.J. McAdam (Eds.), Robbins
and Cotran Pathologic Basis of Diseases (10th ed., p.
377). Amsterdam: Elsevier.

68. A patient diagnosed with B B. Trousseau syndrome


pancreatic cancer develops DVT
in his left leg, and spontaneously Trousseau sign is also known as migratory
disappears, only to reappear on thrombophlebitis and occurs in patients with
his right leg. pancreatic adenocarcinoma.

What is this phenomenon Leser-Trelat sign (appearance of multiple


called? seborrheic keratoses) occurs in patients with
possible malignancy of the GIT.
A. Chvostek sign
B. Trousseau syndrome References:
C. Leser-Trelat sign ● Kumar, V., Abbas, A. & Aster, J. (2020). The
pancreas. In A. Maitra, Robbins and Cotran
D. Acanthosis nigricans Pathologic Basis of Diseases (10th ed., p. 893).
Amsterdam: Elsevier.
● Kumar, V., Abbas, A. & Aster, J. (2020). The skin.
In A.J. Lazar (Ed.), Robbins and Cotran
Pathologic Basis of Diseases (10th ed., p. 1142).
Amsterdam: Elsevier.

69. The following autoimmune B B. Type 2 diabetes


diseases can also develop in
patients with Hashimoto Patients with Hashimoto thyroiditis can also

159
thyroiditis, EXCEPT? develop the following autoimmune diseases:
● Type 1 DM
A. Type 1 diabetes ● Autoimmune adrenalitis
B. Type 2 diabetes ● SLE
C. SLE ● MG
D. Myasthenia gravis ● Sjogren syndrome

Reference:
Kumar, V., Abbas, A. & Aster, J. (2020). The endocrine
system. In A. Maitra (Ed.), Robbins and Cotran
Pathologic Basis of Diseases (10th ed., pp. 1129).
Amsterdam: Elsevier.

70. Deficiency of what substance D D. Vitamin B12


in the body leads to
degeneration of ascending and Vitamin B12 deficiency may lead to
descending spinal tracts? degeneration of ascending and descending
spinal tracts or what’s called subacute
A. Vitamin A combined degeneration of the spinal cord.
B. Vitamin E
C. Vitamin B1 Vitamin B1 or thiamine deficiency results in
D. Vitamin B12 Wernicke-Korsakoff syndrome.

Vitamin A deficiency may result in the


appearance of Bitot spots in the eyes.

Vitamin E deficiency can lead to


spinocerebellar degeneration.

Reference:
Kumar, V., Abbas, A. & Aster, J. (2020). The central
nervous system. In M. Margeta & A. Perry (Eds.),
Robbins and Cotran Pathologic Basis of Diseases (10th
ed., pp. 1291). Amsterdam: Elsevier.

71. Which of the following is the D D. Viral infections


most common cause of
myocarditis? Coxsackie viruses A and B are the most
common causes of myocarditis.
A. Acute coronary
syndrome Reference:
B. Chagas disease Kumar, V., Abbas, A. & Aster, J. (2020). The heart. In
R.N. Mitchell & A.J. Connolly (Eds.), Robbins and
C. Alcohol Cotran Pathologic Basis of Diseases (10th ed., p. 575).
D. Viral infections Amsterdam: Elsevier.

72. What is the known leading C C. Coronary Artery Disease


cause of sudden cardiac death?
The leading cause of SCD is CAD.
A. Aortic stenosis
B. Rheumatic heart disease Reference:
C. Coronary artery disease Kumar, V., Abbas, A. & Aster, J. (2020). The heart. In
R.N. Mitchell & A.J. Connolly (Eds.), Robbins and
D. Alcohol cardiomyopathy Cotran Pathologic Basis of Diseases (10th ed., p. 555).
Amsterdam: Elsevier.

73. What is the most common B B. Aspiration

160
mechanism by which pathogenic
organisms gain access to the The most common mechanism by which
lung and cause lung abscesses? pathogenic organisms gain access to the lung
to cause lung abscess is aspiration of
A. Septic embolism infective material.
B. Aspiration
C. Luminal obstruction from Risk factors to this include:
a mass ● Suppressed cough reflexes
D. All of the above ● Dysphagia
● Protracted vomiting
● Poor dental hygiene

Reference:
Kumar, V., Abbas, A. & Aster, J. (2020). The lung. In A.
Chang & Z.G. Laszik (Eds.), Robbins and Cotran
Pathologic Basis of Diseases (10th ed., p. 711).
Amsterdam: Elsevier.

74. Which of the following is A A. More common on the left lung


NOT true for lung abscesses?
Lung abscesses from aspiration are more
A. More common on the left common on the right lung because the right
lung main bronchus is more vertical than the left.
B. May present with foul-
smelling sputum Reference:
C. Clubbing of the toes may Kumar, V., Abbas, A. & Aster, J. (2020). The lung. In A.
Chang & Z.G. Laszik (Eds.), Robbins and Cotran
be present Pathologic Basis of Diseases (10th ed., p. 711).
D. None of the above Amsterdam: Elsevier.

75. What is considered as the A A. Aphthous ulcer


earliest lesion of Crohn's
disease? All the options are present in Crohn's disease.
But the earliest lesions in Crohn's disease are
A. Aphthous ulcer aphthous ulcers.
B. Crypt abscesses
C. Creeping fat Reference:
D. Noncaseating Kumar, V., Abbas, A. & Aster, J. (2020). The
gastrointestinal tract. In Robbins and Cotran Pathologic
granulomas Basis of Diseases (10th ed., p. 800). Amsterdam:
Elsevier.

76. Which of the following B B. Silicosis


pneumoconioses can manifest
radiographically as eggshell Eggshell calcifications are seen
calcification? radiographically in patients with silicosis.

A. Coal workers’ Reference:


pneumoconiosis Kumar, V., Abbas, A. & Aster, J. (2020). The lung. In A.
Chang & Z.G. Laszik (Eds.), Robbins and Cotran
B. Silicosis Pathologic Basis of Diseases (10th ed., p. 694).
C. Asbestosis Amsterdam: Elsevier.
D. Beryllium exposure

77. Which of the following is A A. Associated with higher risk of lung cancer
TRUE in patients who are
smokers and with heavy Patients who are smokers and with heavy

161
asbestos exposure? asbestos exposure are at higher risk to
develop asbestos-related lung cancer.
A. Associated with higher
risk of lung cancer There is no increased risk of developing
B. Associated with higher mesothelioma in patients who smoke and are
risk of malignant exposed to asbestos.
mesothelioma
C. Associated with lower Reference:
risk of dying from lung Kumar, V., Abbas, A. & Aster, J. (2020). The lung. In A.
Chang & Z.G. Laszik (Eds.), Robbins and Cotran
cancer Pathologic Basis of Diseases (10th ed., p. 726).
D. None of the above Amsterdam: Elsevier.

78. Which of the following is A A. Osteoclastic metastasis


NOT present in patients with
prostate cancer? Osteoblastic metastasis occurs in prostate
cancer, not osteoclastic.
A. Osteoclastic metastasis
B. Osteoblastic metastasis Reference:
C. Hematogenously spreads Kumar, V., Abbas, A. & Aster, J. (2020).The lower
urinary tract and male genital system. In G.J. Netto &
to the bones M.B. Amin, Robbins and Cotran Pathologic Basis of
D. None of the above Diseases (10th ed., p. 979). Amsterdam: Elsevier.

79. Which of the following C C. Seminoma


testicular tumors has the best
prognosis? Of the testicular tumors, seminomas, which
are both radio- and chemosensitive, have the
A. Yolk sac tumor best prognosis.
B. Pure embryonal
carcinoma Reference:
C. Seminoma Kumar, V., Abbas, A. & Aster, J. (2020).The lower
urinary tract and male genital system. In G.J. Netto &
D. Sex cord-gonadal M.B. Amin, Robbins and Cotran Pathologic Basis of
stromal tumors Diseases (10th ed., p. 973). Amsterdam: Elsevier.

80. The appearance of gas- B B. Emphysematous cholecystitis


forming bacteria in the
gallbladder such as clostridia Gas-forming bacteria in the gallbladder such
may result in which of the as clostridia may result in emphysematous
following? cholecystitis.

A. Gangrenous cholecystitis Reference:


B. Emphysematous Kumar, V., Abbas, A. & Aster, J. (2020). Liver and
gallbladder. In R.M. Gill & S. Kakar, Robbins and
cholecystitis Cotran Pathologic Basis of Diseases (10th ed., p. 875).
C. Calculous cholecystitis Amsterdam: Elsevier.
D. Acalculous cholecystitis

81. Which of the following does D D. Hyposecretion of mucus


NOT contribute to the formation
of cholesterol gallstones? A, B, and C lead to the formation of
cholesterol stones.
A. Supersaturation of bile
with cholesterol D should be HYPERsecretion of mucus, not
B. Gallstone hypomotility hyposecretion.
C. Cholesterol crystal Reference:
nucleation Kumar, V., Abbas, A. & Aster, J. (2020). Liver and
gallbladder. In R.M. Gill & S. Kakar, Robbins and

162
D. Hyposecretion of mucus Cotran Pathologic Basis of Diseases (10th ed., p. 875).
Amsterdam: Elsevier.

82. The parasite that causes A. A. Triatomine bugs


Chagas disease is transmitted to
humans by ___: The parasite (Trypanosoma cruzi) that causes
Chagas disease or American trypanosomiasis
A. Triatomine bugs is transmitted to humans by triatomine bugs
B. Tsetse flies (a.k.a. kissing bugs or reduviid bugs).
C. Cysts in meat
D. Cysts in water Tsetse fly is responsible for transmitting
Trypanosoma brucei which causes African
sleeping sickness or African trypanosomiasis.

Reference:
Kumar, V., Abbas, A. and Aster, J. (2020). Infectious
diseases. In K. Frank & A.J. McAdam (Eds.), Robbins
and Cotran Pathologic Basis of Diseases (10th ed., pp.
392-393). Amsterdam: Elsevier.

83. Exposure to which of the A A. Arsenic


following environmental
carcinogens can cause skin Arsenic - Skin cancer
cancer? Benzene - Acute myeloid leukemia
Cadmium - Prostate cancer
A. Arsenic Vinyl chloride - Hepatic angiosarcoma
B. Benzene
C. Cadmium Reference:
D. Vinyl chloride Kumar, V., Abbas, A. and Aster, J. (2020). Neoplasia. In
Robbins and Cotran Pathologic Basis of Diseases (10th
ed., p. 280). Amsterdam: Elsevier.

84. Which of the following D D. Small cell lung cancer


cancers can cause SIADH as a
paraneoplastic syndrome? Aside from causing SIADH as a
paraneoplastic syndrome, small cell lung
A. Renal cancer cancer can also cause Cushing syndrome.
B. Cerebellar hemangioma
C. Breast cancer Renal carcinoma can cause hypercalcemia
D. Small cell lung cancer and polycythemia.

Cerebellar hemangioma can cause


polycythemia.

Breast cancer can cause hypercalcemia.

Reference:
Kumar, V., Abbas, A. and Aster, J. (2020). Neoplasia. In
Robbins and Cotran Pathologic Basis of Diseases (10th
ed., p. 329). Amsterdam: Elsevier.

85. Which of the following can B B. Vitamin A excess


cause pseudotumor cerebri?
Acute Vitamin A toxicity may result in
A. Vitamin A deficiency pseudotumor cerebri.
B. Vitamin A excess

163
C. Vitamin B12 deficiency Acute Vitamin A toxicity may cause signs and
D. Vitamin C deficiency symptoms such as headache, vomiting,
blurred vision which are similar to a patient
with increased ICP from a brain tumor.

Reference:
Kumar, V., Abbas, A. and Aster, J. (2020).
Environmental and nutritional deficiencies. In Robbins
and Cotran Pathologic Basis of Diseases (10th ed., p.
438). Amsterdam: Elsevier.

86. What is the most common A A. IgG


monoclonal immunoglobulin in
multiple myeloma? IgG (~55%) is the most common monoclonal
immunoglobulin in multiple myeloma followed
A. IgG by IgA (~25%).
B. IgA
C. IgM Reference:
D. IgD Kumar, V., Abbas, A. & Aster, J. (2020). Diseases of
white blood cells, lymph nodes, spleen, and thymus. In
A. Chang & Z.G. Laszik (Eds.), Robbins and Cotran
Pathologic Basis of Diseases (10th ed., p. 609).
Amsterdam: Elsevier.

87. An elderly patient was B B. Bullous pemphigoid


referred to you for the
appearance of tense bullae on Bullous pemphigoid
the flexor aspects of her ● Commonly seen in elderly
forearms, axillae, and groin. ● Tense bullae
● Associated with anti-hemidesmosome
On immunofluorescence of the (IgG)
skin, there was a note of linear ● On immunofluorescence (IF), linear
deposition of complement along deposition at the dermoepidermal
the dermoepidermal junction. junction

What is the most likely Pemphigus vulgaris


diagnosis? ● Flaccid intraepidermal bullae
● Also seen in elderly. Potentially fatal
A. Pemphigus vulgaris ● Associated with anti-desmosome
B. Bullous pemphigoid (anti-desmoglein; IgG)
C. Dermatitis herpetiformis ● On IF, appears as reticular pattern
D. None of the above along plasma membranes of
keratinocytes
References:
● Le, T., Bhushan, V., & Sochat, M. (2023). First Aid
for the USMLE Step 1 2022, Thirty Third Edition
(33rd ed., p. 489). McGraw-Hill Education
● Kumar, V., Abbas, A. & Aster, J. (2020). The skin.
In A.J. Lazar (Ed.), Robbins and Cotran
Pathologic Basis of Diseases (10th ed., pp. 1159-
1162). Amsterdam: Elsevier.

88. What is the most significant C C. Tobacco


risk factor for the development of
renal cell carcinoma? The most significant risk factor for
development of renal cell carcinoma is
A. Alcohol tobacco smoking.

164
B. Exposure to radiation
C. Tobacco Exposure to vinyl chloride predisposes to
D. Vinyl chloride hepatic angiosarcoma.

Reference:
Kumar, V., Abbas, A. & Aster, J. (2020). The kidney. In
A. Chang & Z.G. Laszik (Eds.), Robbins and Cotran
Pathologic Basis of Diseases (10th ed., p. 948).
Amsterdam: Elsevier.

89. Which of the following is C C. Paget disease


described as a unilateral
erythematous lesion with a scaly Paget disease of the nipple is an extension
crust over the nipple and areola? from DCIS within the ductal system via the
lactiferous sinuses into the nipple skin.
A. Ductal carcinoma in situ
B. Lobular carcinoma in situ This is a rare presentation of breast
C. Paget disease carcinoma.
D. Inflammatory breast
carcinoma Reference:
Kumar, V., Abbas, A. & Aster, J. (2020). The breast. In
S.C. Lester (Ed.), Robbins and Cotran Pathologic Basis
of Diseases (10th ed., pp. 1053-1054). Amsterdam:
Elsevier.

90. These are skin lesions which C C. Lichen planus


are described as pruritic, purple,
polygonal, planar, papules and Skin lesions described as pruritic, purple,
plaques: polygonal, planar, papules and plaques are
characteristic of Lichen Planus.
A. Psoriasis These descriptions are what’s known as the
B. Wickham striae “six Ps” of lichen planus.
C. Lichen planus
D. Dermatitis herpertiformis Wickham striae are white dots or lines that
highlight the papules that are present in lichen
planus.

Psoriasis is described by skin lesions (e.g.,


plaques) with silver scales.

Dermatitis herpetiformis lesions are


described as itchy papules, vesicles or bullae
that are distributed symmetrically and involve
the extensor surfaces such as elbows, knees,
upper back and buttocks.

References:
● Kumar, V., Abbas, A. & Aster, J. (2020). The skin.
In A.J. Lazar (Ed.), Robbins and Cotran
Pathologic Basis of Diseases (10th ed., pp. 1158,
1163). Amsterdam: Elsevier.
● Le, T., Bhushan, V., & Sochat, M. (2023). First Aid
for the USMLE Step 1 2022, Thirty Third Edition
(33rd ed., p. 490). McGraw-Hill Education

91. Which of the following skin B B. Dermatitis herpetiformis


disorders is associated with

165
celiac disease? Dermatitis herpetiformis is associated with
celiac disease.
A. Erythema multiforme
B. Dematitis herpetiformis In patients who have IgA antibodies to dietary
C. Psoriasis gluten, they develop skin blisters from the
D. Lichen planus cross-reaction of the antibodies to the reticulin
present in the skin which leads to
manifestations of dermatitis herpetiformis.

References:
● Kumar, V., Abbas, A. & Aster, J. (2020). The skin.
In A.J. Lazar (Ed.), Robbins and Cotran
Pathologic Basis of Diseases (10th ed., p. 1163).
Amsterdam: Elsevier.
● Le, T., Bhushan, V., & Sochat, M. (2023). First Aid
for the USMLE Step 1 2022, Thirty Third Edition
(33rd ed., pp. 388, 490). McGraw-Hill Education

92. Which of the following stone B B. Magnesium ammonium phosphate stones


types form after infection from
urea-splitting bacteria? Infections by urea-splitting bacteria (that
convert urea to ammonia) such as Proteus
A. Uric acid stones can lead to formation of magnesium
B. Magnesium ammonium ammonium phosphate stones in the kidney.
phosphate stones
C. Calcium oxalate stones They can form large stones in the renal pelvis
D. Cystine stones leading to so-called staghorn calculi.
Reference:
Kumar, V., Abbas, A. & Aster, J. (2020). The kidney. In
A. Chang & Z.G. Laszik (Eds.), Robbins and Cotran
Pathologic Basis of Diseases (10th ed., p. 947).
Amsterdam: Elsevier.

93. What is the most common A A. Calcium oxalate stone


type of renal calculi?
The most common type of renal calculi is
A. Calcium oxalate stone calcium oxalate stones or calcium oxalate
B. Cystine stone mixed with calcium phosphate.
C. Uric acid stone
D. Struvite stone Reference:
Kumar, V., Abbas, A. & Aster, J. (2020). The kidney. In
A. Chang & Z.G. Laszik (Eds.), Robbins and Cotran
Pathologic Basis of Diseases (10th ed., p. 947).
Amsterdam: Elsevier.

94. Which of the following rare A A. Alpha cell tumors


pancreatic endocrine neoplasms
is associated with diabetes Alpha cell tumors, also called
mellitus, a characteristic skin glucagonomas (with high plasma glucagon
rash, and anemia? levels), are associated with diabetes mellitus,
a characteristic skin rash (necrolytic migratory
A. Alpha cell tumors erythema), and anemia.
B. Delta cell tumors
C. VIPomas Delta cell tumors are associated with
D. Pancreatic carcinoid diabetes mellitus, cholelithiasis, steatorrhea,
tumors and hypochlorhydria.

166
VIPomas are described as having watery
diarrhea, hypokalemia, and achlorhydria or
also called WDHA syndrome. These are
caused by release of vasoactive intestinal
peptides from the tumor.

Reference:
Kumar, V., Abbas, A. & Aster, J. (2020). The endocrine
system. In A. Maitra (Ed.), Robbins and Cotran
Pathologic Basis of Diseases (10th ed., p. 1114).
Amsterdam: Elsevier.

95. The “rule of 10s” in B B. 10% are not biologically malignant


pheochromocytoma states the
following, EXCEPT? The “rule of 10s” in pheochromocytoma
states the following:
A. 10% are bilateral ● 10% are extraadrenal
B. 10% are not biologically ● 10% are bilateral
malignant ● 10% are biologically malignant
C. 10% do not have ● 10% are not associated with
hypertension hypertension
D. None of the above
Reference:
Kumar, V., Abbas, A. & Aster, J. (2020). The endocrine
system. In A. Maitra (Ed.), Robbins and Cotran
Pathologic Basis of Diseases (10th ed., p. 1127).
Amsterdam: Elsevier.

96. Which of the following MENs A A. MEN1


is not associated with
pheochromocytoma? The 3Ps in MEN1 are:
● Pituitary tumors
A. MEN1 ● Pancreatic endocrine tumors
B. MEN2A ● Parathyroid adenomas
C. MEN2B
D. None of the above MEN2A and MEN2B are associated with
pheochromocytomas

Reference:
Le, T., Bhushan, V., & Sochat, M. (2023). First Aid for
the USMLE Step 1 2022, Thirty Third Edition (33rd ed.,
p. 356). McGraw-Hill Education

97. What is the most commonly C C. Knees


affected joint in calcium
pyrophosphate crystal deposition The most commonly affected joints in calcium
disease (CPPD)? pyrophosphate crystal deposition disease
(CPPD) are the knees.
A. Hips
B. Lumbar vertebrae Reference:
C. Knees Kumar, V., Abbas, A. & Aster, J. (2020). Bones, joints,
and soft tissue tumors. In A. Horvai (Ed.), Robbins and
D. 1st metatarsophalangeal Cotran Pathologic Basis of Diseases (10th ed., p. 1206).
joint Amsterdam: Elsevier.

98. What is the most B B. Chromatin condensation


characteristic feature of

167
apoptosis? The most characteristic feature of apoptosis is
chromatin condensation.
A. Cell shrinkage
B. Chromatin condensation The other options are also morphologic
C. Apoptotic bodies features that occur in cells undergoing
D. Phagocytosis of apoptosis, but are not considered most
apoptotic cells characteristic.

Reference:
Kumar, V., Abbas, A. & Aster, J. (2020). Cell injury, cell
death, and adaptations. In S.A. Oakes (Ed.), Robbins
and Cotran Pathologic Basis of Diseases (10th ed., p.
43). Amsterdam: Elsevier

99. Which of the following indoor D D. Radon


air pollutants is considered the
number one cause of lung Radon is an indoor air pollutant and is
cancer in nonsmokers? considered the number 1 cause of lung
cancer among nonsmokers.
A. Bioaerosols Carbon monoxide is considered an outdoor
B. Formaldehyde air pollutant.
C. Carbon monoxide
D. Radon Formaldehyde and bioaerosols are indoor air
pollutants but not considered to cause lung
cancer in nonsmokers.

Reference:
Kumar, V., Abbas, A. and Aster, J. (2020).
Environmental and nutritional deficiencies. In Robbins
and Cotran Pathologic Basis of Diseases (10th ed., p.
410). Amsterdam: Elsevier.

100. Acute poisoning of which of C C. Carbon monoxide


the following air pollutants is
characterized by cherry-red color Acute poisoning of carbon monoxide can
of the skin? manifest with cherry-red discoloration of the
skin and mucous membranes from increased
A. Antifreeze levels of carboxyhemoglobin.
B. Ozone
C. Carbon monoxide Ozone exposure can lead to symptoms of
D. Carbon dioxide chest discomfort and URT inflammation.

Antifreeze uses ethylene glycol which can


lead to severe damage to the CNS, heart,
lungs and kidneys.

Reference:
● Kumar, V., Abbas, A. and Aster, J. (2020).
Environmental and nutritional deficiencies. In
Robbins and Cotran Pathologic Basis of Diseases
(10th ed., p. 410). Amsterdam: Elsevier.\
● Loscalzo J, & Fauci A, & Kasper D, & Hauser S, &
Longo D, & Jameson J. (2022). Acidosis and
alkalosis. In T.D. DuBose, Jr. (Ed.), Harrison's
Principles of Internal Medicine (21st ed., Vol.1, pp.
1711). New York, NY: McGraw Hill.

168
PHARMACOLOGY

169
QUESTION ANSWER EXPLANATION

1. The use of which of the following A A. Diphenhydramine


should be avoided before operating
machinery? Diphenhydramine is considered a 1st-
generation antihistamine and therefore
A. Diphenhydramine has more sedating effects than 2nd-
B. Cetirizine generation agents.
C. Fexofenadine
D. Loratadine Examples of 2nd-generation
antihistamines are the following:
● Cetirizine
● Fexofenadine
● Loratadine

Reference:
Trevor, A. J., Katzung, B. G., & Kruidering-Hall,
M. (2018). Histamine, serotonin, & the ergot
alkaloids. In Katzung & Trevor’s Pharmacology
Examination and Board Review (12th ed., p.
147). New York, NY: McGraw-Hill Education.

2. The 2nd-generation antihistamines A A. Less lipid soluble


have less autonomic side effects
compared to 1st-generation agents The 2nd-generation antihistamines
because they are ___: are less lipid soluble than the 1st
generation agents (1st generation
A. Less lipid-soluble agents are more lipid soluble,
B. More lipid-soluble nonionized, uncharged).
C. Mostly nonionized
D. Mostly uncharged The 2nd-generation antihistamines do
not readily enter the CNS (less lipid
soluble) which gives them less ability to
cause sedating and autonomic effects.

Reference:
Trevor, A. J., Katzung, B. G., & Kruidering-Hall,
M. (2018). Histamine, serotonin, & the ergot
alkaloids. In Katzung & Trevor’s Pharmacology
Examination and Board Review (12th ed., p.
147). New York, NY: McGraw-Hill Education.

3. The following anticholinesterase D D. Neostigmine


drugs are used in the treatment of
Alzheimer disease, EXCEPT: Donepezil, rivastigmine, and
galantamine are used as 1st line
A. Donepezil treatment for patients with Alzheimer
B. Galantamine disease.
C. Rivastigmine
D. Neostigmine References:
● Le, T., Bhushan, V., & Sochat, M. (2023).
First Aid for the USMLE Step 1 2022, Thirty
Third Edition (33rd ed., p. 239). McGraw-
Hill Education.
● Trevor, A. J., Katzung, B. G., & Kruidering-
Hall, M. (2018). Histamine, serotonin, & the
ergot alkaloids. In Katzung & Trevor’s
Pharmacology Examination and Board

170
Review (12th ed., p. 68). New York, NY:
McGraw-Hill Education.

4. Which of the following B B. Losartan


antihypertensive agents are absolutely
contraindicated in pregnant patients Losartan is an ACEi and is therefore
because of the risk of major renal absolutely contraindicated in the
damage in the fetus? treatment of hypertension in pregnant
patients because of the risk of renal
A. Labetalol damage in the fetus.
B. Losartan
C. Hydralazine The other drugs are included in the
D. NIfedipine choices of therapy for treatment of
hypertension in pregnancy.

Reference:
Trevor, A. J., Katzung, B. G., & Kruidering-Hall,
M. (2018). Drugs used in hypertension. In
Katzung & Trevor’s Pharmacology Examination
and Board Review (12th ed., p. 97). New York,
NY: McGraw-Hill Education.

5. Tolerance to most of the B B. Miosis


pharmacologic effects of opioids
occurs, EXCEPT for: Tolerance to most of the
pharmacologic effects of opioids
A. Respiratory depression occurs, except for miosis and
B. Miosis constipation.
C. Nausea and vomiting
D. Sedation Reference:
Trevor, A. J., Katzung, B. G., & Kruidering-Hall,
M. (2018). Opioid analgesics & antagonists. In
Katzung & Trevor’s Pharmacology Examination
and Board Review (12th ed., p. 260). New York,
NY: McGraw-Hill Education.

6. One characteristic effect of all opioids B B. Meperidine


is pupillary constriction, EXCEPT?
All opioids cause pupillary constriction
A. Morphine and contraction of biliary tract smooth
B. Meperidine muscle, EXCEPT for meperidine.
C. Fentanyl
D. Hydrocodone Reference:
Trevor, A. J., Katzung, B. G., & Kruidering-Hall,
M. (2018). Opioid analgesics & antagonists. In
Katzung & Trevor’s Pharmacology Examination
and Board Review (12th ed., p. 260). New York,
NY: McGraw-Hill Education.

7. Overdose in opioid analgesics is D D. Diarrhea


characterized by the following,
EXCEPT? The triad of pupillary constriction,
comatose, and respiratory depression
A. Pupillary constriction describes overdose in opioid
B. Comatose analgesics.
C. Respiratory depression
D. Diarrhea Reference:
Trevor, A. J., Katzung, B. G., & Kruidering-Hall,

171
M. (2018). Opioid analgesics & antagonists. In
Katzung & Trevor’s Pharmacology Examination
and Board Review (12th ed., p. 261). New York,
NY: McGraw-Hill Education.

8. Tamoxifen can be used in the B B. Endometrial cancer


prevention of breast cancer, but it can
increase the risk of having which of the Tamoxifen is a selective estrogen
following? receptor modulator (SERM) that serves
as an antagonist in breast tissue (can
A. Osteoporosis prevent or treat breast cancer) but an
B. Endometrial cancer agonist in endometrial receptors which
C. Cervical cancer increases risk of endometrial cancer.
D. All of the above
It can also predispose to development
of venous thrombosis.

It has an agonist effect in bones, so it


does NOT increase the risk of
osteoporosis.

Reference:
Trevor, A. J., Katzung, B. G., & Kruidering-Hall,
M. (2018). Gonadal hormones & inhibitors. In
Katzung & Trevor’s Pharmacology Examination
and Board Review (12th ed., p. 341). New York,
NY: McGraw-Hill Education.

9. Which of the following drugs inhibits B B. Anastrozole


the enzyme that promotes the
conversion of androstenedione to Anastrozole is an aromatase inhibitor
estrone and is used in the treatment of and is used in the treatment of
ER (+) breast cancer? postmenopausal women with ER (+)
breast cancer.
A. Raloxifene
B. Anastrozole Raloxifene is used as a SERM and,
C. Fulvestrant just like Tamoxifen, is an antagonist at
D. Mifepristone the breast. But unlike Tamoxifen, it
doesn’t increase the risk of endometrial
cancer.

Fulvestrant is a full estrogen receptor


antagonist and is also used in the
treatment of breast cancer.

Reference:
Trevor, A. J., Katzung, B. G., & Kruidering-Hall,
M. (2018). Cancer chemotherapy. In Katzung &
Trevor’s Pharmacology Examination and Board
Review (12th ed., p. 458). New York, NY:
McGraw-Hill Education.

10. What is the main toxicity of low A A. Esophageal irritation


doses of oral bisphosphonates used for
osteoporosis? Esophageal and gastric irritation is the
main toxicity that patients with
A. Esophageal irritation osteoporosis will likely experience

172
B. Acute kidney injury when taking low doses of oral
C. Osteonecrosis of the jaw bisphosphonates.
D. All of the above
To avoid esophageal irritation, the oral
bisphosphonates must be taken with
large amounts of water and the patient
should remain upright for at least
30min after taking the drug.

Higher doses of bisphosphonates are


associated with renal impairment and
osteonecrosis of the jaw.

Reference:
Trevor, A. J., Katzung, B. G., & Kruidering-Hall,
M. (2018). Drugs that affect bone mineral
homeostasis. In Katzung & Trevor’s
Pharmacology Examination and Board Review
(12th ed., p. 361). New York, NY: McGraw-Hill
Education.

11. Which of the following antidiabetic B B. Linagliptin


drugs work by inhibiting the enzyme
that degrades GLP-1? Linagliptin is classified as a DPP-4
inhibitor.
A. Liraglutide
B. Linagliptin Dipeptidyl peptidase 4 (DDP-4) is the
C. Canagliflozin enzyme that deactivates GLP-1.
D. Glipizide
Inhibition of this enzyme by DPP-4
inhibitors (e.g., linagliptin) leads to
increased insulin release and
decreased glucagon.

Other examples of DPP-4 inhibitors:


● Sitagliptin
● Saxagliptin
● Vildagliptin

Liraglutide is a GLP-1 analog.

Canagliflozin is an SGLT2 inhibitor.

Glipizide is a sulfonylurea.

Reference:
Trevor, A. J., Katzung, B. G., & Kruidering-Hall,
M. (2018). Pancreatic hormones, antidiabetic
agents, & glucagon. In Katzung & Trevor’s
Pharmacology Examination and Board Review
(12th ed., p. 352). New York, NY: McGraw-Hill
Education.

12. A neonate was incidentally found to C C. Indomethacin


have a cardiac murmur. On
auscultation, it was characterized as: The case described is a neonate with
Continuous, machinery-like murmur PDA or patent ductus arteriosus.

173
best heard at the left infraclavicular
region. Indomethacin is an NSAID that can
hasten the closure of a PDA.
Which of the following can be used to
hasten the closure of this congenital PGE is used to maintain patency of the
abnormality? PDA.

A. Prostaglandin E References:
B. Prostagladin D ● Trevor, A. J., Katzung, B. G., & Kruidering-
C. Indomethacin Hall, M. (2018). Prostaglandins & other
eicosanoids. In Katzung & Trevor’s
D. High levels of oxygen Pharmacology Examination and Board
Review (12th ed., p. 164). New York, NY:
McGraw-Hill Education.
● Le, T., Bhushan, V., & Sochat, M. (2023).
First Aid for the USMLE Step 1 2022, Thirty
Third Edition (33rd ed., p. 303). McGraw-
Hill Education.

13. Which of the following is used as an A A. Mifepristone


abortifacient in early pregnancy by
acting as a progesterone and Mifepristone (RU- 486) is an
glucocorticoid receptor antagonist? abortifacient in early pregnancy by
acting as a progesterone and
A. Mifepristone glucocorticoid receptor antagonist.
B. Spironolactone
C. Misoprostol Misoprostol is a PGE1 derivative that
D. Aspirin increases uterine motility.

Spironolactone is not considered an


abortifacient.

Aspirin is an irreversible inhibitor of


COX1 and COX2.
References:
● Trevor, A. J., Katzung, B. G., & Kruidering-
Hall, M. (2018). Gonadal hormones &
inhibitors. In Katzung & Trevor’s
Pharmacology Examination and Board
Review (12th ed., p. 341). New York, NY:
McGraw-Hill Education.
● Trevor, A. J., Katzung, B. G., & Kruidering-
Hall, M. (2018). Keywords for key drugs. In
Katzung & Trevor’s Pharmacology
Examination and Board Review (12th ed.,
p. 530). New York, NY: McGraw-Hill
Education.

14. Which of the following is considered B B. Clomiphene


a partial agonist at estrogen receptors
and is commonly used to induce Raloxifene is used as a SERM and,
ovulation for patients who want to just like Tamoxifen, is an antagonist at
become pregnant? the breast. But unlike Tamoxifen, it
doesn’t increase the risk of endometrial
A. Raloxifene cancer.
B. Clomiphene
C. Danazol Danazol is used in the treatment of
D. Flutamide patients with endometriosis. It is a

174
weak partial agonist at progestin,
androgen, and glucocorticoid
receptors.

Flutamide is used in patients with


prostate cancer and acts as
competitive antagonists of androgen
receptors.

Reference:
Trevor, A. J., Katzung, B. G., & Kruidering-Hall,
M. (2018). Gonadal hormones & inhibitors. In
Katzung & Trevor’s Pharmacology Examination
and Board Review (12th ed., p. 341). New York,
NY: McGraw-Hill Education.

15. Hypertensive crisis occurs when B B. MAO inhibitors


patients eat tyramine-rich foods while
taking which of the following Intake of tyramine-rich foods such as
medications? aged cheese and cured meats while
taking MAO inhibitors may result in
A. SSRIs hypertensive crisis.
B. MAO inhibitors
C. SNRIs Normally, tyramine is metabolized by
D. TCAs MAO.

But when taking MAO inhibitors,


tyramine is not metabolized which
leads to tyramine releasing stored
Norepinephrine into the synaptic cleft
and resulting to increased sympathetic
stimulation.

References:
● Le, T., Bhushan, V., & Sochat, M. (2023).
First Aid for the USMLE Step 1 2022, Thirty
Third Edition (33rd ed., p. 589). McGraw-
Hill Education.
● Trevor, A. J., Katzung, B. G., & Kruidering-
Hall, M. (2018). Cardiovascular drugs. In
Katzung & Trevor’s Pharmacology
Examination and Board Review (12th ed.,
p. 78). New York, NY: McGraw-Hill
Education.

16. Which of the following is considered A A. Eperisone


as a skeletal muscle relaxant?
Eperisone is a skeletal muscle
A. Eperisone relaxant. (I can’t find this drug in
B. Hydrocortisone Katzung or Harrison’s so I used
C. Risperidone UpToDate. And yes. This was
D. Dapsone apparently asked in the last PLE)

Reference:
Lexicomp. (n.d.). Eperisone: International drug
information (concise). UpToDate. Retrieved
February 1, 2023, from
https://www.uptodate.com/contents/eperisone-

175
international-drug-information-
concise?search=eperisone&source=panel_sear
ch_result&selectedTitle=1~1&usage_type=pane
l&kp_tab=drug_international&display_rank=1

17. Which of the following components B B. Chlorpheniramine


of drugs used for common cold (e.g.,
Decolgen Forte) is responsible for Chlorpheniramine, also known as
causing sedation? chlorphenamine, is classified as a 1st-
generation antihistamine and is
A. Phenylpropanolamine responsible for the sedating effect.
B. Chlorpheniramine
C. Paracetamol Reference:
D. Any of the above Trevor, A. J., Katzung, B. G., & Kruidering-Hall,
M. (2018). Histamine, serotonin, & the ergot
alkaloids. In Katzung & Trevor’s Pharmacology
Examination and Board Review (12th ed., p.
147). New York, NY: McGraw-Hill Education.

18. Drug metabolism of some drugs D D. Furanocoumarins


may be decreased by which of the
following? Furanocoumarins present in
grapefruit juice inhibit drug-
A. Barbiturates metabolizing enzymes such as the
B. Carbamazepine CYP3A4 isozyme of cytochrome P450.
C. Rifampin
D. Furanocoumarins Here’s a list of other cytochrome P-450
inducers and inhibitors from First Aid
for the USMLE Step 1 2023:

Cytochrome P-450 Interactions

Inducers (+) Inhibitors (-)

St. John’s wort Sodium valproate


Phenytoin Isoniazid
Phenobarbital Cimetidine
Modafinil Ketoconazole
Nevirapine Fluconazole
Rifampin Acute alcohol
Griseofulvin overuse
Carbamazepine Chloramphenicol
Chronic alcohol Erythromycin/
overuse clarithromycin
Sulfonamides
Ciprofloxacin
Omeprazole
Amiodarone
Ritonavir
Grapefruit juice

MNEMONICS
● Inducers (+)
St. John’s funny funny (phen-
phen) mom never refuses greasy
carbs and chronic alcohol

176
● Inhibitors (-)
SICK FACES come when I am
really drinking grapefruit juice

References:
● Le, T., Bhushan, V., & Sochat, M. (2023).
First Aid for the USMLE Step 1 2022, Thirty
Third Edition (33rd ed., p. 251). McGraw-
Hill Education.
● Trevor, A. J., Katzung, B. G., & Kruidering-
Hall, M. (2018). Drug interactions. In
Katzung & Trevor’s Pharmacology
Examination and Board Review (12th ed.,
p. 512). New York, NY: McGraw-Hill
Education.

19. The anticoagulant effects of C C. Phenytoin


warfarin can be increased by the
following drugs, EXCEPT? Phenytoin is a cytochrome P450
inducer which means it increases
A. Grapefruit juice warfarin’s clearance and thereby
B. Cimetidine decreasing its anticoagulant effect.
C. Phenytoin
D. Amiodarone Grapefruit juice (containing
furanocoumarins), cimetidine and
amiodarone are cytochrome P450
inhibitors which decrease the
clearance of warfarin and increase its
anticoagulant effects.

Reference:
Trevor, A. J., Katzung, B. G., & Kruidering-Hall,
M. (2018). Drugs used in coagulation disorders.
In Katzung & Trevor’s Pharmacology
Examination and Board Review (12th ed., p.
287). New York, NY: McGraw-Hill Education.

20. Which of the following B B. Heparin


anticoagulants activate antithrombin III
which leads to inactivation of thrombin Heparin activates antithrombin III
and factor Xa? which leads to inactivation of thrombin
and factor Xa.
A. Warfarin
B. Heparin Warfarin inhibits Vitamin K epoxide
C. Aspirin reductase (VKOR) leading to
D. All of the above decreased production of Vitamin-K
dependent clotting factors.

Reference:
Trevor, A. J., Katzung, B. G., & Kruidering-Hall,
M. (2018). Drugs used in coagulation disorders.
In Katzung & Trevor’s Pharmacology
Examination and Board Review (12th ed., p.
287). New York, NY: McGraw-Hill Education.

21. What is drug of choice for pregnant A A. Heparin


patients who need an anticoagulant?

177
The drug of choice for pregnant
A. Heparin patients who need an anticoagulant is
B. Warfarin heparin because it does not cross the
C. Idarucizumab placental barrier.
D. Dabigatran
Warfarin is contraindicated in pregnant
patients due to risks of bone defects
and hemorrhage in the fetus.

Idarucizumab is used to reverse the


effects of dabigatran.

Reference:
Trevor, A. J., Katzung, B. G., & Kruidering-Hall,
M. (2018). Drugs used in coagulation disorders.
In Katzung & Trevor’s Pharmacology
Examination and Board Review (12th ed., p.
286). New York, NY: McGraw-Hill Education.

22. What is the antidote for heparin A A. Protamine sulfate


overdose? Protamine is the reversal agent for
heparin.
A. Protamine sulfate
B. Vitamin K Vitamin K and prothrombin complex
C. Prothrombin complex concentrate are used to reverse the
concentrate effects of warfarin.
D. Any of the above
Reference:
Trevor, A. J., Katzung, B. G., & Kruidering-Hall,
M. (2018). Drugs used in coagulation disorders.
In Katzung & Trevor’s Pharmacology
Examination and Board Review (12th ed., p.
285). New York, NY: McGraw-Hill Education.

23. The following are side effects of D D. None of the above


using 1st-generation antihistamine,
EXCEPT? All the options are side effects
associated with 1st-generation
A. Dry mouth antihistamines.
B. Blurred vision
C. Sedation Reference:
D. None of the above Trevor, A. J., Katzung, B. G., & Kruidering-Hall,
M. (2018). Histamine, serotonin, & the ergot
alkaloids. In Katzung & Trevor’s Pharmacology
Examination and Board Review (12th ed., p.
147). New York, NY: McGraw-Hill Education.

24. Which of the following treatments B B. Baclofen


for muscle spasticity acts as a GABA
agonist? Baclofen is a GABAB agonist in the
spinal cord which decreases muscle
A. Dantrolene spasticity.
B. Baclofen
C. Tizanidine Tizanidine acts an alpha2 agonist that
D. All the above reduces the tonic output of the primary
spinal motoneurons.

178
Dantrolene acts to reduce the release
of calcium from the sarcoplasmic
reticulum.

Reference:
Trevor, A. J., Katzung, B. G., & Kruidering-Hall,
M. (2018). Skeletal muscle relaxants. In
Katzung & Trevor’s Pharmacology Examination
and Board Review (12th ed., p. 228). New York,
NY: McGraw-Hill Education.

25. Organophosphate poisoning B B. Miosis


presents with the following signs and
symptoms, EXCEPT? Organophosphate poisoning leads to
miosis, not mydriasis.
A. Diarrhea
B. Mydriasis Other signs and symptoms of
C. Bronchospasm organophosphate poisoning (from
D. Emesis irreversible inhibition of
acetylcholinesterase) are known by:
● Diarrhea
● Urination
● Miosis
● Bronchospasm
● Bradycardia
● Emesis
● Lacrimation
● Sweating
● Salivation

MNEMONIC: DUMBBELSS

Reference:
Le, T., Bhushan, V., & Sochat, M. (2023). First
Aid for the USMLE Step 1 2022, Thirty Third
Edition (33rd ed., p. 239). McGraw-Hill
Education.

26. Baclofen is considered a ___: C C. Spasmolytic

A. Depolarizing neuromuscular Baclofen is a spasmolytic which


blocking drug means it’s a drug that reduces spasms
B. Nondepolarizing neuromuscular without paralysis.
blocking drug
C. Spasmolytic A and B result in muscle paralysis.
D. None of the above
Reference:
Trevor, A. J., Katzung, B. G., & Kruidering-Hall,
M. (2018). Skeletal muscle relaxants. In
Katzung & Trevor’s Pharmacology Examination
and Board Review (12th ed., p. 226). New York,
NY: McGraw-Hill Education.

27. Which of the following is used as A A. Flumazenil


part of the management of
benzodiazepine overdose? Flumazenil is an antagonist at
benzodiazepine sites and is used as

179
A. Flumazenil part of the management of
B. Pralidoxime benzodiazepine overdose.
C. Naloxone
D. Naltrexone But flumazenil is not used routinely
because it can lead to seizures.

Reference:
Trevor, A. J., Katzung, B. G., & Kruidering-Hall,
M. (2018). Drugs that act in the central nervous
system. In Katzung & Trevor’s Pharmacology
Examination and Board Review (12th ed., p.
192). New York, NY: McGraw-Hill Education.

28. In a patient who was accidentally A A. Atropine


poisoned with parathion, which of the
following is the antidote of first choice? Parathion is an example of an
organophosphate
A. Atropine (acetylcholinesterase, or AChE,
B. Rivastigmine inhibitor).
C. Glucagon
D. Neostigmine The antidote of first choice for patients
with organophosphate poisoning is
atropine.

Reference:
Trevor, A. J., Katzung, B. G., & Kruidering-Hall,
M. (2018). Cholinoceptor blockers &
cholinesterase regenerators. In Katzung &
Trevor’s Pharmacology Examination and Board
Review (12th ed., p. 64). New York, NY:
McGraw-Hill Education.

29. In patients with organophosphate B B. Pralidoxime


poisoning, which of the following can
regenerate cholinesterase? Pralidoxime regenerates
cholinesterase in patients with
A. Atropine exposure to high doses of
B. Pralidoxime organophosphate AChE inhibitor
C. Diazepam insecticides.
D. Succinylcholine
Reference:
Trevor, A. J., Katzung, B. G., & Kruidering-Hall,
M. (2018). Cholinoceptor blockers &
cholinesterase regenerators. In Katzung &
Trevor’s Pharmacology Examination and Board
Review (12th ed., p. 72). New York, NY:
McGraw-Hill Education.

30. Isoproterenol is a _______. A A. Beta agonist

A. Beta agonist Isoproterenol is a Beta2-agonist.


B. Beta blocker
C. Alpha1-agonist Reference:
D. Alpha1-antagonist Trevor, A. J., Katzung, B. G., & Kruidering-Hall,
M. (2018). Sympathomimetics. In Katzung &
Trevor’s Pharmacology Examination and Board
Review (12th ed., p. 78). New York, NY:
McGraw-Hill Education.

180
31. Which of the following Group 1 D D. Lidocaine
antiarrhythmics tend to shorten the
action potential in some cardiac Lidocaine belongs to Group 1B
tissues? antiarrhythmics and tend to shorten the
AP in some cardiac tissues.
A. Procainamide
B. Quinidine The rest of the options are under
C. Disopyramide Group 1A antiarrhythmics. They tend to
D. Lidocaine prolong the AP.

Reference:
Trevor, A. J., Katzung, B. G., & Kruidering-Hall,
M. (2018). Antiarrhythmic drugs. In Katzung &
Trevor’s Pharmacology Examination and Board
Review (12th ed., pp. 124-125). New York, NY:
McGraw-Hill Education.

32. Which of the following can occur in D D. Hyperglycemia


patients being given thiazides for
treatment of hypertension? Hyperglycemia, increased serum uric
acid, and lipid levels may occur in
A. Decreased serum uric acid patients taking thiazide diuretics.
B. Decreased serum lipid levels
C. Hypoglycemia Reference:
D. Hyperglycemia Trevor, A. J., Katzung, B. G., & Kruidering-Hall,
M. (2018). Diuretics & other drugs that act on
the kidney. In Katzung & Trevor’s
Pharmacology Examination and Board Review
(12th ed., p. 138). New York, NY: McGraw-Hill
Education.

33. Which of the following drugs is used C C. Fenofibrate


as treatment for hypertriglyceridemia?
Fenofibrate is a fibric acid derivative
A. Cholestyramine that serves as a ligand for the PPAR-
B. Ezetimibe alpha protein. It is used to treat
C. Fenofibrate hypertriglyceridemia.
D. Any of the above
Cholestyramine is a resin that is used
for hypercholesterolemia.

Ezetimibe is used for treatment of


hypercholesterolemia and
phytosterolemia.

Reference:
Trevor, A. J., Katzung, B. G., & Kruidering-Hall,
M. (2018). Agents used in dyslipidemia. In
Katzung & Trevor’s Pharmacology Examination
and Board Review (12th ed., p. 300). New York,
NY: McGraw-Hill Education.

34. Diabetes insipidus (DI) can occur in C C. Thiazides


patients given any of the following
drugs, EXCEPT: Thiazides do not cause nephrogenic
DI, but instead, are used in its
A. Lithium treatment.
B. Demeclocycline

181
C. Thiazides Lithium and demeclocycline may both
D. None of the above cause nephrogenic DI.

References:
● Trevor, A. J., Katzung, B. G., & Kruidering-
Hall, M. (2018). Diuretics & other drugs that
act on the kidney. In Katzung & Trevor’s
Pharmacology Examination and Board
Review (12th ed., p. 138). New York, NY:
McGraw-Hill Education.
● Trevor, A. J., Katzung, B. G., & Kruidering-
Hall, M. (2018). Antipsychotic agents &
lithium. In Katzung & Trevor’s
Pharmacology Examination and Board
Review (12th ed., p. 245). New York, NY:
McGraw-Hill Education.

35. The following drugs cause A A. Pilocarpine


mydriasis, EXCEPT?
Pilocarpine is a
A. Pilocarpine parasympathomimetic, therefore, it
B. Antihistamines causes decreased pupil size or miosis.
C. Cocaine
D. Meperidine Antihistamines and cocaine cause
mydriasis.

All opioids cause pupillary constriction


and contraction of biliary tract smooth
muscle, EXCEPT for meperidine.

Reference:
Le, T., Bhushan, V., & Sochat, M. (2023). First
Aid for the USMLE Step 1 2022, Thirty Third
Edition (33rd ed., p. 251). McGraw-Hill
Education.

36. The use of ethinyl estradiol without D D. None of the above


progesterone can predispose to which
of the following, EXCEPT? Use of ethinyl estradiol (without
progesterone) can predispose to:
A. Endometrial cancer ● Endometrial cancer
B. DVT ● Deep venous thrombosis
C. Hypertriglyceridemia ● Small risk of breast cancer and
D. None of the above cardiovascular events
● Increased migraine risk
● Hypertriglyceridemia
● Hypertension
Reference:
Trevor, A. J., Katzung, B. G., & Kruidering-Hall,
M. (2018). Gonadal hormones & inhibitors. In
Katzung & Trevor’s Pharmacology Examination
and Board Review (12th ed., p. 338). New York,
NY: McGraw-Hill Education

37. Which of the following is an A A. Thiopental


intravenous anesthetic used for
anesthesia induction for short surgical Thiopental is known to have high lipid

182
procedures and is characterized by solubility and therefore fast onset and
having a fast onset and short duration? short duration.

A. Thiopental Nitrous oxide and sevoflurane are


B. Nitrous oxide inhaled anesthetics, not intravenous.
C. Sevoflurane
D. Midazolam Midazolam has a slower onset but
longer duration than barbiturates.

Reference:
Trevor, A. J., Katzung, B. G., & Kruidering-Hall,
M. (2018). General anesthetics. In Katzung &
Trevor’s Pharmacology Examination and Board
Review (12th ed., p. 219). New York, NY:
McGraw-Hill Education.

38. Which of the following antiseizure A A. Carbamazepine


medications work by blocking voltage-
gated sodium channels? Carbamazepine works as an
antiseizure drug by blocking voltage-
A. Carbamazepine gated sodium channels in nerve
B. Levetiracetam membranes.
C. Gabapentin
D. Ethosuximide Gabapentin and ethosuximide block
voltage-gated calcium channels.

Levetiracetam blocks synaptic vesicle


protein 2A or SV2A.

Reference:
Trevor, A. J., Katzung, B. G., & Kruidering-Hall,
M. (2018). Antiseizure drugs. In Katzung &
Trevor’s Pharmacology Examination and Board
Review (12th ed., p. 205). New York, NY:
McGraw-Hill Education.

39. What is the preferred drug for D D. Ethosuximide


absence seizure which works by
blocking voltage-gated calcium Ethosuximide blocks voltage-gated
channels? calcium channels and is the preferred
drug for absence seizures.
A. Carbamazepine Another drug used also in absence
B. Levetiracetam seizure is valproic acid.
C. Gabapentin
D. Ethosuximide Reference:
Trevor, A. J., Katzung, B. G., & Kruidering-Hall,
M. (2018). Antiseizure drugs. In Katzung &
Trevor’s Pharmacology Examination and Board
Review (12th ed., p. 206). New York, NY:
McGraw-Hill Education.

40. All local anesthetics are A A. Cocaine


vasodilators, EXCEPT?
All local anesthetics are vasodilators
A. Cocaine except for cocaine.

183
B. Tetracaine
C. Procaine Reference:
D. Lidocaine Trevor, A. J., Katzung, B. G., & Kruidering-Hall,
M. (2018). Local anesthetics. In Katzung &
Trevor’s Pharmacology Examination and Board
Review (12th ed., p. 222). New York, NY:
McGraw-Hill Education.

41. Which of the following local A A. Bupivacaine


anesthetics cannot be used in patients
at risk for cardiac arrhythmias? Bupivacaine is associated with severe
cardiovascular toxicity and therefore
A. Bupivacaine should not be used in patients at risk
B. Lidocaine for cardiac arrhythmias.
C. Benzocaine
D. Prilocaine Reference:
Trevor, A. J., Katzung, B. G., & Kruidering-Hall,
M. (2018). Local anesthetics. In Katzung &
Trevor’s Pharmacology Examination and Board
Review (12th ed., p. 222). New York, NY:
McGraw-Hill Education.

42. What local anesthetic can lead to D D. Prilocaine


development of methemoglobinemia?
Prilocaine can be metabolized to
A. Bupivacaine products that convert hemoglobin to
B. Lidocaine methemoglobin.
C. Benzocaine
D. Prilocaine Reference:
Trevor, A. J., Katzung, B. G., & Kruidering-Hall,
M. (2018). Local anesthetics. In Katzung &
Trevor’s Pharmacology Examination and Board
Review (12th ed., p. 222). New York, NY:
McGraw-Hill Education.

43. Administration of phenylephrine D D. None of the above


(found in medications such as Neozep
Forte) can result to the following, Administration of phenylephrine can
EXCEPT: result in increased peripheral
resistance, increased mean BP and
A. Increased peripheral vascular reflex bradycardia mediated by the
resistance vagus nerve.
B. Increased mean BP
C. Decreased heart rate Reference:
D. None of the above Trevor, A. J., Katzung, B. G., & Kruidering-Hall,
M. (2018). Sympathomimetics. In Katzung &
Trevor’s Pharmacology Examination and Board
Review (12th ed., p. 79). New York, NY:
McGraw-Hill Education.

44. What is the most important anti- A A. Corticosteroids


inflammatory drug used in asthma
treatment? Inhaled corticosteroids are the most
important anti-inflammatory drugs used
A. Corticosteroids in asthma treatment.
B. Leukotriene antagonists
C. Theophylline Reference:

184
D. Beta2 agonists Trevor, A. J., Katzung, B. G., & Kruidering-Hall,
M. (2018). Drugs used in asthma & chronic
obstructive pulmonary disease. In Katzung &
Trevor’s Pharmacology Examination and Board
Review (12th ed., p. 173). New York, NY:
McGraw-Hill Education.

45. Which of the following treatments B B. Omalizumab


for asthma functions as a monoclonal
antibody to human IgE? Omalizumab is a monoclonal antibody
against IgE that is used in treatment of
A. Nedocromil severe asthma.
B. Omalizumab
C. Benralizumab Benralizumab is an antibody against
D. Prednisone IL5.

The rest of the options are not


antibodies.

Reference:
Trevor, A. J., Katzung, B. G., & Kruidering-Hall,
M. (2018). Drugs used in asthma & chronic
obstructive pulmonary disease. In Katzung &
Trevor’s Pharmacology Examination and Board
Review (12th ed., p. 177). New York, NY:
McGraw-Hill Education.

46. What drug used in asthma A A. Ipratropium


treatment works by blocking muscarinic
M3 receptors in the airways to prevent Ipratropium is a muscarinic antagonist
bronchoconstriction? that functions by blocking muscarinic
M3 receptors in the airways to prevent
A. Ipratropium bronchoconstriction.
B. Budesonide
C. Aminophylline Budesonide is an inhaled steroid.
D. Salbutamol
Aminophylline inhibits
phosphodiesterase to cause
bronchodilation.

Salbutamol is a beta2 agonist that


causes bronchodilation.

Reference:
Trevor, A. J., Katzung, B. G., & Kruidering-Hall,
M. (2018). Drugs used in asthma & chronic
obstructive pulmonary disease. In Katzung &
Trevor’s Pharmacology Examination and Board
Review (12th ed., p. 175). New York, NY:
McGraw-Hill Education.

47. Overdose of methylxanthines, such A A. Arrhythmias


as theophylline used in treatment of
asthma, may lead to _________. Overdose of methylxanthines, such as
theophylline used in treatment of
A. Arrhythmias asthma, may lead to arrhythmias and
B. Stroke seizures.
C. Worsening of asthma

185
D. Oral candidiasis Reference:
Trevor, A. J., Katzung, B. G., & Kruidering-Hall,
M. (2018). Drugs used in asthma & chronic
obstructive pulmonary disease. In Katzung &
Trevor’s Pharmacology Examination and Board
Review (12th ed., p. 175). New York, NY:
McGraw-Hill Education.

48. Azithromycin can be given to A A. Long half-life


patients with community-acquired
pneumonia using once-daily dosing due The half-life of azithromycin is >70
to its _________. hours. This permits its once-daily
dosing for treatment of patients with
A. Long half-life CAP.
B. Extensive drug interactions
C. Ability to inhibit cytochrome Reference:
P450 enzymes Trevor, A. J., Katzung, B. G., & Kruidering-Hall,
M. (2018). Chloramphenicol, tetracyclines,
D. All of the above macrolides, clindamycin, streptogramins, &
linezolid. In Katzung & Trevor’s Pharmacology
Examination and Board Review (12th ed., p.
383). New York, NY: McGraw-Hill Education.

49. The following macrolides inhibit C C. Azithromycin


cytochrome P450 enzymes which lead
to increased serum levels of various Azithromycin does not inhibit
drugs (such as warfarin), EXCEPT? cytochrome P450 enzymes, unlike
clarithromycin and erythromycin.
A. Erythromycin
B. Clarithromycin P 380
C. Azithromycin
D. None of the above Reference:
Trevor, A. J., Katzung, B. G., & Kruidering-Hall,
M. (2018). Chloramphenicol, tetracyclines,
macrolides, clindamycin, streptogramins, &
linezolid. In Katzung & Trevor’s Pharmacology
Examination and Board Review (12th ed., p.
380). New York, NY: McGraw-Hill Education.

50. What drug is used as primary B B. Azithromycin


prophylaxis against Mycobacterium
avium complex (MAC) in AIDS patients Azithromycin is used as primary
with CD4 counts < 50/uL? prophylaxis against Mycobacterium
avium complex (MAC) in AIDS patients
A. Amikacin with CD4 counts < 50/uL
B. Azithromycin
C. Isoniazid Reference:
D. Pyrazinamide Trevor, A. J., Katzung, B. G., & Kruidering-Hall,
M. (2018). Antimycobacterial drugs. In Katzung
& Trevor’s Pharmacology Examination and
Board Review (12th ed., p. 400). New York, NY:
McGraw-Hill Education.

51. What is the drug of choice for A A. Penicillin G


treatment of syphilis?
Penicillin G is still the drug of choice
A. Penicillin G for treatment of syphilis.
B. Ceftriaxone
C. Oxacillin Reference:
Trevor, A. J., Katzung, B. G., & Kruidering-Hall,

186
D. Azithromycin M. (2018). Beta-lactam antibiotics & other cell
wall synthesis inhibitors. In Katzung & Trevor’s
Pharmacology Examination and Board Review
(12th ed., p. 361). New York, NY: McGraw-Hill
Education.

52. The plasma half-life of levodopa is D D. Carbidopa


prolonged with the administration of
________. Carbidopa inhibits dopa
decarboxylase in peripheral tissues
A. Pramipexole which then prolongs levodopa’s serum
B. Apomorphine half-life.
C. Tolcapone
D. Carbidopa Reference:
Trevor, A. J., Katzung, B. G., & Kruidering-Hall,
M. (2018). Drugs used in Parkinsonism & other
movement disorders. In Katzung & Trevor’s
Pharmacology Examination and Board Review
(12th ed., p. 234). New York, NY: McGraw-Hill
Education.

53. What drug used in Parkinson A A. Tolcapone


disease acts by inhibiting the enzyme
that converts levodopa to 3-O- Tolcapone is a catechol-O-
methyldopa? methyltransferase (COMT) inhibitor.

A. Tolcapone COMT is an enzyme that converts


B. Selegiline levodopa to 3-O-methyldopa.
C. Levodopa-carbidopa
D. Bromocriptine Selegiline is a selective inhibitor of
MAO type B (enzyme that metabolizes
dopamine).

Bromocriptine is a partial agonist at


the dopamine D2 receptors in the
brain.

Carbidopa inhibits dopa


decarboxylase in peripheral tissues
which then prolongs levodopa’s serum
half-life.

Reference:
Trevor, A. J., Katzung, B. G., & Kruidering-Hall,
M. (2018). Drugs used in Parkinsonism & other
movement disorders. In Katzung & Trevor’s
Pharmacology Examination and Board Review
(12th ed., pp. 235-236). New York, NY:
McGraw-Hill Education.

54. Hyperprolactinemia is a side effect C C. Risperidone


of which of the following drugs?
Dopamine tonically inhibits the
A. Bromocriptine
B. Cabergoline secretion of prolactin from the anterior
C. Risperidone pituitary.
D. All of the above

187
Dopamine receptor blockade removes
the tonic inhibition of dopamine →
increased prolactin secretion.

Risperidone is a dopamine receptor


blocker which results in elevated
prolactin secretion.

Cabergoline and bromocriptine are


dopamine agonists. They inhibit
prolactin secretion.

Reference:
● Trevor, A. J., Katzung, B. G., & Kruidering-
Hall, M. (2018). Antidepressants. In
Katzung & Trevor’s Pharmacology
Examination and Board Review (12th ed.,
p. 244). New York, NY: McGraw-Hill
Education.
● Le, T., Bhushan, V., & Sochat, M. (2023).
First Aid for the USMLE Step 1 2022, Thirty
Third Edition (33rd ed., p. 334). McGraw-
Hill Education.

55. Which of the following is used in A A. Prazosin


benign prostatic hyperplasia and acts to
reduce prostatic smooth muscle tone? Prazosin is a selective alpha1 blocker
used in BPH which reduces prostatic
A. Prazosin smooth muscle tone.
B. Finasteride
C. Dutasteride B and C are 5alpha-reductase
D. All of the above inhibitors (also used in BPH) which
reduce the conversion of testosterone
to dihydrotestosterone.
Reference:
Trevor, A. J., Katzung, B. G., & Kruidering-Hall,
M. (2018). Drugs used in hypertension. In
Katzung & Trevor’s Pharmacology Examination
and Board Review (12th ed., p. 101). New York,
NY: McGraw-Hill Education.

56. What drug used in acute B B. Milrinone


decompensated heart failure to
increase myocardial contractility works Milrinone is a PDE3 inhibitor that
by increasing cAMP through inhibition
of PDE-3? increases cAMP → increased
myocardial contractility.
A. Dobutamine
It is used in patients with acute
B. Milrinone
decompensated HF.
C. Dopamine
D. Norepinephrine Reference:
Le, T., Bhushan, V., & Sochat, M. (2023). First
Aid for the USMLE Step 1 2022, Thirty Third
Edition (33rd ed., p. 245). McGraw-Hill
Education.

188
57. Which of the following drugs inhibits A A. Cilostazol
platelet aggregation by increasing
levels of platelet cAMP? Cilostazol inhibits phosphodiesterase
A. Cilostazol enzymes → increased platelet cAMP →
B. Heparin inhibition of platelet aggregation
C. Enoxaparin
D. Warfarin
Heparin activates antithrombin III
which leads to inactivation of thrombin
and factor Xa.

Warfarin inhibits Vitamin K epoxide


reductase (VKOR) leading to
decreased production of Vitamin-K
dependent clotting factors.

Reference:
Trevor, A. J., Katzung, B. G., & Kruidering-Hall,
M. (2018). Drugs used in coagulation disorders.
In Katzung & Trevor’s Pharmacology
Examination and Board Review (12th ed., p.
290). New York, NY: McGraw-Hill Education.

58. What drug can be used in the A A. Cilostazol


treatment of peripheral arterial disease
(PAD)? Cilostazol is used in the management
of intermittent claudication in patients
A. Cilostazol with PAD.
B. Heparin
C. Enoxaparin Reference:
D. Warfarin Trevor, A. J., Katzung, B. G., & Kruidering-Hall,
M. (2018). Drugs used in coagulation disorders.
In Katzung & Trevor’s Pharmacology
Examination and Board Review (12th ed., p.
290). New York, NY: McGraw-Hill Education.

59. Which of the following is NOT a C C. Aztreonam


characteristic of Aztreonam?
Aztreonam has NO cross-allergenicity
A. No activity against Gram (+) with penicillins.
bacteria
B. No activity against anaerobes Other characteristics of Aztreonam:
C. Has cross-allergenicity with ● No activity against Gram (+)
penicillins bacteria
D. None of the above ● No activity against anaerobes
● Synergistic with
aminoglycosides

Reference:
Trevor, A. J., Katzung, B. G., & Kruidering-Hall,
M. (2018). Beta-lactam antibiotics & other cell
wall synthesis inhibitors. In Katzung & Trevor’s
Pharmacology Examination and Board Review
(12th ed., p. 371). New York, NY: McGraw-Hill
Education.

189
60. Rapid infusion of vancomycin may A A. Red man syndrome
lead to _________.
Red man syndrome (diffuse cutaneous
A. Red man syndrome flushing) results from rapid IV infusion
B. Jarisch-Herxheimer syndrome of vancomycin.
C. Cyanosis
D. All of the above Reference:
Trevor, A. J., Katzung, B. G., & Kruidering-Hall,
M. (2018). Beta-lactam antibiotics & other cell
wall synthesis inhibitors. In Katzung & Trevor’s
Pharmacology Examination and Board Review
(12th ed., p. 372). New York, NY: McGraw-Hill
Education.

61. Which of the following drugs that C C. Denosumab


affect bone mineral metabolism can be
administered subcutaneously? Denosumab prevents the binding of
RANKL inhibition of osteoclast
A. Alendronate differentiation and function\
B. Raloxifene
C. Denosumab Among the choices, Denosumab is
administered subcutaneously (SQ).
D. Cinacalcet
Reference:
Trevor, A. J., Katzung, B. G., & Kruidering-Hall,
M. (2018). Drugs that affect bone mineral
homeostasis. In Katzung & Trevor’s
Pharmacology Examination and Board Review
(12th ed., p. 365). New York, NY: McGraw-Hill
Education.

62. Which drug used in severe plaque A A. Secukinumab


psoriasis is a human IgG1 monoclonal
antibody that prevents the binding of IL- Secukinumab is a human IgG1
17A to IL-17A receptor? monoclonal antibody that prevents the
binding of IL-17A to IL-17A receptor. It
A. Secukinumab is used in severe plaque psoriasis.
B. Infliximab
C. Methotrexate Infliximab is an anti-TNF alpha
monoclonal antibody.
D. Tocilizumab
Methotrexate is a dihydrofolate
reductase inhibitor.

Tocilizumab is anti-IL6.

References:
● Le, T., Bhushan, V., & Sochat, M. (2023).
First Aid for the USMLE Step 1 2022, Thirty
Third Edition (33rd ed., p. 444, 497).
McGraw-Hill Education.
● Katzung, B. G. (2017). NSAIDs,
antirheumatic drugs, nonopioid analgesics,
& drugs used in gout. In Basic and Clinical
Pharmacology (14th ed., p. 656). McGraw-
Hill Education.

190
63. What is the DMARD of first choice A A. Methotrexate
of the treatment of early rheumatoid
arthritis? The DMARD of first choice for early RA
treatment is Methotrexate.
A. Methotrexate
B. Sulfasalazine Reference:
Loscalzo J, & Fauci A, & Kasper D, & Hauser S,
C. Hydroxychloroquine & Longo D, & Jameson J. (2022). Rheumatoid
D. Rituximab Arthritis. In A. Shah & E.W. St. Clair (Eds.),
Harrison's Principles of Internal Medicine (21st
ed., Vol. 2, p. 11677). New York, NY: McGraw
Hill.

64. Which of the following drugs used in B B. Colchicine


Gout acts as an inhibitor of microtubule
assembly leading to decreased Colchicine decreases leukocyte
migration of leukocytes? migration and phagocytosis by acting
as a selective inhibitor of microtubule
A. Allopurinol assembly.
B. Colchicine
C. Probenecid Probenecid is a uricosuric drug.
D. Febuxostat
Allopurinol and febuxostat are
xanthine oxidase inhibitors.

Reference:
Trevor, A. J., Katzung, B. G., & Kruidering-Hall,
M. (2018). NSAIDs, acetaminophen, & drugs
used in rheumatoid arthritis & gout. In Katzung
& Trevor’s Pharmacology Examination and
Board Review (12th ed., p. 309). New York, NY:
McGraw-Hill Education.

65. A patient was referred to you for C C. Desmopressin


passing large amounts of urine. The
urine specific gravity was noted to be This is a case of central diabetes
1.001 while the serum osmolality is insipidus.
increased. He was in a car accident last
week and is now admitted in the ICU. Clues to the diagnosis:
● Had a possible brain injury from the
Which of the following is the most car accident
appropriate treatment? ● Passing large amounts of urine with
decreased urine SG
A. Hypertonic saline
B. Thiazide diuretics The treatment for central DI is
C. Desmopressin Desmopressin.
D. Water restriction
Hypertonic saline is treatment for
SIADH.

Water restriction is for primary


polydipsia.

Thiazide diuretics are used in


nephrogenic DI.

Reference:

191
Le, T., Bhushan, V., & Sochat, M. (2023). First
Aid for the USMLE Step 1 2022, Thirty Third
Edition (33rd ed., p. 342). McGraw-Hill
Education.

66. A patient was documented to have B B. Ethacrynic acid


had an allergic reaction to sulfonamide
antibiotics in the past. Ethacrynic acid is a non-sulfonamide
diuretic and can be used in cases of
The following drugs should be avoided sulfa allergy.
in this patient, EXCEPT?
Patients with sulfa allergies should
A. Thiazides avoid sulfonamide antibiotics and other
B. Ethacrynic acid chemically related sulfa drugs such as:
C. Furosemide ● Sulfasalazine
D. Sulfonylureas ● Probenecid
● Furosemide
● Acetazolamide
● Celecoxib
● Thiazides
● Sulfonylureas

References:
● Le, T., Bhushan, V., & Sochat, M. (2023).
First Aid for the USMLE Step 1 2022, Thirty
Third Edition (33rd ed., p. 251). McGraw-
Hill Education.
● Trevor, A. J., Katzung, B. G., & Kruidering-
Hall, M. (2018). Key words for key drugs. In
Katzung & Trevor’s Pharmacology
Examination and Board Review (12th ed.,
p. 527). New York, NY: McGraw-Hill
Education.

67. Which of the following can be used B B. Tolvaptan


as a treatment in patients with SIADH
that functions as an ADH antagonist? Tolvaptan is an antagonist of
vasopressin receptors.
A. Hydrochlorothiazide
B. Tolvaptan Desmopressin is an agonist of
C. Desmopressin vasopressin receptors.
D. Amiloride Reference:
Trevor, A. J., Katzung, B. G., & Kruidering-Hall,
M. (2018). NSAIDs, acetaminophen, & drugs
used in rheumatoid arthritis & gout. In Katzung
& Trevor’s Pharmacology Examination and
Board Review (12th ed., p. 323). New York, NY:
McGraw-Hill Education.

192
68. Continuous administration of GnRH A A. Leuprolide
agonists is used in patients in advanced
prostate cancer. Leuprolide is a GnRH agonist.
Continuous administration of GnRH
Which of the following is considered a agonists is used in patients in
GnRH agonist? advanced prostate cancer.

A. Leuprolide Degarelix is a GnRH antagonist.


B. Degarelix
Reference:
C. Clomiphene Le, T., Bhushan, V., & Sochat, M. (2023). First
D. Anastrozole Aid for the USMLE Step 1 2022, Thirty Third
Edition (33rd ed., p. 676). McGraw-Hill
Education.

69. Continuous administration of GnRH D D. Increased hematocrit


agonists is used in patients with
advanced prostate cancer. The answer should be decreased
hematocrit NOT increased hematocrit.
This can result in the following adverse
effects, EXCEPT? Adverse effects of continuous
treatment with a GnRH agonist:
A. Hot flushes
B. Gynecomastia In men
C. Reduced bone density ● Hot flushes
● Sweats
D. Increased hematocrit
● Gynecomastia
● Reduced libido
● Decreased hematocrit
● Reduced bone density

In women
● Typical symptoms of menopause
(hot flushes, sweats, headache)
● Risk of bone loss and
osteoporosis (long-term
treatment)

Reference:
Trevor, A. J., Katzung, B. G., & Kruidering-Hall,
M. (2018). Hypothalamic & pituitary hormones.
In Katzung & Trevor’s Pharmacology
Examination and Board Review (12th ed., p.
318). New York, NY: McGraw-Hill Education.

70. Methimazole use during pregnancy A A. Aplasia cutis


is associated with which of the following
teratogenic effects? Methimazole is potentially teratogenic
and can cause aplasia cutis in the
A. Aplasia cutis newborn.
B. Thyroglossal duct cyst
Reference:
C. Agranulocytosis Le, T., Bhushan, V., & Sochat, M. (2023). First
D. All of the above Aid for the USMLE Step 1 2022, Thirty Third
Edition (33rd ed., p. 360). McGraw-Hill
Education.

193
71. What drug can be used in von B B. Desmopressin
Willebrand disease to release vWF from
endothelium stores? Desmopressin is used in vWD to
release von Willebrand factor from
A. Demeclocycline endothelium stores.
B. Desmopressin
C. Dexamethasone Demeclocycline is an ADH
antagonist.
D. Doxycycline
Reference:
Le, T., Bhushan, V., & Sochat, M. (2023). First
Aid for the USMLE Step 1 2022, Thirty Third
Edition (33rd ed., p. 433). McGraw-Hill
Education.

72. Misoprostol is sometimes used in A A. Increasing mucosal protection


patients with risk of developing ulcers
such as those taking NSAIDs. It acts by Misoprostol is an analog of PGE1 and
__________. increases mucosal protection.

A. Increasing mucosal protection B – MoA of PPIs


B. Irreversibly inactivating H+/K+
ATPase in the parietal cell C – MoA of antacids
C. Neutralizing stomach acid
D. Acting as H2 receptor D – MoA of H2 antagonists
antagonists
Reference:
Trevor, A. J., Katzung, B. G., & Kruidering-Hall,
M. (2018). Drugs used in gastrointestinal
disorders. In Katzung & Trevor’s Pharmacology
Examination and Board Review (12th ed., pp.
498-499). New York, NY: McGraw-Hill
Education.

73. Which of the following drugs used in C C. Teriparatide


patients with osteoporosis increases
osteoblastic activity by acting as PTH Teriparatide increases osteoblastic
analog? activity by acting as a recombinant
PTH analog. It used in patients with
A. Bisphosphonates osteoporosis.
B. Strontium
C. Teriparatide Reference:
D. Glucocorticoids Trevor, A. J., Katzung, B. G., & Kruidering-Hall,
M. (2018). Keywords for key drugs. In Katzung
& Trevor’s Pharmacology Examination and
Board Review (12th ed., p. 496). New York, NY:
McGraw-Hill Education.

74. Amphotericin B acts as an B B. Ergosterol


antifungal by creating artificial pores in
the fungal cell membrane by binding to Amphotericin B acts as an antifungal
__________. by creating artificial pores in the fungal
cell membrane by binding to ergosterol
A. Cholesterol (sterol specific to fungal cell
B. Ergosterol membranes).
C. Lanosterol
D. Squalene Reference:
Trevor, A. J., Katzung, B. G., & Kruidering-Hall,

194
M. (2018). Antifungal agents. In Katzung &
Trevor’s Pharmacology Examination and Board
Review (12th ed., p. 404). New York, NY:
McGraw-Hill Education.

75. What drug used in A A. Griseofulvin


dermatophytoses acts by interfering
with microtubule function? Griseofulvin is used in
dermatophytoses and acts by
A. Griseofulvin interfering with microtubule function.
B. Terbinafine
C. Amphotericin B Azoles inhibit the synthesis of
D. Ketoconazole ergosterol interfering with permeability
of the fungal cell membrane.

Amphotericin B acts as an antifungal


by creating artificial pores in the fungal
cell membrane by binding to ergosterol
(sterol specific to fungal cell
membranes).

Terbinafine inhibits squalene


epoxidase to interfere with the
synthesis of ergosterol.

Reference:
Trevor, A. J., Katzung, B. G., & Kruidering-Hall,
M. (2018). Antifungal agents. In Katzung &
Trevor’s Pharmacology Examination and Board
Review (12th ed., pp. 406-407). New York, NY:
McGraw-Hill Education.

76. Which of the following A A. Ertapenem


carbapenems is not active against
Pseudomonas? Carbapenems are active against
Pseudomonas except for ertapenem.
A. Ertapenem
B. Meropenem Reference:
C. Imipenem Trevor, A. J., Katzung, B. G., & Kruidering-Hall,
M. (2018). Beta-lactam antibiotics & other cell
D. None of the above wall synthesis inhibitors. In Katzung & Trevor’s
Pharmacology Examination and Board Review
(12th ed., p. 372). New York, NY: McGraw-Hill
Education.

77. What is the suggested treatment for A A. Ceftriaxone


gonorrhea?
The treatment for gonorrhea is single-
A. Ceftriaxone dose IM ceftriaxone.
B. Amoxicillin
C. Meropenem Reference:
D. Piperacillin tazobactam Le, T., Bhushan, V., & Sochat, M. (2023). First
Aid for the USMLE Step 1 2022, Thirty Third

195
Edition (33rd ed., p. 140). McGraw-Hill
Education.

78. Which of the following antibiotics D D. Fluoroquinolones


can cause tendon rupture in patients
>60 years old? In patients > 60 years old, tendon
rupture or tendinitis may occur with the
A. Daptomycin use of fluoroquinolones.
B. Metronidazole
C. Penicillins Reference:
D. Fluoroquinolones Trevor, A. J., Katzung, B. G., & Kruidering-Hall,
M. (2018). Sulfonamides, trimethoprim, &
fluoroquinolones. In Katzung & Trevor’s
Pharmacology Examination and Board Review
(12th ed., p. 385). New York, NY: McGraw-Hill
Education.

79. Overdose of which of the following C C. TCAs


drugs can manifest with prolonged QT,
convulsions, and coma? TCA overdose may predispose the
patient to prolonged QT, coma, and
A. SSRIs convulsions.
B. Lithium
C. TCAs SSRI overdose may predispose to
D. Any of the above serotonin syndrome manifested by
neuromuscular hyperexcitabilty,
autonomic instability, and altered
mental status.

Lithium overdose may lead to


nausea, vomiting, hyperreflexia,
seizures, ataxia, and nephrogenic DI.

Reference:
Le, T., Bhushan, V., & Sochat, M. (2023). First
Aid for the USMLE Step 1 2022, Thirty Third
Edition (33rd ed., p. 589). McGraw-Hill
Education.

80. What is considered as the single A A. Isoniazid


most important drug used in TB?
The single most important drug in TB,
A. Isoniazid according to Katzung, is Isoniazid.
B. Rifampin
C. Pyrazinamide Reference:
D. Ethambutol Trevor, A. J., Katzung, B. G., & Kruidering-Hall,
M. (2018). Antimycobacterial drugs. In Katzung
& Trevor’s Pharmacology Examination and
Board Review (12th ed., p. 398). New York, NY:
McGraw-Hill Education.

196
81. Which of the following anti-TB drugs B B. Rifampin
can increase the rate of elimination of
certain drugs such as contraceptives? Isoniazid may inhibit drug-metabolizing
enzymes.
A. Isoniazid
B. Rifampin While rifampin tends to induce drug-
C. Pyrazinamide metabolizing enzymes or increase the
D. Ethambutol rate of elimination of certain drugs.

Reference:
Trevor, A. J., Katzung, B. G., & Kruidering-Hall,
M. (2018). Antimycobacterial drugs. In Katzung
& Trevor’s Pharmacology Examination and
Board Review (12th ed., p. 398). New York, NY:
McGraw-Hill Education.

82. What drug used in heart failure C C. Digoxin


increases cardiac contractility by
inhibiting the Na+/K+ ATPase pump? Digoxin increases cardiac contractility
A. Dobutamine by blocking the Na+/K+ ATPase pump
B. Norepinephrine → increased intracellular Ca2+ →
C. Digoxin
D. None of the above increased cardiac contractility.

Dobutamine is a B1-selective agonist


that increases cardiac contractility.

Reference:
Trevor, A. J., Katzung, B. G., & Kruidering-Hall,
M. (2018). Drugs used in heart failure. In
Katzung & Trevor’s Pharmacology Examination
and Board Review (12th ed., p. 120). New York,
NY: McGraw-Hill Education.

83. Which of the following conditions D D. All of the above


can be exacerbated by the
administration of metoprolol? All of the conditions in the options can
be exacerbated by metoprolol.
A. Asthma
B. Vasospastic angina Reference:
C. COPD Le, T., Bhushan, V., & Sochat, M. (2023). First
Aid for the USMLE Step 1 2022, Thirty Third
D. All of the above Edition (33rd ed., p. 327). McGraw-Hill
Education.

84. Administration of typical A A. Benztropine


antipsychotics can precipitate an
episode of acute dystonia. Which of the Acute dystonias can be treated with
following can be given to treat this benztropine (muscarinic receptor
condition? blocker) or diphenhydramine.

A. Benztropine Cyproheptadine can be used in


B. Phentolamine serotonin syndrome.
C. Lithium
D. Cyproheptadine References:
● Le, T., Bhushan, V., & Sochat, M. (2023).
First Aid for the USMLE Step 1 2022, Thirty

197
Third Edition (33rd ed., p. 589). McGraw-
Hill Education.
● Trevor, A. J., Katzung, B. G., & Kruidering-
Hall, M. (2018). Drugs used in
Parkinsonism & other movement disorders.
In Katzung & Trevor’s Pharmacology
Examination and Board Review (12th ed.,
p. 237). New York, NY: McGraw-Hill
Education.

85. What can be administered to A A. Mesna


reduce the incidence of hemorrhagic
cystitis in patients being given Giving mesna to patients being given
cyclophosphamide? cyclophosphamide can reduce the
chance of developing hemorrhagic
A. Mesna cystitis by trapping the acrolein
B. Dexrazoxane released from cyclophosphamide.
C. Leucovorin
D. Folic acid Dexrazoxane protects against
cardiotoxicity from doxorubicin.

Leucovorin rescues normal cells after


patient receives high doses of
methotrexate.

Reference:
Trevor, A. J., Katzung, B. G., & Kruidering-Hall,
M. (2018). Cancer chemotherapy. In Katzung &
Trevor’s Pharmacology Examination and Board
Review (12th ed., p. 452). New York, NY:
McGraw-Hill Education.

86. Which of the following rescue B B. Dexrazoxane


therapies can be given to patients
receiving anthracyclines to reduce the Dexrazoxane protects against
chance of developing cardiotoxicity? cardiotoxicity from anthracyclines such
as doxorubicin.
A. Mesna
B. Dexrazoxane Leucovorin rescues normal cells after
C. Leucovorin patient receives high doses of
D. Folic acid methotrexate.

Reference:
Trevor, A. J., Katzung, B. G., & Kruidering-Hall,
M. (2018). Cancer chemotherapy. In Katzung &
Trevor’s Pharmacology Examination and Board
Review (12th ed., p. 452). New York, NY:
McGraw-Hill Education.

87. Which of the following anti-ischemic C C. Trimetazidine


drugs act as partial fatty acid oxidation
inhibitors? Trimetazidine is a partial fatty acid
oxidation inhibitor that shifts the energy
A. Metoprolol preference of the heart to glucose from
B. Verapamil fatty acids.
C. Trimetazidine
Reference:

198
D. Ivabradine Trevor, A. J., Katzung, B. G., & Kruidering-Hall,
M. (2018). Drugs used in treatment of angina
pectoris. In Katzung & Trevor’s Pharmacology
Examination and Board Review (12th ed., p.
105). New York, NY: McGraw-Hill Education.

88. Which of the following is A A. Sildenafil


contraindicated when taking
nitroglycerin? Sildenafil is a PDE- inhibitor which
causes vasodilation.
A. Sildenafil Coadministration of sildenafil with
B. Metoprolol nitrates may precipitate severe
C. Losartan hypotension.
D. Atorvastatin
Reference:
Le, T., Bhushan, V., & Sochat, M. (2023). First
Aid for the USMLE Step 1 2022, Thirty Third
Edition (33rd ed., p. 707). McGraw-Hill
Education.

89. Which of the following is an opioid A A. Dextromethorphan


found in cough syrups that act to
suppress debilitating cough? Dextromethorphan is a weak μ
receptor agonist that acts as an
A. Dextromethorphan antitussive in cough syrups.
B. Fentanyl
C. Morphine Reference:
D. Phenylephrine Trevor, A. J., Katzung, B. G., & Kruidering-Hall,
M. (2018). Opioid analgesics & antagonists. In
Katzung & Trevor’s Pharmacology Examination
and Board Review (12th ed., p. 265). New York,
NY: McGraw-Hill Education.

90. Which of the following is NOT A A. Highly emetogenic


associated with Propofol?
Propofol has antiemetic actions.
A. Highly emetogenic It is an IV anesthetic that can cause
B. May cause hypotension during marked decrease in BP during
anesthesia induction anesthesia induction.
C. Administered IV
D. None of the above Reference:
Trevor, A. J., Katzung, B. G., & Kruidering-Hall,
M. (2018). General anesthetics. In Katzung &
Trevor’s Pharmacology Examination and Board
Review (12th ed., p. 215). New York, NY:
McGraw-Hill Education.

91. What antimuscarinic drug used in B B. Benztropine


Parkinson disease can improve
symptoms of rigidity and tremors but Benztropine is an antimuscarinic drug
has minimal effect on bradykinesia? used in Parksinson disease that can
improve symptoms of rigidity and
A. Amantadine tremors but has minimal effect on
B. Benztropine bradykinesia.
C. Entacapone

199
D. Selegiline Reference:
Le, T., Bhushan, V., & Sochat, M. (2023). First
Aid for the USMLE Step 1 2022, Thirty Third
Edition (33rd ed., p. 565). McGraw-Hill
Education.

92. A patient taking fluoxetine suddenly A A. Tramadol


develops neuromuscular
hyperexcitability, autonomic instability, This is a case of serotonin syndrome.
and agitation and hyperthermia.
The most likely precipitant drug is
What drug given to the patient most tramadol.
likely precipitated his current condition?
Other drugs that can precipitate
A. Tramadol serotonin syndrome are the following:
B. Paracetamol ● Any drug that increases 5-HT
C. Metoprolol ● Psychiatric drugs
D. All of the above o MAO inhibitors
o SSRIs
o SNRIs
o TCAs
o Vilazodone
o Vortioxetine
o Buspirone
● Nonpsychiatric drugs
o Tramadol
o Ondansetron
o Triptans
o Linezolid
o MDMA
o Dextromethorphan
o Meperidine
o St. John’s wort

Reference:
Le, T., Bhushan, V., & Sochat, M. (2023). First
Aid for the USMLE Step 1 2022, Thirty Third
Edition (33rd ed., p. 589). McGraw-Hill
Education.

93. Which of the following is NOT a D D. None of the above


characteristic of Tramadol?
All are true of tramadol.
A. Weak mu receptor agonist
B. Blocks reuptake of serotonin It is a weak mu receptor agonist
C. Should not be given in patients It blocks reuptake of serotonin (which
with history of seizure is the reason why it can precipitate
D. None of the above serotonin syndrome in patients taking
SSRIs)
It should not be given in patients with
history of seizure.

Reference:
Trevor, A. J., Katzung, B. G., & Kruidering-Hall,
M. (2018). Opioid analgesics & antagonists. In
Katzung & Trevor’s Pharmacology Examination
and Board Review (12th ed., pp. 262-262). New

200
York, NY: McGraw-Hill Education.

94. A child who developed symptoms of B B. Reye syndrome


chickenpox over the last few days was
given an unrecalled drug for his fever. Reye syndrome is a rare disease that
can occur when aspirin is used to treat
He then developed hepatic a viral infection (e.g., VZV and
encephalopathy with vomiting and influenza).
episodes of hypoglycemia.
This can lead to symptoms of hepatic
What is the most likely diagnosis? encephalopathy such as coma,
vomiting, hypoglycemia, and
A. Viral hepatitis A hepatomegaly.
B. Reye syndrome
C. Wilson disease Reference:
D. Acute Hepatitis B Le, T., Bhushan, V., & Sochat, M. (2023). First
Aid for the USMLE Step 1 2022, Thirty Third
Edition (33rd ed., p. 397). McGraw-Hill
Education.

95. What is the most likely cause of the B B. Administering aspirin


above patient case?
Reye syndrome is a rare disease that
A. Administering paracetamol can occur when aspirin is used to treat
B. Administering aspirin a viral infection (e.g., VZV and
C. Exposure to a virus influenza).
D. Autosomal recessive mutation
This can lead to symptoms of hepatic
encephalopathy such as coma,
vomiting, hypoglycemia, and
hepatomegaly.

Wilson disease is an autosomal


recessive mutation.

Reference:
Le, T., Bhushan, V., & Sochat, M. (2023). First
Aid for the USMLE Step 1 2022, Thirty Third
Edition (33rd ed., p. 397). McGraw-Hill
Education.

96. Ceftaroline, a 5th generation A A. Pseudomonas


cephalosporin, has no coverage
against? Ceftaroline has no coverage against
Pseudomonas but covers MRSA and E
A. Pseudomonas faecalis.
B. MRSA
C. Enterococcus Reference:
D. None of the above Le, T., Bhushan, V., & Sochat, M. (2023). First
Aid for the USMLE Step 1 2022, Thirty Third
Edition (33rd ed., p. 186). McGraw-Hill
Education.

201
97. Which of the following B B. Imipenem
carbapenems should always be
administered with cilastatin? Imipenem should always be
administered with cilastatin to increase
A. Meropenem its plasma half-life through the
B. Imipenem inhibition of dehydropeptidase I (which
C. Ertapenem inactivates imipenem) present in the
D. Doripenem kidney.

Reference:
Trevor, A. J., Katzung, B. G., & Kruidering-Hall,
M. (2018). Beta-lactam antibiotics & other cell
wall synthesis inhibitors. In Katzung & Trevor’s
Pharmacology Examination and Board Review
(12th ed., p. 372). New York, NY: McGraw-Hill
Education.

98. Which of the following drugs can kill A A. Aminoglycosides


more bacteria as its plasma level
increases above its minimal inhibitory Aminoglycosides exhibit what’s called
concentration (MIC)? concentration-dependent killing action
wherein there’s an increasing number
A. Aminoglycosides of bacteria killed as the plasma level of
B. Cephalosporins the drug increases above the MIC.
C. Penicillins
D. All of the above Cephalosporins and penicillins
exhibit time-dependent killing action
where the effectiveness is related to
the time above the MI, independent of
the concentration.

Reference:
Trevor, A. J., Katzung, B. G., & Kruidering-Hall,
M. (2018). Aminoglycosides. In Katzung &
Trevor’s Pharmacology Examination and Board
Review (12th ed., p. 385). New York, NY:
McGraw-Hill Education.

99. Which of the following antibiotics A A. Amikacin


function by binding to the 30S subunit?
Amikacin is an aminoglycoside. It is
A. Amikacin bactericidal and functions by inhibition
B. Clindamycin of the 30S subunit.
C. Erythromycin
D. Linezolid The rest of the choices are considered
50S inhibitors.

Reference:
Le, T., Bhushan, V., & Sochat, M. (2023). First
Aid for the USMLE Step 1 2022, Thirty Third
Edition (33rd ed., p. 188). McGraw-Hill
Education.

202
100. What is the drug of first choice in C C
the prophylaxis against P. jiroveci in
patients with AIDS? TMP-SMZ is the drug of first choice in
the prophylaxis against P. jiroveci in
A. Pentamidine patients with AIDS.
B. Atovaquone
C. Trimethoprim-sulfamethoxazole Reference:
D. Melarsoprol Trevor, A. J., Katzung, B. G., & Kruidering-Hall,
M. (2018). Sulfonamides, trimethoprim, &
fluoroquinolones. In Katzung & Trevor’s
Pharmacology Examination and Board Review
(12th ed., p. 386). New York, NY: McGraw-Hill
Education.

203

You might also like